MRCS Recalls Q and Ans Combined

You might also like

Download as pdf or txt
Download as pdf or txt
You are on page 1of 226

Q1: Tumor marker for Testicular teratoma?

ANS: AFP (correct)

Q2: Numbness of the lower lips, which nerve involved?

ANS: Mental

Q3: DM patient undergoing angiography. Which drug causes LACTIC ACIDOSIS?

ANS: metformin (correct)

Q4: Malignant melanoma. Which one is the good prognostic factor?

ANS: Breslow’s skin thickness (correct)

Q5: Statistics question asking what is proportion of test positive from all
positive?

ANS: Sensitivity

Q6: There has been ongoing audit regarding what is done to the patient. What
type of audit?

ANS: Outcome audit

Q7: A patient is being admitted for scheduled elective surgery. He has been found
to have nasal MRSA positive. What to do?

ANS: Mupirocin ointment X 5 days (correct)

Q8: A patient is shifted to ward after laparotomy. Blood culture is positive for
Gram +ve cocci. He is asymptomatic. But another patient in the ward has MRSA
infection. What Rx would you give?

ANS: Oral vancomycin

Q9: Femoral Hernia Q – What is behind the neck of the hernia sac

ANS: Pectineal ligament (correct)

Q10: A clinical vignette describing a Peutz-Jeghers syndrome. What type of polyp


is found?

ANS: Hamartomatous (correct)


Q11: Patient is found to have extreme hypothermia. Which of the following be
best management for rewarming the pt.?

ANS: Intraperitoneal fluid infusion (correct)

Q12: What is method to loss temperature during laparotomy?

ANS: radiation

Q13: External spermatic fascia of the cord is derived from which structure?

ANS: External oblique aponeurosis

Q14: A Clinical Vignette describing a nodule of 3 cm size in the region of elbow in


the extensor surface. Biopsy showed necrobiotic granulomatous inflammation.
Dx?

ANS: Rheumatoid nodule (answer)

Q15: A clinical scenario of polyuria, polydipsia with 11.3 mmol/L of blood sugar.
Dx?

ANS: Diabetes Mellitus (answer)

Q16: During cholecystectomy, pt. develops extreme bradycardia. Past history of


heart transplant present. Which drug to give?

ANS: Glycopyrrolate

Q17: Old lady with Colle’s fracture. She has CKD and IHD.

ANS: x-ray re-do after two weeks

Q18: Nerve emerging from lateral border of psoas major cause numbness to
lateral thigh

ANS: Meralgia paraesthetica

Q19: Neonate with cyanotic heart disease. Dx?

ANS: TGA

Q20: Wound healing one week? What process is undergoing?


ANS: Angiogenesis

Q21: To estimate GFR, which substance is used?

ANS: Inulin

Q22: A clinical vignette easily diagnosed as TB. Lymph node was excised. Which
of the following cells are in the periphery of this lesion seen in the
histopathology?

ANS: Macrophage

Q23: Type of amyloid in Renal failure

ANS: AL

Q24: Which of the following is the 1st branch of internal carotid as soon as it
leaves the cavernous sinus?

ANS: Ophthalmic Artery (correct)

Q25: Badly infected wound with crepitus. Dx?

ANS: Clostridium perfringens

Q26: Rose pink rash around cellulitic wound which was also indurated. Which
bacteria?

ANS: S. pyogenes

Q27: Pt undergoes graft transplantation (CABG) and after one week his wound
secretion was cultured contain g+ve cocci bacteria what is cause of his wound
infection?

ANS: Staph epidemis

Q28: Surgeon expose to needle stick and pt is HBV+VE what anti gene that mean
this pt in risky period?

ANS: HBVeAg

Q29: 60% burn case with bilateral leg edema. Cause?

ANS: Hypoalbuminemia
Q30: A clinical scenario of hypercalcemia. What is the first step in Rx?

ANS: Rehydration

Q31: A child with audible bowel sound in left chest. What is the congenital defect?

ANS: Failure to closure of pleuroperitoneal membrane

Q32: A 3-day old neonate had abnormal meconium stain in the infra-umbilical
part of the diaper. Dx?

ANS: Patent vitello-intestinal duct

Q33: Which of the following serves as the axis of gut rotation during
development?

ANS: SMA

Q34: A injury to orbit of eye due to direct hit by a squash ball. Now presents with
diplopia. Visual acuity Normal. X-ray orbit anatomy is normal. Best next
investigation?

ANS: MRI

Q35: Case of pneumonia in a 3 years post renal transplant patient not responding
to antibiotics. Which organism responsible?

ANS: CMV

Q36: Morton’s neuroma cause or pathophysiology?

ANS: nerve tissue with fibrosis

Q37: Patient removed from burnt house. In what scenario patient needs
intubation?

ANS: A respiratory rate of 35

Q38: Clinical Vignette describing tibia fracture who underwent internal fixation
developing respiratory distress

ANS: a. Fat embolism


Q39: What investigation is best for a patient thought to have labral tear of
acetabulum.

ANS: MRI

Q40: True about FFP

ANS: can't use them after 24 hours out of fridge

Q41: Pt with serum potassium 6. What should be imidiate step?

ANS: iv fluid was the only logical answer

Q42: Which of the following requires immediate renal replacement?

ANS: K+ 6.1 mmol/L

Q43: A clinical vignette that describes a woman with hypothyroidism and


microcytic anemia (MCV was low).Which of the following is the patient at highest
risk of during perioperative period?

ANS: Decreased cardiac contractility

Q44: Clinical vignette describing gross picture on UGIE pertinent to atrophic


gastritis. Which of the following antibodies wil be found?

ANS: Antibody to parietal cells

Q45: Clinical scenario describing a DRE findings of BPH. What pathology is


underlying?

ANS: Hyperplasia

Q46: Pregnant lady with anemia what you should give her?

ANS: Vitamin B12 (methylcobalamin IM)

Q47: Where exactly lymph drainage of scrotum?

ANS: Medial horizontal superficial inguinal LN

Q48: Renal transplant patient described and a table given showing high calcium/
low phosphate/ high alkaline phosphate. What is the diagnosis?

ANS: Tertiary hyperparathyroidism


Q49: Post-operative patient who is recently getting restless. Urine output chat is
tabulated hourly as 80 ml / 80ml/ 80 ml /0ml/ 0ml/ 0ml in last 5 hours. What is
the cause?

ANS: Blocked Catheter

Q50: Over infusion 3.5 L of NS to shock patient ? Which of the following is the
potential result?

ANS: Hyperchloremic acidosis

Q51: Fracture Shaft of Femur was reduced and implant placed. At the end of
operation you note that the lower leg seems pale with reduced pulse. What is the
best next step in management?

ANS: Compartment release

Q52: Can't pass colonoscope beyond hepatic flexure. What investigation shall
follow?

ANS: Ct colonography

Q53: During splenectomy, short gastric arteries ligated. Which structure at risk
of direct damage?

ANS: Tail of pancreas

Q54: Clinical Vignette describing locked knee joint in a young male with trauma

ANS: medial meniscus tear

Q55: Clinical Vignette describing locked knee joint in old age non-trauma patient

ANS: Loose body

Q56: Patient can't lift hand from table patient can't extend distal interphalangeal
of thumb

ANS: Radial post interosseous

Q57: Which of the following muscles is responsible for quiet respiration

ANS: Diaphragm

Q58: EMQ on Breast lesions investigation


Breast abscess

ANS: USG

Q59: B/L implants with suspected mass and positive family history

ANS: MRI

Q60: Retroareolar lobulated mass

ANS: Core Biopsy

Q61: RUOQ 3 cm mass

ANS: FNAC

Q62: Bleeding per rectum in an Old Gentleman. Colonoscopy revealed it’s around
Sigmoid colon. Planned for angio-embolization. Which vessel should the
interventionist enter to stop bleed?
ANS: L colic

Q63: Loss sensation medial leg. Which nerve?

ANS: Long saphenous nerve

Q64: Loss sensation 1st webfoot. Which nerve?

ANS: Deep peroneal

Q65: Action on which of the following receptor is responsible for the depressant
action in an old man who underwent laparotomy and is on Morphine, RR is
6/min?

ANS: Mu receptor

Q66: A young guy with testicular pain describing features of Epididymo-orchitis.


Which organism?

ANS: Chlamydia

Q67: After excision of submandibular deviation of tongue is noted. Nerve injury?

ANS: Hypoglossal

Q68: General sensation lateral tongue by which nerve?


ANS: Lingual

Q69: Complicated Flail segment scenario but no hemo/pneumothorax – cyanotic,


old man

ANS: Intubate and PPV

Q70: Young boy with stable hemodynamics but had few ribs fractured

ANS: Lignocaine block

Q71: Young girl with decreased breath sounds

ANS: Chest tube insertion

Q72: Esophagus hiatus how many cm from incisior to T 10

ANS: 40

Q73: Some post esophagectomy patient having a stricture on anastomotic site

ANS: Stenting

Q74: A women with jaundice, imaging described what they mean to be primary
biliary cirrhosis. Which antibody you test positive?

ANS: Antimitochondrial AB

Q75: Peritonitis and air under diaphragm described. Which Investigaion is best
in detection?

ANS: X-ray erect

Q76: The typical question describing pain in legs on walking and gets better by
walking uphill and pushing trolley.

ANS: Spinal stenosis

Q77: That typical question of patient in intermittent positive pressure ventilator


setting. What occurs?

ANS: Decreased venous return


Q78: A man has continuous ooze from wound for 2 days after a femoropopliteal
bypass. He was on aspirin and clopidogrel till 48 hours before operation. Why do
you think is the bleed?

ANS: Side effects of Aspirin and clopidogrel

Q79: Some fev1 1.8 L fvc 2.1 L what's the pathology?

ANS: Restrictive disease

Q80: A young girl with dyspnea. Fev 1 / fvc 36 then some treatment give and
raises to 78. DX?

ANS: Asthma

Q81: Pelvic trauma. Unable to pass urine. Where is the site of injury?

ANS: Membranous urethra

Q82: That typical question on bladder mucosal change with Schistosoma?

ANS: Squamous metaplasia of transitional cells

Q83: 56 years old male have cough and we are did X-Ray we find it have a a mass
in middle lobe of the lung bronchoscopy was done and send to histopathology,
*keratin was found (note keratin mean skin) most likely to have?

ANS: Squamous cell carcinoma of the lung

Q84: Lung tumor in a woman, nonsmoker and the lesion is situated peripherally
Ali Mansour :) (You might find *glandular in scenario of lung cancer).

ANS: Adenocarcinoma*

Q85: Those multiple times repeat of scaphoid blood supply

ANS: From the distal end

Q86: Neonate abnormal skin crease in groin

ANS: development hip dysplasia

Q87: 65/ F with Breast Ca which is ER –ve . PR –ve but Her2 neu + ve . what postop
regime you give as adjuvant ?

ANS: Herceptin and ECF regime


Q88: DVT diagnosis best initial

ANS: compression ultrasound

Q89: Fracture patella management

ANS: Wire binding (figure of 8 fixation)

Q90: The mentally retarded patient but her parents refuse surgery which seemed
not much urgent

ANS: Consent parents

Q91: Patient with appendicitis and also has Down’s syndrome but intellect good
that she knows consequences and complications of the procedure. Comatose pt
coming to ER

ANS: consent the relatives

Q92: Which of the following is not in immediate relation of 3rd part of duodenum

ANS: Right kidney

Q93: Which of the following makes the posterior wall of inguinal canal?

ANS: Conjoint muscles /tendons

Q94: Which of the structers listed is the reason that testis descends to scrotum?

ANS: Gubernaculum

Q95: Scrotal ulcer

ANS: Superficial inguinal

Q96: Cervical lesion

ANS: Deep Cervical LN

Q97: Anal canal lesion

ANS: Deep inguinal

Q98: From which structure listed does the External spermatic facia derive from ?
ANS: External oblique aponeurosis

Q99: Lateral side of ankle injured in an inversion trauma. Which ligaments?

ANS: calcaneocuboid and Talo-calcaneal*

Q100: Cuboid bone has following joint relations in its distal part

ANS: Fourth and fifth metatarsal

Q101: Prior to Episiotomy, which nerve is blocked?

ANS: internal pudendal nerve block

Q102: Relation of extensor indices to extensor digitorum in index finger?

ANS: Medial and lateral

Q103: Old man with prostatic cancer, has bony mets to shaft of femur and
asymptomatic.

ANS: Radiotherapy

Q104: L chylothorax after thoracotomy and esophagectomy. Where in the course


is the Thoracic duct injured?

ANS: During its course from diaphragm before it reached superior mediastinum

Q105: One Question on SIADH. Which ca lungs

ANS: Adenocarcinoma

Q106: Coin ingested. And seen on xray somewhere

ANS: Lodged at level of cricopharyngeal level

Q107: Mechanism behind pale offensive stools in Obstructive jaundice

ANS: Decreased enterohepatic circulation

Q108: Which of the following intervention will decrease raised ICPP with
immediate afftect but short duration?

ANS: Hyperventilation
Q109: Long scenario on Pancreatitis. Which of the following is the mediator of
injury?

ANS: Lipase

Q110: A clinical vignette, which you will understand easily that it’s a subclavian
steal. What investigation will you do?

ANS: Selective arteriography of aortic branches

Q111: A clinical vignette of A-fib, widened mediastinum and leg going pale

ANS: Aortic dissection

Q112: Four hours of testicular pain in a young boy after bicycle riding.
Examination shows a tender scrotum. No further description. What would you
do?

ANS: Explore immediately

Q113: Tender testis with black dot in upper pole

ANS: Hydatid testicle Morgagni answer

Q114: That typical question of Anaphylaxis but patient was not that sick

ANS: IM adrenaline

Q115: Which of the following changes occur after cholecystectomy?

ANS: Decreased rate of bile flow after meals

Q116: Slowly growing painful lateral swelling in knee

ANS: Meniscal cyst

Q117: A man-developed cellulitis in hand after bitten by some insect. At


operation table it was now found that he has a goiter & heart rate is high. What
drug wil you give?

ANS: Beta adrenoceptor antagonist

Q118: Ova cyst bloody diarrhea

ANS: Metronidazole
Q119: Trauma scenario with tender pain in thigh with external rotation with no
distal pulses

ANS: complex # neck of femur

Q120: Traumatize pt with short and adducted hip joint most likely to have?

ANS: Posterior hip dislocation

Q121: A vignette describing Broca’s Aphasia

ANS: Middle cerebral

Q122: Similar scenario with nystagmus

ANS: PICA Syndrome

Q123: A young guy with Midgut volvulus underwent resection and has 30cm of
gut left

ANS: TPN

Q124: Redo operation for esophageal cancer

ANS: Feeding Jejunostomy placed at the time of operation

Q125: Right hemicolectomy

ANS: Can be fed orally

Q126: A confusing scenario of patient developing sudden urinary retention and


lower limb paresis. Which artery?

ANS: Anterior spinal

Q127: While doing a LP, whats the last structure to pierce ?

ANS: Arachnoid mater

Q128: A case describing a horner syndrome(ptosis,myosis,anhydrosis). Other


features are already given and you have to choose one more.

ANS: Ptosis

Q129: Another similar scenario, that tells you that its horner. Which site may be
affected?
ANS: Lateral horn of spinal cord

Q130: One young guy underwent Panproctocolectomy for UC

ANS: Subtotal coloctomy+end ilistomy( ORDiverting ileostomy with descending –rectal


anastomosis )

Q131: One old man had sigmoid perforation and has fecal peritonitis and is in
shock

ANS: Hartmans procedure

Q132: Pt undergoe sigmoidectomy have copd and past surgical history of Rt hip
replacement 2 year ago what ar you going to do to avoid thrombo-ebmolic
phenomena in this pt?

ANS: LMWH+Thrombo-Embolic Disease Stocking

Q133: Pt undergoes inguinal hernia repair?

ANS: Not require any thing (just early mobilization)

Q134: Pt undergoes right hemicoloctomy for cecal cancer and he take warfarin
for his mechanical valve what ar you going to do to avoid thrombo-ebmolic
phenomena in this pt?

ANS: Systemic hebrinization therapy

Q135: Pt undergo A-p resection for rectal cancer, have a past history of DVT
following fracture femoral shaft 5 years ago what ar you going to do to avoid
thrombo-ebmolic phenomena in this pt?

ANS: High Dose of LMWH+Intra-Operative Pneumatic Calve Compression

Q136: Obese (BMI=30K/m2) lady undergoes cholecystectomy?

ANS: LMWH + Thrombo-Embolic Disease Stocking

Q137: Choose a good maintenance fluid regime for a fit young guy

ANS: 2L 5%dex with 1L Na

Q138: Which is true regarding left coronary?

ANS: It runs in anterior interventicular groove


Q139: A 25 year old male presents with epistaxis, the ENT SpR plans to cauterise
the bleeding point with silver nitrate.

ANS: 1% xylocaine with 1 in 200,000 adrenaline

Q140: An 18 year old boy requires a Zadeks procedure.

ANS: 0.5% Bupivicaine

Q141: A 72 year old woman fractured her distal radius.Biers Block is planned to
facilitate reduction of the fracture.

ANS: Prilocaine 1%

Q142: Pt undergoes inguinal hernia surgery best ansthesia is?

ANS: 0.5 % Bupivacine

Q143: Pt undergoes surgery in his scalp ansthesia of coice is?

ANS: 1% ligocaine +200 000 adrenaline

Q144: Newborn with dyspnea and the NG not passing into stomach

ANS: TOF (trach-esophagial fistula)

Q145: Need to do a tracheostomy or something like that. Which of the listed


structures have the least chance to get injured in this procedure?

ANS: Ascending pharyngeal artery

Q146: Mechanism of action of heparin

ANS: X inhibition

Q147: After vigorous exercise pt have Red Urine What is inx of choice for this pt?

ANS: This pt have Rabdomylysis(you can find CK(creatinine kinase) five time increase)

Q148: A child with neck swellinh at level of sternocledomastoid and diagnose as


branchial cyst what is emberyological level of this swelling?

ANS: 2nd-3rd branchial cleft ( you might find just 2nd OR 3rd sepratly)

Q149: Palatoglussus muscle supply by?


ANS: Vagus nerve(10)

Q150: All muscle of the toung supply by?

ANS: Hypoglassus neve( CN 12)

Q151: Urachus, the remnant will be seen as

ANS: median umbilical ligament

Q152: Spinal cord contents outside without any abnormalities?

ANS: Meningocele

Q153: Abdominal aneurysm 4.7 cm mnx - went with serial ultrasound. Other
option explore

ANS: serial ct

Q154: Bluish lesion of toe and inguinal lymphadenopathy?

ANS: melanoma

Q155: Ranula described. Which gland does it originate from?

ANS: sublingual gland

Q156: Why is it important to excise thyroglossal cyst?

ANS: infection

Q157: trauma to head and verterx is depressed. Which structure may be


damaged?

ANS: superior sagittal sinus

Q158: A female with 2 weeks h/o stridor, dysphagia & hoarseness. O/E thyroid
swelling & lymph node +ve. FNA shows spindle cells etc.

ANS: Anaplastic ca

Q159: Brain Tumor with necrosis and invasion push to opposite side through
corpus callosum.

ANS: GBM
Q160: Young girl cervical lymph node. Diagnosed on Histopath to be MCT . what is
appropriate management ?

ANS: total thyroidectomy with nodal clearance

Q161: lady with anterior neck swelling and it histopathology show psamoma
body and pale empty nuclei what is most likely diagnosis?

ANS: Papillary carcinoma of thyroid gland

Q162: Lady with anterior neck swelling and histopathology show oxiphl cell with
bone mets most likely to have?

ANS: Hurthle cell tumor(follicular carcinoma of thyroid gland)

Q163: Male Pt fallen on outstretched hand, and diagnosis have fracture distal
radius and have pain out of proportion, swelling and stiffness and colour change
in his forearm what is cause of his problem?

ANS: Complex regional pain syndrome (might appear to you in 17 of april)

Q164: Individual has right to act freely from his or her own independent thought?

ANS: autonomy

Q165: A process that provide feedback on doctors performance, monitor in order


to insure focused improvement?

ANS: Appraisal

Q166: provide data for wide group, allowing comparison of individual result with
national level or average?

ANS: Comparative audit

Q167: Refer to the way that the patient is received and managed by services from
time to time?

ANS: process

Q168: Patient chronic smoker with nausea and vomiting and have nicotine stain
in his finger diagnosis as?

ANS: Duodenal ulcer


Q169: Risk factor of hepatocellular carcinoma is?

ANS: Alcohol

Q170: Japanese man of 70 years old have gastric mass suspected to have ca of the
stomach, risk factors is?

ANS: Ménétrier’s disease (Hypertrophic gastropathy)

Q171: use to forward understand of disease, improve surgical outcome or


critically appraise the literature?

ANS: Research

Q172: The collective review evaluation and improvement of practice with the
common aim of improving patient care and outcomes. It performed
retrospectively?

Ans: Audit

Q173: Lymphatic drainage of thyroid gland upper pole?

ANS: Anterior-Superior group of deep cervical LN.

Q174: Lymphatic drainage of lower pole of thyroid gland?

ANS: posterior-inferior deep cervical LN

Q175: At which triangle you can find recurrent laryngeal nerve?

ANS: Beaher triangle

Q176: Left recurrent laryngeal nerve hock around?

ANS: Ligamentum arteriosum

Q177: Right recurrent laryngeal nerve hock around?

ANS: Subclavian artery

Q178: Thyroid cancer follow up?

ANS: Thyroglobulin

Q179:
PART-A APRIL 2008

1. Surface marking: Femoral Artery, Internal Carotid Artery

2. Superficial structure on exposure of the Popliteal Fossa

3. Structures passing underneath the piriformis in the greater sciatic notch

4. Position of brachial artery in the elbow relative to the median nerve

5. 3 questions on three branches of Internal Carotid Artery in association with


various landmarks

6. Inheritance mode of Achondroplasia

7. One structure not part of posterior mediastinum from a choice of five

8. Supply of lacrimal duct from which ganglion

9. At least three scenarios on acid base disturbances

10. At least four scenarios on nerve root values

11. Mechanism of action of Warfarin

12. Route of flow of blood from RA in the foetus

13. Post-op pain (opiate over-dosage and inadequate pain relief), scenarios with
miosis and acid base disturbances respectively

14. Gas exchange in the lungs at which level

15. One direction the lungs do not expand on inspiration

16. Anatomical level of Pancreatic body

17. Mechanism by which smoke causes carcinoma

18. Common cause of oral carcinoma, carcinoma of bladder and pancreatic

19. Granulomatous lesion in the intestine

20. Typical description of tuberculosis: weight loss, lung lesion

21. Typical description of SCC: weight loss, cytology findings and apical mass, hilar
lymphadenopathy

22. One feature present in Tension Pneumothorax


23. One feature present in PE

24. Laceration on the wrist with inability to do thumb adduction, which nerve is
involved?

25. Post-operatively, pt. gets abdominal pain and nausea one hour after eating,
doesn’t relieve until he lies down. Cause?

26. A woman in 30’s presenting with an enlarged lymph node. FNA confirms
malignancy, Type?

27. A mother brings in a child irritable and complaining of pain with diagnosed
hydrocephalus. One feature present in hydrocephalus?

28. An elderly lady on wheel chair has loss of muscles in her legs. Process
involved?

29. A woman cuts her finger. Next morning notices red streaks going up on her
forearm

30. A man brought in after a RTA complaining to pain abdomen radiating to the
back. Sent home with some analgesia. 48 hours later brought back unconscious and
couldn’t be saved. Cause?

PART-1 SEPT-2008

1:Lumber puncture in space??

subarachnoid

subdural

epidural

2:Lavel of termination of conis medularis in neonates???

L1; l2; l3 l5

3:Right leg weakness in a man occlusion of the artery??left antcerebral left middle
cerebral; rt ant cerebral; rt middle cerebral

4:Structure damaged just infront ofthe lateral melleolus muscl?? proneus???

5:a child fell and injured left side of the nec, some neuralgia the recoverd fully what
caused?? neuropexia?? axonalpexia??
6:a man brought to emergency deptt with head injury initially gcs 15, after some time
10 cause subdural; extradural; intraventricular bleeding

7:ASA I,2,3,4,5 RElation to the deasesbp 175/90; copd; diabetic ketoacidosis;


orthopnea at rest

8:lavel of inferior vanacava opening at diaphragm??

9:breast uoq swelling with some sclerotic changes on mamography?? cancer;


fibroadenoma; duct ectesia; fibroadenosis; Pagets disease

9:tibia/fibulla fracture operatedwith intramedullary nailing?? afterwards compartment


syndrome ant compartment; lateral dompartment

10Pupil dialated rt side cause??? 3rd nerveparasympthatic damage; 3rd nerve


sympathatic damage; 2nd nerve 4th nerve...............

*what vitamin supports collagen?vit A ; vit E ; vit B6

*Face lesion with central keratosis?! Bsal cell carcinoma; Malignant melanoma

*Sneezing with eosinophil? Ig A; Ig E; Ig M; Ig g

*Colon Duks classification?*Erection dysfunction--- Splanchic nerve, pudendal


nerve, ilioiguinal nerve

*Nerve supply skin under left eye? Infraorbital; facial; trigeminal

APRIL-2009 P-2

ANAESTHESIA DRUG

1. SCCOLINE2. VECURRONIUM 3. NEOSTIGMINE 4.HALOTHANE


5.PENTOTHAL6. KETAMINE7. NONE OF ABOVE

Q1. ANAESHESIA WHICH CAUSES LIVER DAMAGE/ HEPATITIS IN POST OP.

Q2. NON DEPOLRISUNG MUSCULE BLOCK

Q3. DRUG USED FOR REVERSAL

Q4. DRUG METABOLISED BY PSEUDOCHOLINESTERASES

2 Tumour marker

A. CA125B. CA19-9C. ALPHA FPD. BETA HCGE. NONE OF ABOVE

Q1 TERATOMA
Q2 CA BREAST

Q3 MALIGNANT MELANOMA

Q4 HEPATOMA

3.Pain Releive

MECHANISM OF ACTION

A. RETICULAR ACTIVATING SYSTEM

B. MU AND KAPPA RECEPTOR INHIBITION

C. SPINAL GATE THEORY

D CENTRAL GREY MATTER INHIBITION

Q1 MORPHINE

Q2 TENS

Q3 LIDOCAINE 2%

4.Breast lump

5. Nipple Discharge

duct ectasia

duct papilloma

mastitis

cystadenoma

fibroadenoma

q1 26 year female present with bloody discharge frm one duct

q2 36 year female present with creamy discharge from nipple

6. Wound closure

skin grafting

split skin graft

full thickness graft

local flap

leave at is dressing
q1 1 c.m. loss in cheeck area excision of superficial ulcer

q2. 1 c. m loss of alar cartilage nose

7. Biochemistry renal stone

cysteine

urates

calcium oxalate

tripple phosphate

cholesterol

q1 24 yr male with mid uretric stone with h/o previos stone passage

q2 16 yr male with h/o mid uretric obstruction with family h/o of stone in uncle

8. knee injury

patellar tendon rupture

qudriceps muscle tear

patellar fracture

capsular injury

q1 24 year player had injury around knee with h/o swelling above patella no active
treatment given healed after few monthq2

9.ankle injury

10.elbow injury

11. Hip joint Pain

12. Digital ischaemia

13. Diarrhoea

14. Peri anal conditions

haemorrhoids

anal fistula

anal fissure

pilonidal sinus
q1 h/o painful defecation with little blood on toilet paper

q2 tender large swelleing in between natal cleft

15. Neurosciences

clinical presentation in:

corticospinal tract

basal ganglion

cerbellar lesion

leminiscus lesion

q1 cogwheel rigidity

q2 resting tremors

16. Thyroid profile

17.acid base balance

18. Biochemistry na/k/ca

hypocalcemia

hypokalemia

hyper calcemia

hyper kalemia

normo calcaemia

q1 a 48 yr female gives h/o fall and fracture of vertebrae

q2 a pt with lethargy and weakness with bone pain and renal stone

q3 a 24 yr male with h/o abdominal pain and profuse diarrhoea

19: chest pain

20 chest pain treatment

21 Chest injury treatments

22. Urinary bladder condition

23. Inguinal hernias

Direct inguinal hernia


indirect inguinal hernia

femoral hernia

q1 female with inguinal swelling below and lateral to pubic tubercle

q2 hernia medial to inf epigastric artery

24. Nerves inguinal region

Genital br. of gf nerve

femoral br of gf nerve

Ilioinguinal nerve

Ilio hypogastric nerve

q1 loss of sensation after hernia repair root of penis and adj scrotum

q2 paresthesia over the lateral side of thigh

25. Burn injury

simple occulusive dressing

leave as such

split skin grafting

full thickness graft

q1 abdominal burn 5 % dried wound painless with 24 hr h/o

q2 burn and blister in forearm of young child by boiling water

26. Scrotal swelling

ingunal hernia

hydrocele

torsion cyst of morgagni

epydidmitis

varicocele

sperm granuloma

q1 h/o vasectomy with swelling above testis

q2 h/o swelling in 50 yrs male for 2 year inguklnal and scrotal region both
q3 6 7yr child give h/o painful swelling above testis

27. Jaundice

28: Pancreas

29: Parotid swelling

30. Dysphagia

31. infection soft tissue

32. Infection soar throat

33. Neck swelling

34. Back ache

35. Sensation loss foot/leg

34. Consent

A Consent the patient

B Consent the parents

C Consent by proxy

D Cannot proceed

E Proceed without consent

F Apply for a judge to a court

Match the most appropriate option from the list above to each clinical situation
described below. Each option may be used once only, more than once or not at all.

A 22-year-old male with motorcycle injury – unstable and unconscious, and needs
operation.

A 6-year-old-boy with deteriorating health needs a blood transfusion, the parents


refused for religious reasons.

A 14-year-old girl with iliac fossa pain and vaginal bleeding needs diagnostic
laparoscopy.

35. Report

36. Facial swelling

37. Transplant terminology


38. Bleeding PR

39. A Any medical practitioner

B Coroner

C Coroner’s officer

D Medical practitioner who attended during previous 14 days E Registrar of births


and deaths

For each of the statements below, select the most likely answer from the above list

Certify deathIssue immediate death certificate Call an inquestRegistrar of births and


deaths

1. Organism causing infection after thorn prick.

2. Swelling beside perianal region.

3. swelling in the rectum with discharge.

4. Testicular appendix torsion.

5.ankle brachial INDEX is more then `1.

6.wrist drop nerve injury.

7.ulner nerve palsy.

8.wrist drop with the no loss at dorsum of hand? Level of injury .

9.hepatotoxis GA.

10. depolarizing anaestetic.

11.anaesthesia at the lateral scrotum site. nerve involved .

12. nerve involved in the lateral thifh cutaneous anaesthesia /

13.medial meniscus tear with lock knee.

14.quadriceps tear with selling above patella.

15.lateral ligament injury .of knee.

16.ant cruciate tear . with knee foreward movement .

17.ulcer at the face above line btwn lip and ear .

18. basel cel ca, with 2 cm marginal debridement .


19.branchial cysy,

20.thyroglossal cyst.

21.boerhave syndrome. bleeding after food .

22. oral bleeding in unconcious,have bleeding disorder ? ....ans.oesophagealvarices


.

23.bleeding perrectum[/b][/b]

1.pleomorphic adenoma of the parotid gland .

2.acidosis and alkalosis .

3.leirch syndrome.

4.melroy weiss syndrome.

5.gastric lymphoma.

6.achilles tendon tear.

7.loss of tricep reflex.

8.carple tunnel synd.

9.wrist drop. radial nerve injuury at different levels.

10.posterior interrosius nerve injury.

© Notes by Dr. Sakib (MRCS Online Preparatory Course By Nasa Khan)

MRCS PART-1 09 JAN 13TH QUESTION/THEMES

1. WHAT IS THE ANTERIOR BOUNDARY OF FEMORAL CANAL

2. WHAT IS THE POSTERIOR WALL OF FEMORAL CANAL

3. DIRECT INGUINAL HERNIA IS THE WEAKNESS OF WHICH WALL NAME

4. anterior surface of heart is formed by

5. heart valve is made of

6. MEN II b comprises of
7. medullary carcinoma presnts as

8. 1st web is supplied by which nerve

9. distribution of supf peroneal nerve

10. level of bifurcation of aorta

11. level of hilum of kidney

12. level of inf mesenteric artery

13. level of ext iliac artery

14. distribution of genitor femoral nerve

15. definition of hypoxia

16. tissue development in # atrophy

17. tissue development in spina bifida hypoplasia

18. post gastrectomy deficiency of

19. pt with persistent vomiting biochem abnormality

20. passive rectal incontinence sphinter envolved

21. presentation of long standing catheter

22. tubercular cystitis

23. nerve supply of ant aspect of knee jt

24. relation of str at popliteal fossa

25. muscle attach to lat side of popliteal

26. nes of adductor of thigh a urologist is doing some intravesical procedure alll of
sudden patient kicks which nerve may have involved ?

27. flexion at distal ip jt at ring finger

28. flexion at distal ip jt at thumb

29. relation of ulnar nerve with ulnar artery

30. ulnar injury at elbow

31. foment sign test for

32. nerve for adduction of thumb


33. deep br of ulnar nerve supply

34. function of cortisol in stress

35. ecg changes in pulm embolism

36. nerve supply at angle of mouth

37. lateral border of tongue is supplied by

38. gastric lymphoma are mc of which type

39. cut injury at side of face will cut

40. epydymorchitis in sexually active male without uti

41. cause of swelling scrotum with sec neck

42. multiple swelling all over neck/axill /inguinal malignant lymphoma

43. follicular tumour thyroid IOC

44. sesation over deltoid muscle

45. injury medial to deltopectoral groove

46. head of radius articulate with

47. Head of radius is kept in place by which ligament

48. cereberal perfusion pressure

49. central chemo receptors

50. Baroreceptors

51. foramina for transmission of mandibular nerve

52. foramina for transmission of vagus/ hypoglossal

53. foramina for transmission of middle meningeal artery

54. nucleus of 6th and 7th cr. Nerve is at which part of brain

55. surface marking of heart valve

56. cardiac tamponade can cuase sudden death

57. peptic acid secretion is stimulated by

58. bee sting with bp 80/60 and p.rate 122/mt TtOC

59. dorsum of foot supplied by which nerve


60. ligation of varicocele ultimately vein leadin to gonadal vn

61. relation of vein at renal hilum

62. COPD

63. blood gas analysis

64. blood gas analysis65. blood gas analysis

66. difficulty in smiling nerve for it

67. pain in inf molar nerve responsible

68. ganglion for lacrimal dut

69. nerve involved in submandibular gland excision

70. type of reaction rhinorrhea with rashes after two hrs

71. intrinsic factor absorption

72. anemia in gastrctomy

73. iron deficiency anaemia

74. granulomatous intestinal dis crohn.

75. biopsy of gastric antrum

76. causative agent for gastric ulcer and bladder carcinoma

77. virus implicated for cervical cancer

78. virus implicated for anal cancer

79. virus implicated for Kaposi sarcoma

80. pus in diabetic cholycystitis e. g. of

81. injury at third intercostals space at sternum

82. third ventricle to fourth cereberal duct of aqueduct

83. SOL lt in parital causes herniation of

84. vertebral artey supplies which cortex

85. pituitary tumour causing pressure at optic chiasma

86. erosion at lateral angle of eye by swell in three years

87. position of ampulla of vater opening


88. SVR/CO/HR in crdiogenic shock

89. SVR/CO/HR in hemmorrafgicc

90. SVR/CO/HR in septic

91. child with ear discharge fever convulsion high grade fever

92. best criteria fo acute pancreatitis

93. diagnostic marker of carcinoid

94. in aneurysm defect in arterial wall is

95. popliteal aneurysm size 2.5 cnm

96. nodule in rheumatoid arthritis

97. wrist drop wekness at elbow

98. sensation of middle finger root value

99. sensation at lat calf disc protrusion

98 sensation loss at great toe spinal levet

100. lspinal level in uretric pain

101. passage of small stone cause

102. ns of lateral part of arm

103 parasympathetic causes arterial vaso constriction /dilatation

104 sa node is inervated by

105 adrenaline causes st of which receptors

106 role of adrenaline

107 role of dobutamine

108 role of ephedrtine

109 changes in lung capacity in emphysema

110 FEV1/FCV is reduced in

111 Ist to reponse to haemmorrage

112 Patent ductus arteriosus develop fromn which arch

113 Trauma to medial third of clavicle will injure


114 Bendrofluadizide acts at115 Inrease urine osmolalty in

116 Stroidal pt on surgery post op adissonian crisis\

117 Left femoral hernia op for small bowel obstruction lead to chest pain?

118 Factor controlling bp from kidney

119 Post operative hypocalcemia in thyroid ectomy

120 Mode of action in pth in ca metabolism

121 Ca breast role of in osteoporotic Calcitonin

122 Cuases of hypercalcemia burn?

123 b/l swelling in 60% burn pt due to hypoproteinemia

124 insulin is increased with c peptide

125 partail gastrectomy result in dumping syndrome

126 multiple neurological envolvement in diabetes

127 insulin given in hyper glycemic will

128 effect of cortisol on various hormone a patient presented with h/o trauma
cortisol released will have primary effect on a) growth hormone b )thyroxine c)ACTH
d)insulin

129 barret’ esophagus > 5cm pathology envolved

130 para neoplastic syndrome see in small cell tumour

131 in fetal circulation blood passes from rt atrium to left atrium

132 intermittent positive pressure incr/decreases venous return

133 basic mechanism in pulm embolism

134 post splenectomy diffuse opacity lung caused by pneumoccocal pneumonia

135 ca brest operation nerve envolved in winging of scapula

136 upper arm injury with swelling and pain fasciotomy

April 2017 recalls


1. What was that thumb abduction with hand on table answer:I cant remember
maybe EPL
2. what about injury in axilla and numbness in lat forarm:C6
3. source of blood during lumbar puncture ?lumber venous plx
4. type of femur fracture in the
5. child who was hit from side ?tranvese/olique/greenstick/spiral
6. Womnan with mx # same options
7. Tibial twisted-spiral# same
8. Child radial #
9. Pt e prolonged catheter this frank hematuria and w loss :regional transional
ca,diffuse transitional ca/scc/intstisial cystitis
10. Cyst in breast aspirated cytology free what next? I wrote bx, but the asnwer
is dischage
11. There was melonoma in situ removed.? Discharge
12. One was audit of process
13. Second I wrote clinical audit but I cant remember the Q.
14. pt e breast cancer and metastatic disease come e hypotonia:hyper ca
15. After vericose vein surgery numbness dorsum of foot which nerve
damaged: saphenos
16. Ttt of post op phelbitis: bandiage
17. Pt e antithrombin 3 Def and dvt : I choosed short duration heparin but the
correct answer was Warfarin life long
18. Rx for recurrent pulm edema :V.C filter
19. Surface anatomy of IJV ??? From lobe if the ear to sternal edge of clavicle
20. Ulnar nerve , artery relation upper or lower forarm: ulnar to ulnar art.
21. There was this question about GCS. Patient localizes pain , open eye to pain
and sounds whats the GCS ? I chose 9 as no option of 8 in answer choices(I
awkwardly calcute it wrong !)
22. Peronial hematoma and lower abd hematoma, rupture
bladder,bulbar,membranous.prostatic urethra ing.
23. with leg flexion at 30degree and moves lateral after trauma I chose.. medial
colateral injury
24. Then a man hit by car when viewed from above left tibia lie posterior i
chose pcl
25. Sciatic nerve injury what other movement not possible: extention of hip
26. 6 yrs Child e sudden hip pain: perthis
27. Oh there was a question u operating parathyroid and find nodule in thyroid
what to do i chose thyroidectomy( icant remmber the Qs and answer!)
28. Hemithyroidectomy for papillary 0.5cm nodule
29. Total thyroidectomy or medullary CA
30. Anexity adrenaline released from endings or gland ??gland
31. Inj to the dorsal root ganglia???sensoey,motor,sympathetic
,parasympathetic,all of above
32. The one with cervical subcutaneous emphysema. Esophageal rupture :Ix of
choice:contrast study,CXR
33. Ejection systolic murmur with pulmonary edema ; aortic calcification
34. Ascending or descending limb Ascendig loop for furacmide
35. Then there was this methylsomething thiazide where did that act : dct
36. pt had pancreatitis thin hyperpara .. inv what next ?? Thyroid ?Petutary?
Adrenals
37. Factor which doesnt cause colon ca. Chrons, alcohol,smoking,diverticulitis
38. Was it gout or oa ? Golfer with first metatarsophalangeal joint xray showed
decreased space and alsub chondral cyst: Rh arth.OA,Gout
39. Man used opoid then constipation; painful fresh bleeding
40. Then was about prostate lymphatics internal iliac
41. Testis lymphatics: para aortic
42. Scrotum: transverse inguinal
43. Pt e vien graft 16 month ago then stenosis pathophysiology: Fibrosis,
thrombosis or intimal hyperplasia
44. Nerve to upper sctotum and penis ?? ileoinginal
45. Which artery anurysn compressed renal vein:SMA
46. ant relation to rt adrenal gland:IVC
47. Portal vein drainsTo splenic Then splenic and SMV unite... IVC
48. Q about pt e head injury on IPPV ,at the 5 day techypnea and hypoxia ????
What to do CPAp CXR?/ tracheotomy /swangang cathI / Cxr
49. Other q pt start weaning in icu??? Same options above,, CPAP
/tracheotomy
50. Patient underwent operation for IBD complicated by infection...now
developed diarrhea...i pseudomembranous colitis
51. Esophageal perfiration type of feeding :TPN
52. Staged ediphagectomy:PEG
53. Perforated appendix organism? e.coli?
54. Bilious vomting ,scaphoid abdomen And dudenal atresia
55. 9 month What was in child with rt upper quadrant mass empty rectum and
vomiting ? Valvolis?
56. There was a child with family history of cystic fibrosis, meconium ileus?
57. And then there was child vomiting with Distended abdomen And
meconium plug was renoved after 3 days by suppository- hirschprung
58. Test for Addison des? ? short synacthen
59. Spleen rupture without trauma, Ebv
60. study comparing fluid requirements? after burn in male n females...most
patient had small values and some larger values...what test to apply? Mann
whittney/paired /unpaired t test /chi sequare?
61. Case pt have 5 unite blood then bleeding profile: decr pltls ,normal ptt, incr
INR , inc fibroplastin norm fibrinogen products: I choose DIC bcoz I don’t know
the others 😞
62. intrinsic pathway?factor 9
63. (Ascending Lymphangitis) creeping redness around vessels caused after
thorne/gardening? Strep pyogenes or clostridium perfringenes,staph
saprophyticus
64. Root of transmesion of mrca in some surgical ward 5 pt infected?
Inhalation, contact(I choose contact?)
65. Renal mass on USS do CT scan,MRI,IVU
66. 15x9 cm lesion deep fascia,what to do: excision bx,inscion bx ,core bx
67. That liver accumulation prussian blue:iron
68. Pt e history of FAP and colectomy done 30 yr ago now he came e jundice
pale stool and vomiting: deud.Ca
69. Melanoma suspected 1cm; Excise with 2cm margin?
70. Patient 6 days post-op burn surgery with hiccups n abdominal
distension...soft abdomen.. Acute gastric dilatation
71. Forhead mass pt itch cause bleeding follwed by incr in size during 1 yr:
bcc,scc,fibroadenoma
72. Which is the first immunologic cell in rosethorn and infl: Neutophil
73. what does warming reduce levels of? TSH,ACTH,oxitocine,vasoprisin
74. Mediastinal mass with keratin and flat cells :seq cell ca
75. Hyponatraemia in man post op with glucose fluids? Was it stress response
or excess lack of sodium containing fluids,extra fluid admition
76. Heat loss intra op by convection
77. Lung ca brain mets given dexa,paracetol,morphine
78. Marathon runner sudden death; SAH
79. Noturia ,thirsty - FBG:5 glucose then post prandial 2 hr was 7 –
normoglycemia,impaied glucose tolerece test ,DM,D.insipidus
80. What the hell do we give to pregnant lady with megaloblastic macrocytic:
B12 first,folate
81. What about gastric acid secretion reduced by: Protaglandin E2,ADP,
82. What drug is a precursor to noradrenaline? dopamine
83. Duct papilloma proplimatuc : duct excion
84. Post mastoidectomy loss of tast:chora tympany
85. Follicular with bony mets
86. Papillary with cervical lymph nodes
87. Then facial nerve - second arch
88. Pleuroperitoneal memberane for diaphramatic hernia
89. Surgeon clamped the lesser onentum structures which is at direct risk of
injury:ivc, Common bile duct,IVC
90. J waves in hypothermia
91. Exostrophy bladder. Dorsal surface opening of meatus
92. Nerves at risk Of injury surgical approaches:
93. medial ankle ; Saphenous
94. distal femur:tibial
95. During hip arthroplasty surgeon note Artery run in superior border of
pectenius ms ? Sup gluteal artery
96. clamped unbiblical cord ,which will not carry oxigintated blood to the hrt?
Ductus venosis,hepatic artery,umbilical artery
97. Pulm stenosis for tof
98. Burn question with oedema, hypoalbumin
99. For onion peel: Ewing sacoma
100. Aortic dissection.. ascending or descending ? Ascinding
101. thoracoacromial for deltopectoral groove
102. Common peroneal for dorsum of foot loss and dorsiflexion loss?
103. Smoking and microscopic was transitional cell ca
104. And egyptian farmer heamaturia was squamous cell ca
105. And the lft with raised amylase and ggt alcoholic panc
106. one was conus medullaris compression ...L1/L2
107. There was a uterus ligmsmet question/Transverse perineal muscles
108. Pfenen – arcuate,rectus ms rt,liba alba,suprf.exter.rectus appon.
109. Median inscion:Linea alba
110. Inguinal:Ext obliques
111. Dartos in subcutaneous
112. Cremastic muscle - int oblique
113. Osteoporosis question NormoCa
114. Psycholosis,increase ca,tsh low then stopmefication return to normal( milk
alkali synd,vit d toxicity,hyperthyroidism)
115. Keloid scar: ntralitional steroid
116. Median and brachial; Lat ant medial
117. X-ray failed to show scaphoid fracture...what next? Mri or ct scan? Or bone
scan?
118. Parathyroid adenoma - technetium sestimibi?
119. extraintenstinal features of crohns except: polyarteritis?
120. There was a question about low potassium, high creatinine, scarred small
kidneys? Atn, polycystic, chronic pyelonephritis,etc
121. There was rcc for hg 18.3
122. Cause of AAA ?? Htn ,artheroma , marfan,arthrosclerosis
123. Efferent arteiole vasoconsrricted by AT-2 i think
124. Pcwp 20= pul oedema
125. Pcwp 11= ARDS
126. High riding prostate.... Membranous
127. Pineal hematoma+ excessive collection....bladder
128. Cushing... Hypok
129. Metabolic acidosis... Hyperk
130. Wide qrs... Hyperk
131. The aortic valve replacement drop in hr and bp?its block, temponade
132. Gall bladder stone?bile pigment
133. Sickle cell disease +bone pain+pneumonia agent:I wrote stept pn
134. Pneumonia in ventilated... Psedomonas.sterp pn
135. Damage to 3.5 finger sensation :median
136. Guys there was a question about achllis tendon ruptute what was the test ?
I chose Simmons squeeze
137. What do they clean plastic syringes with? Radiation
138. What abt collection when patient lying supine: Hepatorenal pouch of
morisons,oaracolic gutter
139. Acute abd oain ,absolute constiopation,55yrs old abd x ray shows Small n
large bowel Air fluid level: colorectal Ca,adhisions,vololus
140. Tricuspid closes - isovolumetric contraction?
141. Pathophysiology ARDS: Reduced diffusio
142. Techy cardia chest pain: Decreased diastolic interval
143. Tricuspid closes - isovolumetric contraction?
144. Vital capacity
145. eggs at anal verge: Mebendazole or metronida?
146. Cleft palate posterior to incisive foramen due to? Malunion of maxillary n
nasal prominences?i choose palatine shelved
147. There was a q about multiple lytic lessions in pelvis? It was multiple
myeloma
148. *eosinophil q, type 1 hypersensitive?
149. *carcinoid,octreotide?
150. Hassal corpuscles: yes thymoma...
151. Gout, needle shaped negative under scope
152. Breslow depth less than 1mm good prognosis?
153. There melonoma in situ excised with 1 cm margin . Education of delf exam
154. chemo...only that sounded reasonable for metastatic breast Ca....
155. Vesicoureteric stone at?tanverse process L5( idid spinus process😫)
156. best prognosis in breast ca:Er+ve
157. Something to do with infection spreading to brain/inner ear was it sigmoid
sinus/Inner ear or what?sigmoid sinus
158. Brown sequard
159. little's area of nose artery? Sphenopalatine
160. anterior wall of the heart ? Mostly made of rv n ra
161. rt Coronary a arise .arise from post surface of aorta.arise from ant surface
.lei in post Interventricular groove .arise inf to aorta from sinus(Rt coronary ...
ant surface-Lt coronary ... post surface)
162. Depolarizing neuromuscular blocker... suxamethonium
163. Cervical sinus from what branchial cleft?2nd
164. Cleft palate post to inscive foramin what developmental anomaly?palatine
shelves
165. Insertion of ureter in bladder:post
166. Thoraco acromial a injured in deltopectoral groove
167. Second q ant to which part of vertebra ureter insertion
168. Q asking about poas major(T12-L5)
169. Realtion of ulnar artery ana ulnar nerve in wrist(ulnar to ulnar)
170. Ant surface of heart formed by what
171. Nerve lesion in hernia operation cause parathesia at top of scrotum and
base of penis
172. Ant relation to rt adrenal gland
173. Ewing sarcoma onion👌
174. The 4 cases about mechanism of injury
175. Green stick fracture
176. 2 cases wara b3d bilateral lung opacity in the 2 cases but pulmonary wedge
pr is high in one of them I choose for one pulmonary embolism ARDS
177. Artery in epistaxis
178. Q about osteomalacia
179. Q about multiple myeloma
180. After the nurse cut the umbilical cord which of the following will not
contain oxygenated blood any more Ductus arteriosus- ductus venosus-portal v -
umblical a-hepatic ar
181. Invetsigation for boerhaave syndrome
182. This first senario was AAA with low PAWP one post op
183. The second was following pancreatectomy a 3days later low PAWP
184. MEN1 pituitary
185. Medullary ca Tx
186. Follicular Ca Tx
187. Addison > hyperkalemia
188. TB medication adrenal crisis
189. Recurrent PE venacava filter
190. Thrombin 3 deficincy
191. Anasthesisa of pupic , scrotum following hernirophy
192. Impaired OGTT
193. cAMP decrease gastric acid production
194. Noradrenaline septic shock
195. Three audit q came in first paper
196. Then about 6 cases asking about electrolytes
197. 2 Q parkland formula
198. Preganglionic neurotransmitter - i marked ach
199. Resting potential of skilital ms maintained by(Na,K,Ca,Cl)
200. 60 yr old man e itchy and painless yellowish discularation
201. The question was asking about tuberclin test type of hypersensitivity as I
remember: Type 4,T cell mediated
202. Invest oesophageal rupture???baruim salaow
203. Deud ulcer:GD art.
204. Dumping
205. Hip extension due to semitendinosus
206. There was Q about ulcer deep to it swelling contain keratinized skin
calcium and bone What is the pathology:scc
207. Premature baby with inguinal tender swelling.. direct hernia, indirect,
femoral, testis torsion, inguinal lymphadenitis?
208. St segment depression in inferior leads:inf MI
209. A severe COPD patient receiving high flow oxygen became drowsy...?acute
resp failure
210. Predominant cells in granumola/tb...macrophages
211. Wide qrs ....hyperkalemia
212. High jejunal fistula? Electrolyte imbalance
213. A sever copd with peritonitis,He was metabolic acidosis due to peritonitis
214. Organism that causes diverticular abscess in adults..
215. patient presented with knee pain and operted in his left knee after 24 hour
he develop chest pain and hemoptysis shortness of breathing what underline
heamtalogical predispose to his s presentation???
Anemia,Thrombocytosis,Thrombocytopenia,NeutrophilIi
216. Extended match
question;Hyperkalemia,Hypokalemia,Hypercalcemia,Hypocalcemia:

217. 1_wide qrs on ECG


218. 2_absent p wave long QT interval
219. 3_high jujenial fistula
220. 4_cushing syndrome
221. 5_corrected metabolic acidosis
222. Tension pn.thorax
223. Sings of elevated ICP No history of any truma and morning headache nausia
moring bilateral papiloedema :frontal glioma ,chronic subdural
bleeding,subarachnoid hg
224. Most common cause of hydrocephalus:f.monro ,f.magende ,luska,aquidecut
225. which sensation lost on the thumb and lateral side of distal
forearm........answer was post interosseus or radial??
226. Penut inhalation :rt lowe pole
227. +ve pleural pr in :valsalva
228. Medial border to femoral canal:lacunar
229. Pt in sheet fawzia:metabolic acidosis due to pertonitis
230. Not happen in crhons :poly artritis
231. Swelling of whole arm after exercising how to investigate :selective axillar
ar angiogram,duplex,uls?
232. Atrial deporalzation: p wave
233. Spinal acessory post triangle neck
234. Pulm arts come from 6th arch
235. Doesiflexion L5
236. Qs about Pulm valve lt 2nd intercostal space?
237. Pips decrease preload
238. S3=perianal region
239. There is q about chrons disease then have watery diarrhea:hypokalemia

Another collection:
1. a patient with renal failure has Hyperparathyroidism - what kind of changes to
gland- hyperplasia, hypertrophy, metaplasia
2. Waddling gait: nerve— Sup Gluteal, Inf. Gluteal
3. Ulnar nerve relation in wrist.. Ulnar to Ulnar artery
4. Brachial artery to median nerve relation-LAM
5. Ureterovesicular junction: bony landmark on xray: Ischial spine
6. leg twitching during bladder surgery, nerve: Obturator
7. left renal vein thrombosis— anterior relation of vein—SMA,
8. right renal gland relations: IVC
9. T5 vertebra crush fracture; relations, azygos, arch of aorta,
10. st elevation lead II III aVF-Inf wall MI
11. origin of right Coronary
12. lateral border of clavicle, exposure of axially artery- which arterial branch is
anterior
13. 1st web space nerve supply: deep peroneal
14. test used for comparing results of treatment: paired t test, Unpaired t test
15. dumping syndrome path-physiology : hypoglycaemia,
16. dorsal foot surface numbness+ foot drop, which nerve damage: Common peroneal,
17. weakness in arm flexion and numbness in lateral forearm, nerve damage: MC
18. Pterion bones: Perietal,
19. Littles area: source of bleeding
20. ureter insertion in bladder, Lateral wall, posterior wall
21. damage to what structure during Pringle’s manouver : IVC, CBD
22. Direct hernia: medial to Inf epigastric vessels,
23. Femoral hernia medial border of canal Lacunar ligament
24. Dartos muscle scrotum: superficial to superfacial fascia
25. MEN 1A; Parathyroid, pancreas: what else: pituitary
26. fullness, SOB after meal in patient had trauma: Diaphragmatic rupture
27. 21 year old man collapses and dies on post mortem—subarachnoid hemorrhage
28. lacrimal gland ganglion: pterigopalatine
29. numbness on tongue after dental procedure: Lingual nerve
30. Post aaa repair patient breathless: CWP >18 LVF
31. Patient breathless CWP <18 ARDS
32. rib fracture, difficult to ventilate: Tension Pneumothorax
33. 6 year old male limping: Perthes, Slipped femoral head
34. rapidly growing tumour: keratoacanthoma
35. bloody nipple discharge 24 year old female: intraductal papilloma
36. metabolic alkalosis: NG aspiration
37. lymphatic drainage: ovaries: para aortic
38. testis: para aortic
39. scrotum: superficial inguinal
40. prostate: internal iliac
41. premature neonate: right groin tender, irreducible swelling- indirect hernia
42. copd patient: unconscious after oxygen
43. bleeding from lumber puncture: epidural veins, posterior spinal arteries
44. cases of AAA; Atherosclerosis
45. Blocked Fem-distal bypass .. neo intimal hyperplasia
46. fluid in which part in supine patient: right parabolic gutter
47. wound infection: neutrophil infiltration
48. inulin: filters, neither absorbed or excreted
49. method of heat loss from open abdomen: evaporation, convection
50. perforated appendix organism: E coli
51. bendroflumethiazide MOA: Distal convoluted tubule
52. which factor is intrinsic pathway: x
53. recurrent PE with anticoagulation: IVC filter
54. PE in stable patient Rx: LMWH
55. investigation for Mesenteric ischemia: CT, Mesenteric angio
56. stroke patient investigation: duplex, head CT
57. left arm Axially vein thrombosis: Duplex
58. parkland formula: 70 kg patient 50% burn: 14 L
59. Parkland formula 90 kg patient, one leg and perineal burn
60. sudden pain in leg : thompson test
61. trauma leg: tibia posterior at 90 degree: posterior cruciate
62. Excessive lateral movement: medial collateral
63. inguinal hernia repair, numbness scrotum: genital branch of genitofemoral
64. hydrocephalus: blockage at what level, aqueduct, luschka
65. xray - hyoid bone
66. abdominal layer crossed: upper midline: linea alba
67. Pfennesital: rectus sheath
68. inguinal hernia: Ext: oblique
69. damage to nerves: humerus posterior approach: radial
70. posterior approach tom distal femur: tibial
71. Medial approach to ankle: Sephanous
72. shinning nodule on the face: BCC
73. polycythemia with renal mass: adenocarcinoma
74. barrets oesophaugus : adenocarcinoma
75. septic shock: decreased PVR, Output Increased HR
76. chest infection in sickle cell disease organism: Sterp pyogenes
77. leg weakness : artery involved: MCA, ACA
78. mass alterier thigh: biopsy, core,
79. bleeding nipple : duct excision
80. oestrogen +ive metastatic breast ca: rx: chemo,
81. numbness lateral deltoid: axillary
82. pain with shoulder movement : supraspinatous tendinitis
83. branchial cyst: which arch
84. 14 year old, leg swelling ewing sarcoma
85. delay in discharge summary: audit
86. confidential information review
87. flexion of thumb: decor policies longs
88. scaphoid fracture: after X-ray what test, MRI, CT
89. foreign body in a childL right lower lobe
90. landmark for IJV
91. stiffness in the back: ankylosing spondylitis
92. cushing syndrome: hypokalemia, hyperkalemia
93. high output jejunal fistula-hypo hyper kalemia
94. boney mets, myeloma cells: plasma
95. bleeding duodennal ulcer: gastroduodenal artery
96. enter point of in thorax of thoracic duct: t12
97. opioid use and constipation bleeding pattern, painful defecation and fresh blood
98. bladder cancer: Egyptian guy : SCC
99. lady with prolonged catheter: TCC, SCC
100. filling defect in bladder: TCC
101. itching in jaundice: bile salts
102. aDH mechanism of action
103. post phlebitis syndrome rx: compression bandage
104. rya showing joint space narrowing in 1st metatarsophalgeal joint: ?gout
105. types of fracture: spiral
106. transverse
107. oblique
108. greenstick
109. Psoas major origin:
110. Index finger dermatome: c7
111. Palmar surface of hand thumb, index and middle finger: median nerve
112. post gastrerectomy dizziness: Dumping syndrome
113. post esophagectomy feeding: NGT, PEG
MRCS PART A – 10 Jan 2017

1. Creamy breast discharge-ectasia


2. skin tethering-pagets, carcinoma
3. blood stained discharged-papilloma
4. boy hit on side of head , squamous part of temporal bone fractured, extradural
haematoma formed-middle meningeal artery
5. old alcoholic drowsy or loss of consciousness, no hx of trauma, then collapses,
then dies.- acute subdural?
6. Preterm baby drowsy, floppy,features of interventricular hmrhge-
interventricular hemorrhage
7. Fracture femur + PE signs + brain signs--- brain bilateral petechial hemorrhages
8. Hearing loss tinnitus, + some other signs, external + middle ear were normal-
acoustic neuroma schawanoma?? Cant remember options
9. Small ulcer on tip of nose, a lesion was excised from ear few months ago- options
included BCC SCC, keratoacanthoma, Herpes labialis, metastatic,
10. Ulcer painless on lower lip no cervical LNs enlarged - same options as
above
11. Some groin mass, when examined some growth found under big toe nail +
discoloration-
12. Malignant melanoma question what is good prognostic factor- 0.5 mm
thickness?
13. Pain + swelling rt thigh + external rotation of foot- anterior dislocation of
hip, posterior dislocation of hip, intracapsular neck of femur fracture, shaft of
femur fracture
14. Internal rotation of foot- options as above
15. Posterior dislocation of hip +loss of all lower leg muscle what will also be
absent- knee flexion, knee extension
16. Hit on knee+ immediate swelling + haemarthrosis+ aspiration done few
days or months later still pain- acl
17. Anterior drawer sign positive- acl
18. Hx of twisted knee ithink, Unable to full extend knee- medial meniscus
injury??
19. Calf pain artery involved- superficial femoral
20. Pelvis surgery, doin something on lateral pelvic wall, groing discomfort +
patch on medial side of thigh numb- obturator
21. Boy on bicycle, presents to mother with with severe pain scrotum, left
testes normal sight not felt what to do- immediate exploration
22. Lower anal canal lymphatics- superficial inguinal
23. Cervix lymphatics- internal iliac
24. Ovaries lymphatics- paraaortic
25. Gut rotation around which structure- coeliac axis, sma, ima, portal vein
26. Uncinate process question structure passing don’t know exactly.
27. Ant to Head of pancreas structure- pylorus, portal vein , hepatic artery,
common bile duct
28. Calot’s triangle medial boundary- hepatic artery, cystic duct ,cbd, hepatic
duct
29. Post. Duodenal perforation artery involved- gastroduodenal, gastroepiploic
30. Congenital diaphragmatic hernia intestine in chest baby dies, structure
develops from- left pleuroperitoneal membrane, right pleuroperitoneal
membrane, septum transversum
31. Flail chest, no pneumothorax but pain on inspiration- l/a 2% lignocaine,
1% or 0.5 bupivacaine, chest drain,
32. Hx of trauma Raised JVP, trachea shifted to right no breath sounds on left,
what to do- chest drain to anterior left side, chest drain to posterior right side,
ETT.
33. Incision from deltopectoral groove medially, which structure will not be
interfered- axillary ln, brachial plexus, pec major, pec minor
34. Which structure is not posterior relation of kidney- iliacus, psoas, ql,
tranversalis, diaphragm
35. Rectal bleeding, on colonscopy found bleeding from sigmoid colon, artery-
superior rectal, left colic, middle colic
36. Neonate, meconium stained front of diaper- patent urachus, patent
vitellointestinal duct
37. Sausage shaped- intusucception
38. Boy, Cherry red mass protruding from anus, bleeding – hemorrhoid,
juvenile polyp
39. Baby, Abd pain, bilious vomiting- volvulous or other options as above
40. Panproctocolectomy stoma-end ileostomy, end colostomy in LIF, loop
ileostomy, loop colostomy
41. Hartmann’s procedure stoma- end ileostomy, end colostomy in LIF, loop
ileostomy, loop colostomy
42. Bullet hit lateral to rectus abdominis at right subcostal region, which
structure injured- gallbladder, liver, spleen, kidney
43. Transpyloric plane
44. Manubriosternal angle level structure- arch meets descending aorta
45. Esophagus pierced in cm- 22, 27, 38, 40, 15
46. Aortic opening- t12
47. Fluid- 2l glucose + 1l n.s, 0.18 ns+ 5% dextrose, 3 l n/s , 3l glucose
48. Fluid resuscitation best initially- normal saline.
49. Prostate cancer questions x 2 one with obstructive urinary features,
backache- watchful waiting, surgery, turp, hormonal therapy
50. 2nd prostate question?
51. Consent- less than 16 years old, down syndrome patient , can understand
appendicitis procedure and risks- consent the patient, consent the parents, by
proxy, court , proceed without consent
52. Consent- trauma, emergency, intraabdominal bleeding- proceed without
consent
53. Consent- 4 years old, requires blood transfusion , hb deteriorating, parents
donot consent due to religious reasons- consent the patient, consent the parents,
by proxy, court jury , proceed without consent
54. Statistics- paired t test, chi square test
55. Fall on outstretched hand, no fracture but carpal tunnel syndrome features-
scaphoid dislocation, lunate dislocation
56. Shinny tense palm skin + fracture hx?- regional pain syndrome, other
options doesnt remember
a.
57. Starling forces.. calculate filtration pressure bowmen capsule hydrostatic
pressure =33 – outside hydrostatic pressure 15 – ( bowmans oncotic 10 – outside
oncotic 0) = +8
58. Baroreceptor venoconstriction first response
59. PE- t wave inversion v1-v3
60. Sternum fracture, what structure will be damaged.. – RA, RV, esophagus
61. GCS x 2 question.
62. 5% burn on torso full thickness- full thickness graft, split skin, fasciotomy
63. Arm in boiling water, full arm superficial burns, blistering- keep open,
occlusive dressing
64. Pt abduction and adduction lost root involved- t1
65. Dupytrens contracture tm- release trigger finger
66. Mortons neuroma- neurons fibrosis, synovium
67. Head injury long term feeding reuired- gastrostomy
68. Crohns patient feeding
69. Regional pain syndrome in options
70. Pulp infection block- ring block
71. Radius dislocation heart issues
72. Urine output 80 80 0 0 (hrly)—blocked catheter
73. Mycotic aneurysm- streptococcus, candida, aspergillus
74. Thyroid mass, pulsating mass on forehead- follicular ca
75. Rapidly spreading, stridor- anaplastic
76. Within 3hr presentation – femoral embolectomy
77. Dusky leg
78. Femorodistal bypass done 1 yr back, now distal pulses absent but limb
movement present
79. Aortic aneurysm 4.5 cm—ct or us
80. MRSA – patient was mrsa carrier going for surgery tm- oral vanc bd,
teicoplanin iv, mupirocin i/n, rifampicin +
81. Cvp line infection, other patients were mrsa carrier. tm
82. Esophageal ca + mediastinal mets—stent
83. Bone mets pain tm- radio
84. Gastric fundus ca. lymphatics- celiac, paraaortic
85. Pt with abdominal pain, Ca 3.7- Iv fluids or bisphosphonates
86. Ulcer on medial malleolus-
87. Pancoast tumor-t1
88. Neurosurgery bleeding posteriorly- occipital a
89. Cuboid bone anteriorly which bones are present- 4th 5th metatarsals
90. Medial side of leg numb- sephanous n
91. A treatment was started by hospital doctors, but family doctor did not
continue with that treatment.. audt question.
92. Plaster cast leg… CPL
93. Loss of foot eversion + foot dorp- common peroneal component of sciatic
nerve
94. Axillary nerve supply deltoid +…. Teres minor
95. On colonoscopy lots of polyps..type?
96. 10 yrs old bloody diarrhea---IBD
97. Colonic resection done, liver mets asymptomatic
98. Paeds age swallowed coin where it stuck.- at t4, cricoid, tyroid prominence
99. Which level of spine compression least likely to cause spastic paralysis?
100. Appendix pain started at paraumblical region….dermatome…. t10
101. Lower lip numb + muscle of mastication not working—mandibular nerve
102. Mandibular nerve foramen---ovale
103. Middle meningeal artery foramen—spinosum
104. Nystagmus, ataxia….PICA?
105. Lower limb paralysis artery territory—aca
106. Head injury gcs 15, then gcs drop and one side blown pupil???—
transtentorial herniation, hydrocephalus,
107. Sympathetics at spinal cord..—lateral horn
108. Hypercalcemia with renal stone.. Solitary parathyroid adenoma or diffuae
parathyroid hyperplasia?
EMq. acid base: what acid base equation..match with the below scenarios
109. a pt with recurrent gastric pain.. Presented with gatric outlet obstruction .
110. A pt with pyrolus stenosis
111. A pt with malignant came with lethargy vomiting
-----
112. Claundication bt improve with uphill.. Lumbar stenosis?
113. Multiple myeloma test.. protein electrophoresis
114. Spine extension pain.. Facet arthrosis?
115. Cushings triad
116. Septic shock--- dec svr inc hr inc CO?
117. A pt with hypovolenia symptoms.. Given options: decreased extracellular,
increased interstitial.. increase intracellular?
118. A pt with tension pneunothorax symptoms.. What compromised his
cardiopulnonary status .. Decreased venous return n caused decreased cardiac
output?
Emq :
119. 1. Inr deranged .. Came for bleeding haemorrhnoid underlying liver
problem . Vit k?
120. 2. Urgent op for perforation.. Inr deranged.on warfarin.
121. 3. Aortic leaking for op.. Bp lowish.. 2 negative blood?
Wound emq:
122. 1. Post colectomy.. Heel prrssure sore with skin necrosis
123. 2. Blust injury wound
124. 3. Leg burn with deep wound. 3 weeks later with ulcer 7x10 ..tibia exposed
125. Cn IX X XII origin….. Pons or medulla
126. Lytic lesions femoral fracture, where does the primary came from… breast?
127. Axillary dissection done, patient when tries to pick up knife unable to do so
because wrist flexes. – medial cord, posterior cord, median, ulnar, radial
128. Smoking Hx, + shistosoma haematobium Hx…. SCC or TCC bladder
129. Lateral horn of spinal cord…. Sympathetics
130. Patient has loin pain haematuria smoker Hb 113 g/l
131. Femoral canal lateral relation- femoral vein
132. Inguinal canal posterior wall—conjoint tendon
133. External inguinal ring is a defect in…. EOA
134. Hydrocele is surrounded by which scrotal covering-- T.V
135. Boy on bicycle severe pain in scrotum, on exam left testis palpable left not,
what to do next……Exploration
136. Trendelenberg gait muscle- Glut medius
137. Aortic opening level- 12
138. Insulin dependent glucose transporter 2 work where on kidney- PCT
139. Aldosterone action ---- Na Cl in DCT or Na K in CD
140. Renal vein relation—ant to aorta
141. Posterior duodenal ulceration artery—gastroduodenal
142. Recurrent duodenal ulcer ppi used, what will be inc in blood---gastrin
143. Gastric acidity is inc by—vagus+histamine+gastrin
144. Achalasia question??
145. Amylase inc, serum Ca 3.7 --- hyperparathyroidism (sheet fawzia question)
146. Renal transplant few years ago,PO4 is low, Ca2+ is high PTH high---tertiary
HyperPTH
147. Wound healing 7th day---angiogenesis
148. Mediastinal masses plus cervical LN biopsy . characteristics—
reedsternberg or mitotic cells
149. Polyps hx plus oral lesion (peutz jeghr)..characteristics of polyps...
150. Another q from sheet.. hamartoma characteristics were given..
151. Nodule on extensor surface on elbow.....
152. That was another... hassal corpuscle..thymoma
153. Congenital diphragmatic hernia Neonate dies... embryo... right or left
pleuroperitoneal membrane defect?
154. Muscles of facial expression from which paryngeal arch.....2nd arch
155. Patient car accident... she was wearing seatbelt plus mediastinal widening...
aortic disection
156. Parietal pleura nerve supply....intercostals?
157. SA node nerve comes from.....fibers from vagus
158. Breast CA with metastis but general Ms weakness and hypotonia?
159. Hypovolemic shock decreased in extracellular fluid?
160. case post colectomy.. Heel skin necrosis--- conservative mngment?
161. Thyroid ca with invasion to trachea...anaplastic
162. F.B. stuck in child...cricoid
163. A pt alleged hot water splashed over dorsal forearm.. Got blister swollen n
sensitive. Wats the answer? Given option fasciotomy, eschratomy
164. There was a bilirubin question...pale stool dark urine..
165. Fracture sternum..structure injured immediately... That was rt atrium?Rt
ventricle
166. Post renal transplant pneumonia- CMV I think???
167. Mycotic aneurysm bacteria?
168. Lymphodema pt with cellulitis after insect bite
169. Single rectal polyp in 35 man...fibroepithelial??
170. Best investigation for diverticular disease with colovesical fistula..CT
171. There was mixed respiratory and metabolic acidosis scenario rt?
172. And one compensated scenerio too. Although ph was 7.38
173. Ca with met.. Vomiting lethargy
174. FEV1/FVC > 80 restrictive picture- options, Pulmonary fibrosis,
bronchiectasis, copd, asthma
175. Posterior relations of kidney.. Iliacus not one
176. Pulmonary arteries development ...6th arch
177. Vertebral artery of subclavian
178. Resuscitation question.. 2l of glucose+ 1 L of 0.9 saline or the reverse..
2saline+ 1glucose?
179. Pneumobila question?? That was doudenum rt? First time to see the term.
180. Bronchial CA in periphery...that was?Adenocarcinoma
181. Charot triad..cholangitis.. Question
182. Skin lesion one answer was dermatofibroma
183. Prophylaxis for spleenectomy?
184. Liver failure 2,7,9,10
185. Emergency mini tracheostomy... Thyrocricoid membrane or cricoid
membrane
186. One side soft palate n tongue reduced sensation. Which part of brain got
compressed- Medulla or pons?
187. Saccular aneurysm .. What cause it?
188. Hpe shows hyperchromatic nuclei..failed maturation
189. Numbness at chin...mental Nerve?
190. A pt undergone ovarian surgery.. Post op numbness groin n mid thigh
191. Groin mass.. Swelling under big toe bed with deformed toe... Malignant
melanoma?
192. Respiratory receptors sensitivity cannot remember the scenario.
193. Extensor pollices Longus rupture question...same Hasnain discussed the
night be fore exam .👏
194. Modern sterilization for arthroscope--Ethanol oxide or Glutraldehydes
195. Post cabg.. Sternal wound with some discharge Serosanguinous fluid...
options? vanco n genta, other options?
196. A man with gyneocomastia-phyological or pituitary adenoma
197. A pt with low Na.. No hypotensive...urine ouput >3++ siadh?
198. Traumatic eye with normal xray?
199. Post travel with femoral extensive mobile thrombus?
200. Paratid surgery.. Injured facial nerve
201. One of it was pituitary adenoma
202. Ca 3.7... hyperparathyroid
203. Septic shock.... arrows...
204. Doc for anaphylactic shk
205. Otitis media effusion , deep cervical lymph node.. Cant remember scenalio
206. Best investigation in PE
207. Chrons scenario one was most of small intestine resected,,,,TPN
208. Other emq only 30cm was resected,, elimentry feeding
209. A pt got burn.. Some 3weeks later got ulcer exposing tibia.. Wats the
answer?
210. Split thickness?
211. Amount of blood in Aorta per cardiac cycle??70ml
212. I put wound debridement and delayed primary closure
213. What is true about left coronary artery?-- Passes behind pulnonary trunk
214. Siadh with euvolemia.. Wats the answer? Water deprivation??
215. Gunshot passed ant to post lat to right rectus Subcostal margin. --
gallbladder
216. Regarding mortons neuroma.-perineural fibrosis, spindle fibers sth
fibrosis.. Cant really remember

1.The pressure within the pleural space is negative with respect to atmospheric
pressure, except for which of the following?

A. At end expiration B. At end inspiration C During a Valsalva maneuver D. If the patient


has a tracheostomy E. When taking a deep breath
2. A 30-years-old alcoholic man presents with acute severe upper abdominal pain and
vomiting. He is admitted to the intensive therapy unit with a diagnosis of severe acute
pancreatitis Forty-eight hours later he develops peripheral paraesthesiae and
carpopedal spasms. The most likely underlying metabolic abnormality is:-

A. hypercalcaemia B. hypernatraemia C. hypocalcaemia D. hypokalaemia E.


hyponatraemia

3. A 50-year-old woman attends her Genera) Practitioner due to a change in


appearance. She finds difficulty removing her rings, reports: an increase ‘in shoe size
and photographs reveal a change in her facial appearance. Visual field tests are-
performed to direct confrontation. Which of the following defects is most likely to be
associated with her presentation?

A. Binasal hemianopia B. Bitemporal hemianopia C. Homonymous hemianopia D. Inferior


quadrantanopia E. Unilateral loss of vision

4. A 24-year-old man falls off his motorbike, fracturing one arm, both femurs, and
rupturing- his spleen. He required surgery and 20 units of blood. Twenty-four hours
after admission he has passed 350 ml of urine. His blood pressure is 90/50 mmHg. and
he has low skin turgor. Which of the following best describes the changes in the renin-
angiotensin system initiated at the juxtaglomerular apparatus in the patient?

Renin Concentration Aldosterone concentra


Angiotensin II concentration

A ↓ ↓ ↓

↓ ↓
B ↑

C ↑ ↓ ↓

D ↑ ↓ ↑

E ↑ ↑ ↑

5. A 35-year-old woman presents to the Emergency Department with a second attack


of painful frank haematuria. What is the most likely pathology underlying this
presentation?

A. Gram negative urinary tract infection B. Interstitial cystitis C. Renal cell carcinoma D.
Transitional cell carcinoma of bladder E. Urinary tuberculosis

6. 60-year-old man presents with a short history of pain in the right cheek and right
upper teeth Maxillary sinus infection is diagnosed. This sinus is particularly prone to
infection because of?

A. absence of cellia on the epithelium lining the sinus B. closeness of the sinus to the nasal
cavity C. poor blood supply D. Position of the sinus ostium high on the medial wall E
.relationship of the front teeth to the floor of the sinus

7. A 75-year-old insulin-dependent diabetic man has undergone a hemicolectomy. On


the first postoperative day he is nil-by-mouth, on subcutaneous insulin, maintenance
IV infusion with-Saline and intravenous morphine via patient controlled analgesia. He
is confused after a brief convulsion, has slurred speech and weakness of his right side.
His pulse is 110 beats/minute, respiratory rate 25 breaths/minute, blood pressure
160/95 mmHg and Sp02 95% on room air The most likely cause of this convulsion and
confusion is:

A hypoglycemia B. hyponatremia C. inhibition of noradrenaline re-uptake D. stimulation


of ∂ opioid receptors E. stimulation of GABA receptors

8. A 30-year-old woman presents to the general surgical clinic with a 1.5 cm cervical
lymph node along the anterior border of sternocleidomastoid muscle.-Clinical
examination and routine blood tests are ; unremarkable. She undergoes an excisional
biopsy of the lump as a day case. The histology report reveals Encapsulated infiltrative
carcinoma with marked fibrosis and cystic changes within the lymph node. What is the
most likely primary pathology?

A. Anaplastic thyroid carcinoma B. Follicular thyroid carcinoma C. Malignant lymphoma


of the thyroid D. Medullary thyroid carcinoma E. Papillary thyroid carcinoma

9. A 37-year-old man presents with severe headache, photophobia arid neck stiffness.
Karnig's sign is positive and a midline lumbar puncture is performed immediately to
determine if a cerebrospinal fluid (CSF) pathogen is involved. In performing this
procedure, which is 'he first of the following structures to be pierced by the lumbar
puncture needle?

A. Anterior longitudinal ligament B. Arachnoid mater C. Dura mater D Interspinous


ligament
E. Posterior longitudinal ligament

10. A 26-year-old woman presents with severe headache, photophobia and neck
stiffness. Among other investigations, a lumbar puncture is earned out and
cerebrospinal fluid is sent for microscopy and culture. Culture shows a growth of
Neisseria meningitidis. What is likely to be the predominant-cell type in the
cerebrospinal Fluid?

A. Basophils B. Eosinophils C. Lymphocytes D. Macrophages E Neutrophils

11. A 70-year-old woman from a nursing home presents to the! Emergency


Department with abdominal pain and vomiting. On examination, she is dehydrated
and her abdomen is distended. There is a 3 cm 4 cm swelling in her right groin which is
non-tender, and there is no cough impulse. At operation, a femoral Hernia found.
Which of the following lies medial to the neck of the hernia?

A. Femoral artery B. Femoral nerve C. Femoral vein D Lacunar ligament E. Pectineal


ligament

12. A 26-year-old man presents to the Emergency Department with extensive bleeding
from his arm after sustaining a glass injury. On examination there is a 7 cm transverse
laceration across the anterior aspect of his elbow. On exploring the cubital fossa you
would expect the brachial artery to be:

A. anterior to the median nerve B. lateral to the biceps tendon C. lateral to the median
nerve D. medial to the median nerve E. superficial to the bicipital aponeurosis

13. A 52-year-old man is found to have multiple myeloma. What skull X-ray
appearances would be characteristic?

A. Diffuse thickening of the calvarium B. Hair on end appearance. C Multiple fractures D


Multiple osteolytic lesions

E. Multiple osteosclerotic lesions

14. A 50-year-old woman presents with a history of right upper quadrant pain and
jaundice. She reports that her urine was dark in colour and that her stools are offensive
and difficult to flush Which of the following explains the dark urine?

A. increase in conjugated bilirubinuria B. Increase in unconjugated bilirubinuria C.


Increase in urea excretion D. Increase in urinary urobilinogen E. Reduced enterohepatic
bile salt circulation

15. A 45-year-old mart presents with backache and leg pain due to a prolapsed lumbar
Intervertebral disc. The pain, which is aggravated by coughing and sneezing, radiates
to the lateral aspect of the foot. On examination, there is weakness of the plantar
flexors of the foot, which nerve root is most likely to be involved?

A. T12 B. L4 C. L5 D S1 E. S2

16. A 62-year-old woman presents to her General Practitioner with a two-week history
of backpain. She has lost 8 cm in height over the last five years Investigations reveal:

Which of the following is the most likely cause?

A Hypoparathyroidism B Primary hypemarathyroidism C Pseudo hypoparathyroidism D


Secondary hyperparathyroidism E Tertiary hyperparathyroidism

Findings Normal

Adjusted ( corrected) Calcium 2.78 mmol/L 2.15-2.55 mmol/L

Phosphate 0.84 mmol/L 0.8-1.4

Estimated GFR More than 90


0.96ml/minute

PTH 8.9 pmol/L 0.95-5.7

17. A 21-year-old man fractures his ulna and radius. He is treated by means of a plaster
cast, which is removed after four weeks. What pathological process is most likely to
have affected the immobilized muscles?

A. Aplasia B Atrophy C Hyperplasia D Hypertrophy E Neoplasia

18. A 22-year-old man has been stabbed in the left fifth intercostal space at the edge of
the sternum. Which structure is most likely to have been penetrated?

A Left atrium B Left lobe of the liver C Left ventricle D Right atrium E Right ventricle

19. An 80-year-old man dislocates his shoulder it is reduced in the Emergency


Department. At review three weeks later, he is unable to actively initiate abduction of
his arm. Which muscle most likely damaged?

A Clavicular head oi pectoralis major B Infraspinatus C Supraspinatus D Teres major


E Trapezius

20. An 8-year-old boy is admitted for assessment of rectal blood loss. His mother
describes this as bright red blood in the toilet pan. He has no pain on defalcation. There
is no family history. What is the most likely cause?

A. Adenomatous polyp B Familial adenomatous polyposis C Juvenile polyp D Metaplastic


polyp E. Peutz-Jegher's syndrome

21. A 48-year-old woman presents with abdominal pain and five to six episodes of
bloody diarrhoea each day. She also complains of pain- in the knees and elbows and
recent visual problems. What is the most likely diagnosis?

A. Collagenous colitis B. Diverticulitis C. Infective colitis

D. intestinal tuberculosis E Ulcerative colitis

22. A 56-year-old man presents with acute epigastric pain and vomiting. On
examination he has guarding in the upper abdomen. Investigations reveal:

Value Normal

Serum amylase 900 IU/L ≤100

Serum ALT 61 IU/l ≤50

Alkaline Phosphatase 98 IU/l 20-120

Albumin
38 g/L 35-50

Gamma GT 76 IU/L ≤60

Bilirubin 15 umol/L 0-20

Triglyceride 1.6 mmol\L ≤1.7

Corrected Calcium 3.30 mmol\L 2.20-2.70


Which of the following is the most likely etiology of this condition?

A. Alcohol B Choledocholithiasis C Hyperparathyroidism D Hypertriglyceridaemia E


Mumps

23. A 30-year-old man is admitted to the intensive care unit with an isolated severe
head injury. A CT "can shows multiple intracerebral bleeds but no midline shift. He is
intubated and ventilated. His pupils are dilated and react sluggishly to light. His heart
rate is 50 beats/minute blood pressure 170,110 mmHg and his respiratory rate is set at
10 breaths/minute. The rising blood pressure is likely to be caused by:

A. aortic and carotid baroreceptor stimulation B Cortisol stimulation C. renin-angiotensin


stimulation D. sympathetic stimulation related to blood loss E. sympathetic stimulation
related to intracranial pressure

24. A 42-year-old multiparous woman is admitted to the Emergency Department due


to pelvic discomfort. The duty gynecologist diagnoses uterine prolapse. Which
anatomical structure gives significant direct support to the uterus?

A. Cervical ligaments B. Mesometrial part of the broad ligament C Mesosalpingial part of


the broad ligament D. Round ligaments E. Transverse perinea! Muscles

25. An 80-year-old man is admitted to the surgical admission unit with central
abdominal pain. His blood pressure is 100/60 mmHg and his pulse is 110 beats/minute
with a respiratory rate of 25 breaths/minute. He is known to have severe chronic
obstructive pulmonary disease (COPD) and has been a heavy smoker all his life. On
examination he has a rigid abdomen Arterial blood gases show:

What is the most likely diagnosis?

A Hypovolemic shock complicated by COPD B Metabolic acidosis due to COPD C


Metabolic acidosis due to peritonitis D Respiratory-acidosis due to COPD

E Respiratory acidosis due to peritonitis

26. In describing the sino-atrial node, which one of the following statements is correct?

A it is part of the somatic nervous system B It is usually supplied by the left coronary artery
C If lies in the wall of the left atrium D It lies in the wall of the right ventricle E It receives
fibers derived from the vagus nerves

27. A fit 21-year-old man is admitted with an acute abdomen, subsequently diagnosed
as gastroenteritis. As part of the host immune response, which of the following
immunoglobulins and cells are correctly paired?

A. IgA-macrophage B IgD-small bowel epithelium C IgE-endothelium D IgG-plasma


cell E IgM thymocyte

28. A 45-year-old man presents with fever and pain in his right loin and groin. A soft
swelling was noted in his femoral triangle. Diagnosis of a psoas abscess was made.
Which of the following statements is most accurate regarding psoas major?

A. It arises from the lateral borders of the bodies of T12 to L5 • B. It extends the hip C. It
inserts into the greater trochanter of the femur D. It is innervated from T12 and LI

E. It passes posterior to the capsule of the hip joint

Bicarbonat
PH pO2 pCo2 Base Excess
e

Finding 7.24 9 kPa 5.3 kPA 17 mmol/L -3

Normal 7.35-7.45 11.9-13.3 4.7-6.0 22-26 -2 to +2

29. Which one of me following muscles is an extensor of the hip?

A. Adductor longus B. Gracilis C. Iliopsoas D. Pectineus

E. Semitendinosus

30. A tall, thin, 25-year-old woman presents to the surgical clinic with a complaint of
swelling in the front of her neck for the last four months. On clinical examination she
has a swelling in the left lobe of the thyroid and multiple neuromatous lesions within
the oral cavity. Her blood pressure is 220/120 mmHg. Laboratory investigations show
that her calcium and electrolytes are normal and serum calcitonin and urinary VMA
levels are elevated, Which of the following is the most likely cause of the
hypertension?

A Conn's syndrome B Cushing's disease C Essential hypertension D Phaeochromocytoma


E Renal artery stenosis

31. A 65-year-old man presents with haematuria and left loin pain. Computerised
tomography demonstrates a left renal tumour. He undergoes a left radical
nephrectomy. Where does the left renal artery lie?
A. Anterior to the left gonadal vein B Anterior to the left renal vein C Anterior to the
splenic vein D Posterior to the left renal vein

E Superior to the superior mesenteric artery

32. Within the posterior triangle, which nerve is at particular risk of damage during
surgery? A. Auricular branch of the facial B Glossopharyngeal

C Phrenic D Recurrent laryngeal E Spinal accessory

33. The right and left pulmonary arteries are derived from which of the following
embryological aortic arches? A Second aortic arch B Third aortic arch

C Fourth aortic arch D fifth aortic arch' E Sixth aortic arch

34. A 47-year-old man presents to the Emergency Department six hours after a
climbing accident initial assessment suggests that he had lost 1 L of blood. His blood
pressure is 105/70 mmHg and his pulse rate is 88 beats/minute. He is catheterised and
his bladder contains 120 ml of urine. Activation of which of the following transport
systems best describes how aldosterone leads to maintenance of the intravascular
volume and oliguria in this men?

A. Na+/glucose in the proximal tubule B. Na/H+ in the descending loop of Henle C. Na+/K
in the ascending loop of Henle D. Na+/Cl in the distal tubule E. Na/K in the collecting ducts

35. During arch aortography, the vertebral artery would be seen to arise directly from
which of the following? A. Arch of the aorta B Brachiocephalic artery

C Common carotid artery D Internal carotid artery E Subclavian artery

36. A 65-year-old man, remains, on the high dependency unit following a gastrectomy,
three days previously. His urine output: has averaged 80ml/hour for the last 24 hours.
He has one dry abdominal drain arid no nasogastric losses: His urea and electrolytes
are normal He is to stay nil by mouth for a further 24 hours. Which of the following
fluid regimes is most appropriate to his electrolyte and water requirements over the
next 24 hours?

A. 1 L of 0.9 % saline plus 1.5 L. of 4 % dextrose/0.18 % saline B. 1 L of 0.9 % saline plus 1.5
L of 5 % dextrose C. 1 L of 0.9 % saline plus 1.5 L of Hartmann's solution D. 2 L of 0.9%
saline

E. 2 L of 4 % dextrose/0,18 % saline

37. A 3-year-old boy is admitted to hospital with severe vomiting. Radiographic


examination reveals that he is suffering from annular pancreas. Which of these
structures is constricted? A First part of duodenum B Second part of duodenum C Third
part of duodenum D Proximal jejunum E Pylorus of stomach

38. A 20-year-old man with a severe head injury is being ventilated using intermittent
positive pressure. Which of the following is a physiological consequence of this? A
Decreased extracellular fluid volume B Decreased cardiac preload

C Increase in intrathoracic blood volume D Increased cardiac preload E Increased cardiac


stroke volume

39. A 36-year-old man presents with a two-month history of low back pain radiating to
his left leg After clinical examination he is referred for an MRI scan. This shows a
prolapsed intervertebral disc. The clinical signs are consistent with pressure on SI
roots. Which combination of clinical signs is most likely?

A Weak ankle plantarfiexion; altered sensation on the dorsum of the foot; normal ankle
jerk reflex B Weak ankle plantarfiexion; altered sensation on the sole of the foot; loss of
ankle jerk reflex. C Weak hallux dorsiflexion; altered sensation on the anterior surface of
the leg; loss-of ankle jerk reflex. D. Weak hallux dorsiflexion; altered sensation on the
dorsum of the foot; normal ankle jerk reflex. E. Weak hallux dorsiflexion; altered sensation
on the sole of the foot; loss of knee jerk reflex

40. 55-year-old woman undergoes a cholecystectomy; The gallbladder contains


multiple dark irregular small stones. The most likely, cause of this type of gallstone is
an increased blood level of:

A Bile acids B calcium C cholesterol D conjugated bilirubin E unconjugated bilirubin 41.


A 55-year-old man presents with a deep venous thrombosis. His full blood count shows
a haemoglobin of 18.3 g/dL (normal 13.5-17.5), a white cell count of 8 x 109/L (normal
3.9- 10.0 x 109) and a platelet count of 200 x 109/L (normal 150-400 x 109) This
hematological picture is most likely to be associated with:

A bronchial carcinoma B pancreatic carcinoma C prostatic carcinoma D renal cell


carcinoma E transitional cell carcinoma of the kidney

42. A 27-year-old has had pancolitis for the last five years. Which one of these
complications is he least likely to develop? A. Arthritis B. Cholangitis

C Iritis D Polyarteritis E Toxic megacolon

43. A surgeon makes a Pfannenstiel incision for access to the pelvic organs. He incises
theabdominal wall down to and through the rectus sheath.-He retracts the rectus
abdominis muscles laterally from the midline to expose the: A. linea alba

B. peritoneum C. posterior rectus sheath D. transversalis fascia E. transversus abdominis


muscle 44. A 25-year-old man undergoes splenectomy following blunt abdominal
trauma. Three days postoperatively venous blood testing is likely to reveal: A. high
platelet count B. high red cell count C. low platelet count D. low while cell count E. low
red cell count

45. A 46-year-old woman is seen in the pre-admission clinic. She had a myocardial
infarction two years ago. On examination she is in atrial fibrillation and a recent
echocardiogram shows that she has a dilated left ventricle: She is on warfarin and her
INR is 6.1. Warfarin inhibits which of the following?

A. Antithrombin B. Kallikrein C Plasminogen D Prothrombin E. von Willebrand's factor

46. A 75-year-old man is admitted for elective repair of an abdominal aortic aneurysm.
Following the operation his systolic blood pressure falls below 90 mmHg and during
resuscitation he is given an infusion of fluids. There is a choice of colloid or crystalloid.
Which of the following best describes why the intensive therapy unit registrar decides
to administer a colloid?

A Colloids are not freely filterable across semi-permeable membranes B Colloids can
prevent pulmonary oedema by inducing fluid flow out of the interstitial space C Colloids
decrease the transcapillary oncotic pressure gradient D Colloids expand the plasma
volume by 200 ml for each liter infused E Colloids migrate from the intravascular space

47. A 35-year-old man develops septicaemia following surgery for perforated acute
appendicitis He is hypotensive. Arterial blood gas analysis reveals:

What is the most likely explanation for these readings?

Bicarbonat
pO2 pCo2
PH e

Finding 7.25 7.2 kPa 7.5 kPA 17 mmol/L

Normal
7.35-7.45 11.9-13.3 4.7-6.0 22-26

A Compensated metabolic acidosis B Compensated respiratory acidosis C Mixed


metabolic and respiratory acidosis D Uncompensated metabolic acidosis E
Uncompensated respiratory acidosis

48. A.20-year-old man presents to the Emergency Department with a stab injury to the
thenar eminence. On examination he is found to have a 2 cm long laceration with loss
of sensation in the thumb and index finger and weakness: of the thenar muscles.
Which of the following structures is most likely to have been injured?

A. Anterior interosseous nerve B. Recurrent branch of median nerve C. Sensory and


motor branches of median nerve D. Sensory and motor branches of radial nerve E. Sensory
and motor branches of ulnar nerve

49. A 65-year-old man presents with vomiting and .weight loss. On examination he
had 3 palpable epigastric mass. The transpyloric plane lies at which; vertebral level?

A T11 B T12 C L1 D L2 E L3 50. A 30-year-cid motorcyclist is brought to the


Emergency Department after a road traffic accident. He has an open fracture of the
right femur. On arrival he is tachypnoeic and. confused, with cold and clammy skin.
Which of the following physiological changes is most likely to occur?

A. Decreased reabsorption of water from the collecting tubules B Decreased serum


bicarbonate level C Increased pH of blood D. Increased secretion of sodium in the urine

E. increased synthesis of glycogen in the liver

51. In an emergency situation what is the most appropriate surgical method of


obtaining an airway? A Cutting and retracting the cricothyroid muscle B Cutting the
thyrohyoid membrane .

C Dividing the thyroid isthmus D Entering the trachea at the C7 level E Piercing the
cricothyroid membrane

52. A 75-year-old man presents with hepatomegaly and ascites. Computerised


tomography shows evidence of post hepatic portal hypertension. The inferior vena
cava passes through the Diaphragm at which vertebral level? A. T8 B. T9 C. T10 D
T11 E T12

53. A 20-year-old woman presents acutely with abdominal pain, imaging reveals a right
ovarian cystic lesion which is excised. Histoiogicai examination shows a cyst lined by
keratinizing stratified squamous epithelium. Areas of fat, muscle, thyroid and neural
tissue are seen in the wall. What is the appropriate pathological designation for this
lesion?

A Adenocarcinoma B Cystadenoma C Dysgerminoma D Squamous cell carcinoma E


Teratoma

54. A 25-year old woman who is two months post-partum, presents with a tender mass
in the left breast. She is pyrexial with a raised white cell count and a raised C-reactive
protein level.-A diagnosis of lactational mastitis with abscess is made. Which of the
following is the most likely causative organism?

A Aeromonas hydrophilia B Escherichia coli C Streptococcus' miileri D Lactobacillus caseii

E Staphylococcus aureus

55. A 62-year-oid alcoholic man is admitted with severe acute pancreatitis. Three days
after admission he becomes hypoxic (pOa is 7.5 kPa) and confused. Chest X-ray shows
a uniform opacification of both lung fields. Which of the following is the most likely
clinical diagnosis?

A Acute left ventricular failure B Adult respiratory distress syndrome C. Bilateral


bronchopneumonia D. Bilateral lobar pneumonia E. Massive pulmonary embolism

56. A 58-year-oid woman presented to the Emergency Department with a large


fluctuant swelling the site of a recent insect bite. She is anxious, tachycardic, and
pyrexial. An ECG shows critical fibrillation. She is noted to have a goiter. The swelling
at the site of the bite requires surgical drainage. Which of the following classes of drug
would be most appropriate as part of her preoperative preparation for surgery?

A. Alpha-1 adrenoceptor agonist B Alpha-2 adrenoceptor agonist C Alpha adrenoceptor


blocker D Beta adrenoceptor agonist

E Beta adrenoceptor blocker

57. A nulliparous 30-year-old Woman presents with a recurrent painful red area in the
areola. She gives a history of smoking cigarettes. The lesion is excised and the
histology shows squamous metaplasia of lactiferous ducts. Microbiological culture
does not grow any organism. Which of the following is the most likely diagnosis?

A Breast abscess B Carcinoma of the breast C Fibroadenoma D Galactorrhea E


Subareolar abscess

58. A 65-year-old man undergoes right hemicolectomy; On the second postoperative


day he is afebrile, nauseous has a distended abdomen but very little pain. He is on
intravenous morphine via a patient-controlled analgesia pump. Plain X-ray confirms
distended loops of small intestine. The physiologic basis of postoperative paralytic
ileus involves:
A. stimulation of dopaminergic receptors B. stimulation of reversed migrating motor
complexes C. stimulation of p receptors D. suppression of muscarinic receptors. E.
suppression of motilin due to starvation

59. A 42-year-old woman has a cholecystectomy arid develops a self-limiting


postoperative wound infection. By what process would bacterial ingestion have been
enhanced?

A Apoptosis B Autophagy C Metaplasia D Opsonisation E Phagocytosis 60. A 60-year-


old non-smoker presents with three-month history of loss of weight, malaise and
breathlessness. On examination he has left supraclavicular lymph node enlargement.
Chest radiograph reveals multiple bilateral opacities. What abnormality would a biopsy
of a left supra-clavicular lymph node most probably show? A. Chronic inflammatory
cells B. Granuloma C. Langerhan's type giant cells D. Multiple abnormal mitotic figures in
cells E. Reed Sternberg cells

61. A 78-year-old woman with emphysema receiving 28% oxygen by mask has the
following blood gas results The most likely interpretation is:

A. mixed respiratory and metabolic acidosis B. partially compensated metabolic acidosis C.


partially compensated respiratory acidosis D. uncompensated metabolic acidosis

E. uncompensated respiratory acidosis

62. A 2-year-old child is referred because of an impalpable testis in the left scrotum.
Which of the following positions describes an ectopic testis? A At the deep inguinal
ring B At the root of the penis

C At the superficial inguinal ring D In the inguinal canal. E in the upper part of the
scrotum 63. A 60-year-oid man is found to have a 2 cm diameter mass in the upper
lobe of his left lung.The lesion is excised and it is found that the lesion includes
connective tissue, mature cartilage and ciliated epithelium. What is the most likely
diagnosis? A. Adenoma B. Carcinoma C. Chondroma D. Hamartoma E. Sarcoma

64. A 23-year-oid man presents three months following extraction of carious teeth
with a plaque like infiltration over the upper jaw with sinus formation. Yellow granules
are present in the discharging pus. What is the most likely causative organism? A
Actinomyces

B. Aspergillus C. Blastomyces D. Cryptococcus E. Histoplasma

65. An 82-year-old man has complete occlusion of his inferior mesenteric artery on
angiography but no symptoms or signs of colonic ischemia. Which of the following
arteries is the most likely additional source of blood supply to the territory of the
inferior mesenteric artery?

A Left colic B Left gastroepiploic C Middle colic D Splenic E Superior rectal

Bicarbonat
PH pO2 pCo2 Base Excess
e

Finding 7.28 6.2 kPa 8 kPA 36 mmol/L +5

Normal 7.35-7.45 11.9-13.3 4.7-6.0 22-26 -2 to +2

66. An intravenous drug abuser is having an echocardiogram for suspected


endocarditis. Closure of the tricuspid valve occurs at the onset of which phase of the
cardiac cycle?

A. Atrial systole B Isovolumetric contraction C Isovolumetric relaxation D Rapid ejection E


Rapid ventricular filling

67. After being stabbed in the the left groin, a previously healthy 20-year-old man was
admitted to The emergency Department. He developed hemorrhagic shock. During
resuscitation, 10 units of type O negative red cells and 6 L of colloid were
administered; Twenty-four hours later, he noted to have severe dyspnoea for which he
required intubation and ventilation on the intensive care unit. Which of the following
would be the most likely cause of his respiratory insufficiency?

A. ABO blood incompatibility B. Adult respiratory distress syndrome (ARDS) C.


Congestive cardiac failure D. Tension pneumothorax E. Viral pneumonitis 68. An 80-
year-old man with a history of hypertension, controlled with thiazide diuretics.
undergoes transurethral resection of prostate under general anaesthetic. The
surgery.is prolonged and in the recovery room he complains of nausea and a headache.
He later becomes agitated and confused. Which of the following is the most likely
diagnosis?

A Hypercalcaemia B Hyperuricaemia C Hypoglycaemia D Hypokalaemia E Hyponatremia

69. A 35-year-old man is admitted to hospital with vomiting, nausea and severe
headaches. An MRI scan reveals a tumour of the cerebellopontine angle. Which one of
the following pairs of cranial nerves is most likely to be compressed by this tumour? A
Accessory and vagus B Facial and vagus C Facial and vestibulocochlear D
Glossopharyngeal and vestibulocochlear E Vagus and vestibulocochlear

70. After resection of d rectal tumour a patient experiences erectile dysfunction. Which
of the following nerves is most likely to have been damaged in surgery?

A Genitofemoral nerve B Lumbosacral plexus C Pelvic splanchnic nerves

D Perineal branch of S4 E Pudendal nerve 71. A 45-year-old woman presents with pain,
swelling and stiffness affecting all her metacarpophalangeal joints. She has noticed
weakness in her grip and her handwriting has changed. She is unable to extend her
fingers fully. The index, middle and ring fingers in both hands show an ulnar deviation.
The most likely diagnosis is: A Dupuytren's contracture B gouty arthritis C
osteoarthritis D radial nerve palsy E rheumatoid arthritis 72. A 22-year-old man arrives
to the Emergency Department with sudden breath less ness due to a large
pneumothorax. A chest drain is inserted into the fifth left intercostal space in 5th mid-
axillary line. There is haemorrhape into the drainage bottle. Which of the following
structures is the most likely cause of this acute haemorrhage? A Intercostal artery B
Left pericardiophrenic artery C Lingula of the lung D Right ventricle of the heart E
Spleen 73. 65-year-old woman collapses after a total hip replacement. A pulmonary
embolism is suspected. Which of the following electrocardiogram changes would
support this diagnosis?

A Dominant R wave in V6 B Left axis deviation C Left bundle branch block D ST elevation
in V1-V3 /

E T wave inversion in V1-V3

74. 40-year-old woman had the anterior lobe of the pituitary removed because of a
tumour. Without postoperative supplements, which of the following could occur?

A Failure to produce adequate amounts of thyroxine B Fail to produce parathyroid


hormone in response to hypocalcaemia C Failure to secrete catecholamine in response to
stress D Failure to secrete insulin in hyperglycemia E inability to concentrate urine in
response to water deprivation

75. A 38-year-oid man in end-stage renal failure resulting from polycystic kidney
disease receives a cadaveric renal transplant. Good renal function is established but
four weeks later deteriorates, the serum creatinine rising by 25%. Which of the
following processes is most likely to be responsible for this deterioration?

A B-cell mediated rejection B Circulating immune complex disease


C IgG antibody mediated rejection D Post-transplant lymphoproliferative disorder E T-cell
mediated rejection

76. In order to expose the right axillary artery, a transverse skin incision is typically
made below the clavicle from a point just lateral to the sternal end of the clavicle to the
deltopectoral groove. Which of the following structures would be encountered in the
dissection down to the vessel?

A Lateral thoracic artery B Phrenic nerve C Suprascapular artery D Thoracic duct

E Thoraco-acromial artery

77. A 4-year-old boy presents to the Emergency Department with a two-day history of
headache vomiting and drowsiness. A CT scan reveals dilatation of both his lateral
ventricles and his third cerebral ventricle. His fourth ventricle was of normal size. It is
suspected that he has an obstruction to his cerebrospinal fluid flow. At which of the
following sites is the obstruction most likely to be?

A Cerebral aqueduct (of Sylvius) B Infundibular recess C Interventricular foramen (of


Monro) D Lateral foramen of fourth ventricle (foramen of Luschka) E Median foramen of
fourth ventricle (foramen of Magendie) 78. A 12-year-old boy presents to the
Emergency Department two hours after helping his father cut the grass. He complains
of rhinorrhea, itchy eyes, sneezing and a blocked nose. He is apyrexial with a
haemoglobin of 12.2 g/dl and white blood cell count of 6.8 * 109/L with a raised
eosinophil count. Chest X-ray is clear. Which immunoglobulin is most likely to cause
this reaction?

A IgA B IgD C IgE D IgG E IgM

79. A 35-year-old woman undergoes gastric bypass surgery for morbid obesity. At a
subesequent surgical clinic review she complains of dizziness, sweating, palpitation
and collapsing episodes after big meals without any vomiting or pain. she is otherwise
well and has no medical problems. What is the most likely explanation for her
symptoms?

A. Operative denervation of stomach B. Release of gastrin C. Release of glucagon D.


Release of insulin

E. Vasovagal syncope

80. A 56-year-old man with chronic emphysema is on the high dependency unit (HDU),
ten days after anterior resection. He has developed acute shortness of breath' and
hypotension): Which of the following landmarks would be the most appropriate to use
to obtain a femoral arterial blood gas sample?

A. A point midway between the anterior superior iliac spine and the pubic tubercle B. A
point midway between the anterior superior iliac-spine and the' pubic symphysis C. A
point midway between the greater trochanter and the pubic symphysis D. A point 1 cm
inferior and 4 cm lateral to the pubic tubercle E. A point 2 cm lateral to the midway point
between the anterior superior iliac spine and pubic symphysis

81. A 70-year-old man complains of persistent numbness since.an inguinal hernia


repair one year previously. The numbness affects the top of the scrotum, root of penis,
and a small area below the medial part of the inguinal ligament. Which nerve is likely
to have been damaged?

A. Femoral branch of the genitofemoral nerve B. Genital branch of the genitofemoral


nerve C. lliohypogastric nerve D Ilioinguinal nerve

E. Medial cutaneous branch of the femoral nerve

82. A 12-year-old girl has a diastolic murmur. It is maximally audible in the second left
intercostal space. Pathology of which structure is the most likely cause? A. Aortic
valve B. Ductus arteriosus

C. Mitral valve D. Pulmonary valve E. Tricuspid valve

83. A 34-year-old pregnant woman develops a swollen leg. Her mother and maternal
aunt also had similar problems during their pregnancies. Which of the following tests is
likely to be positive? A. Antiendomysial antibodies

B. Antimitochondrial, antibodies C. Antinuclear antibodies D Antiphospholipid antibodies


E. Antithyroglobuiin antibodies

84. The "fight or flight", response produces a release of epinephrine (adrenaline). What
is the primary metabolic effect of epinephrine?

A. Alanine shunt activation B. Cortisol release C. Glycolysis

D Tachycardia E Vasoconstriction

85. A 58-year-old-man underwent an emergency appendectomy. Which of the


following physiological parameters are consistent with sepsis?

Heart Rate Systemic Vascular Resistance


Cardiac output
A ↓ ↓ ↑

B ↑ ↑ ↓

C ↑ ↓

D ↓ ↑ ↓

E ↑ ↓

83. A 50-year-old man with a 30-year history of pancolitis undergoes surveillance


colonoscopy which reveals a plaque-like lesion in the descending colon. Biopsy reveals
a pre-malignant change. What is the name of this pre-malignant change? A
Anaplasia B Dysplasia C. Hyperplasia D. Metaplasia E. Neoplasia

87. A 60-year-old man undergoes cystectomy for a bladder carcinoma. During surgery,
the ureters are identified. On which region of the bladder do the ureters pierce the
bladder wall? A. Anterior surface

B Apex C Lateral surfaces D Neck E Posterior surface

88. A year-old patient in the intensive care unit has a tracheostomy performed via the
second, third and fourth tracheal rings. Which intervening structure is most likely to
require transection?

A Anterior jugular vein B Inferior thyroid veins C Sternothyroid muscle D Thymus

E Thyroid isthmus

89. A 23-year old man is assaulted and sustains a stabbing injury to the left groin. The
wound- is cleaned and a simple dressing applied; Six months later the patient returns
with pulsatile swelling at the site of the injury. The most probable: lesion present is:

A Abscess B False aneurysm of the femoral artery C Lymphocoele D Mycotic aneurysm


of the femoral artery E. Saphena varix

90. A 72-year-old man with a body mass index of 18.4 has not eaten for four days
following the removal of an adenocarcinoma from his descending colon. His urea is
found to be 12 mmol/L (normal 3.2-7.5) and creatinine 346 umol/L (normal 35-110). A
blood gas profile is ordered. The most likely set of results would be:

PH pO2 pCo2 Bicarbonate

A 7.23 13.6 kPa 3.8 kPA 13.5 mmol/L

B 7.30
8.8 kPa 8.3 kPa 30.5 mmol/L

C 7.36 11.5 kPa 5.3 kPa 30.5 mmol/L

D 7.43 16.4 kPa 3.7 kPa 21.0 mmol/L

24.0
E 7.49 12.6 kPa 3.9 kPa
mmol/L

Normal 7.35-7.45
11.9-13.3 4.7-6.0 22-26

91. A 16-year-old boy is hit on the left side of the face by a ball. There are no broken
bones but the boy complains of numbness of the face below the eye. Which nerve has
most likely been compromised? A Abducens nerve.

B Facial nerve C Glossopharyngeal nerve . D infra-orbital nerve E Zygomatic nerve

92. A 74-year-old man, who has undergone emergency major abdominal surgery two-
days previously, is noted to be confused. He has been on furosemide for mild heart
failure. The plasma sodium is 122 mmol/L Inspection of the fluid chart shows that he
has been written- up for 1L, four-hourly intravenous 5% glucose infusions. What is the
most likely cause for the hyponatremia?

A. An ACTH (Adrenocorticotrophic hormone) response to surgery B. Excessive sodium-free


intravenous fluid administration C. Osmotic effect of hyperglycemia induced by glucose
infusions D. Syndrome of inappropriate antidiuretic hormone

E. Use of loop diuretic in the long term

93. The Cori cycle is important in lactate metabolism in the Septic surgical patient It is
used to describe a pathway in which glucose is metabolised anaerobically to lactate in
one tissue and the lactate is converted back to glucose in another; Which one of the
following relies on this cycle to meet all of its energy needs?

A. Erythrocyte B. Hepatocyte C. Leukocyte D. Osleocyte E. Pneumocyte

94. After receiving an intermascular Injection in the buttock, a 25 year-old man


complains of inability to evert his foot Which nerve is most likely to have been injured?

A Common peroneal (fibular) component of sciatic B Inferior gluteal C Pudendal D


Superior gluteal

E Tibial component of sciatic

95. A 73-year-old man with a history of irregular bowel movements presents with
dysuria, pneumaturia and an Escherichia ccli urinary tract Infection. CT scans show a
mass involving the sigmoid colon and the bladder. What is the commonest cause of
this presentation?

A. Adenocarcinoma of the sigmoid colon B Colonic diverticular disease C Crohn's


disease D Transitional cell carcinoma of the bladder E Ulcerative colitis

96. A 7 year-old woman is in the recovery area arid receives 28% oxygen by mask.
Blood Gas Shows

Reduced sensitivity of which receptors is most likely to be responsible for this blood
gas picture?

A. Adrenergic receptors B. Baro receptors C Central chemo-receptors D J receptors E


Lung stretch-receptors

PH pO2 pCo2

Finding 7.2 12 kPa 10 kPA

Normal 7.35-7.45 11.9-13.3 4.7-6.0

97. A 76-year-old woman falls and sustains an inter-trochanteric fractured neck of


femur Following operative fixation her recovery is prolonged and she is discharged to a
rehabilitation unit Six weeks after her original operation she is readmitted. She is
drowsy, hypotensive and bradycardic. An ECG shows low voltage complexes and a
prolonged QT interval Under-activity of which of the following glands gives the best
explanation of the clinical picture?
A. Adrenal B Pancreas C Parathyroid D Pituitary E Thyroid

98. A 36-year-old man falls on his outstretched right hand. Examination reveals
tenderness in the anatomical snuff box. Which one of the following tendons form a
boundary of the anatomical snuff box? A Abductor pollicis brevis

B Extensor carpi radialis brevis C Extensor carpi radialis longus D Extensor indicis E
Extensor pollicis longus

99. A 65-year-old woman with metastatic breast cancer is admitted to hospital


confused and acutely unwell, with nausea and vomiting. Her vital signs are within
normal limits. What abnormality is most likely to contribute to her clinical condition?

A Hypercalcaemia B Hyperkalemia C Hypocalcaemia D Hypokalemia . E Hyponatremia'

100. A 4-year-old boy presents to the Emergency Department with a two-day history
of fever, difficulty walking and is unable to weight bear on the right leg. He has been
on oral Amoxycillin 250 mg three times a day for a chest infection over the last five
days. He is irritable with a temperature of 39.4°C. He does not allow examination and
keeps his right hip flexed and abducted. Blood tests are: white blood cell count; (WBC)
12.3 x -109/dL, C-Reactive protein (CRP) 146, haemoglobin 11.3 g/dL What is the most
likely diagnosis?

A. Acute avascular necrosis of hip B. Dislocation of hip C. Perthes disease D Septic


arthritis

E. Slipped upper femoral epiphysis

101. A 35-year-old woman presents with recurrent peptic ulceration. She is on proton
pump inhibitors and previously received Helicobacter pylori eradication therapy three
months ago. Which of the following is likely to be raised on venous blood testing?

A Cholecystokinin B Gastrin C Histamine D Pancreozymin E Secretin

102. A 55-year-old man presents with back ache. Neurological examination reveals lack
of extension of the left great toe.; Which of the following spinal cord segments are
likely to be involved? A L1, L2 & L3

B L2, L3 & L4 C L4, L5 & S1 D S1, S2, & S3 E S3, S4 & S5

103. A 34-year-old man is seen in the Emergency Department with a fibular fracture
following football match. On examination he is noted to have loss of foot eversion.
Which area of skin should be examined to confirm. loss of the cutaneous distribution of
the affected-nerve?

A. Along the lateral aspect of the foot B. Along the medial aspect of the foot C. Cross the
dorsal surface of the foot D. On the plantar Surface of the foot E. Over the heel

104. A 68-year-old man with insulin-dependent diabetes presents to the preadmission


clinic prior to surgery for rectal carcinoma. His creatinine is found to be 590 pmol/L. He
is referred to a renal unit where a measure of his glomerular filtration rate (GFR) is
made. Which of the following best describes why inulin could be used to measure his
GFR?

A It is filtered and not reabsorbed B It is filtered and reabsorbed C It is filtered, secreted


and reabsorbed' D It is filtered, secreted but not reabsorbed E It is not filtered but is
secreted and not reabsorbed

105. A 60-year-old man presents with a one-month history of headache, confusion and
recent onset of right sided hemiparesis. A CT scan demonstrates a 4 cm >- 4 cm lesion
with central necrosis in the left frontal lobe. It is seen extending across the midline
along the corpus callosum. There is extensive oedema around the lesion. Which of the
following is the most likely diagnosis of this lesion?

A Central neurocytoma B. Ependymoma C. Glioblastoma D. Meningioma

E. Oligodendroglioma

106. A 65-year-old woman presents with a 2.5 cm diameter mass in the upper outer
quadrant of the left breast with associated axillary lymphadenopathy. A core biopsy is
taken which confirms the presence of carcinoma. Which of the following types of
carcinoma is this most likely to be?

A Invasive ductal carcinoma B Invasive lobular carcinoma C Medullary carcinoma D


Mucinous carcinoma

E Tubular carcinoma

107. A 26-year-old man is admitted to the Emergency Department with multiple


peripheral fractures. He is clinically shocked. Which is the structure responsible for the
first; hemostatic response to a fall in systemic arterial blood pressure?

A. Adenohypophysis B. Baroreceptor C. Chemoreceptor D. Kidney

E. Neurohypophysis

108. Vitamin K is required for normal blood clotting. Which one of the following
statements is true about the effects and availability of vitamin K?

A Affects platelet function B Causes bleeding if taken in excess C Is provided only by fresh
food D Is depleted by broad spectrum oral antibiotics E Vitamin K absorption is affected
by resection of the terminal ileum

109. In septic shock, norepinephrine (noradrenaline) is used to increase the systemic


vascular resistance. This action is a result of the stimulation of which of the following
receptors? A

B Cβ1 Dβ2 E. ∂1

α
1

α
2

110. A 10-year-old child presents with progressive facial weakness and a squint. On
examination there is a lower motor neuron facial weakness and failure of abduction of
one eye. An MRI scan shows a small, intrinsic mass lesion within the central nervous
system. Where is this most likely to be located? A The cerebellar vermis B The
cerebellar tonsil C The medulla D The midbrain E The pons 111. A 65-year-old man,
with a history of ischaemic heart disease, has an abdomino- perineal excision of the
rectum after which he is stable. Two days later he is clammy, has cool peripheries, a
tachycardia and a blood pressure of 80/60 mmHg. His temperature is normal. What is
the most likely cause?

A. Anaphylactic shock B. Cardiogenic shock C. Hypovolaemic shock D. Neurogenic shock

E. Septic shock

112. A 55-year-old1 man presents to his General Practitioner with a four-month history
of feeling tired. The General Practitioner notes breast enlargement. Investigations
reveal:

Which of the following is the most likely cause?

A Primary hyperthyroidism B Primary hypothyroidism C Secondary hyperthyroidism D


Secondary hypothyroidism E Sick euthyroid syndrome

113. A 40-year-old woman presents with a parotid tumour. A biopsy reveals extensive
perineural invasion. Which is the most likely pathology?
A. Acinic cell carcinoma B. Adenocarcinoma C Adenoid cystic carcinoma D.
Lymphoma E. Pleomorphic salivary adenoma

114. A 50-year-old man presents with malaise, abdominal pain, weight loss, fever and
myalgia Polyarteritis nodosa is associated with all of the following except:

A. Erythema nodosum B. livedo reticularis

Result Norma
l

0.2
TSH 0.4-4.3
mIU/L

Free
6 mmol/L 9-23
T4

C mononeuritis multiplex D nailfold infarcts E palpable purpura.

115. A 30-year-old woman undergoes subtotal thyroidectomy. Five days later the
wound appears red and inflamed. Which of the following is the most likely causative
organism?

A. Haemophilus influenzae B. Proteus-mirabilis C. Pseudomonas aeruginosa D.


Staphylococcus aureus

E. Streptococcus pyogenes

116. A 50-year-old man complains of recent onset diplopia. On examination he is


unable to lookinwards and downwards with his right eye; The most likely diagnosis is a
lesion of the?

A. abducens nerve B. ciliary ganglion C. nasociliary nerve D. oculomotor nerve E. trochlear


nerve

117. A 57-year-old heart transplant recipient is keen-to join the cardiac, rehabilitation
programme Which of the following factors is most likely to increase cardiac output in
this patient during moderate exercise?

A. Decreased negative intrathoracic pressure B. Decreased venous tone C Decreased


ventricular compliance D Increased atrial filling ITS

E. Increased intrapericardial Pressure


118. A 69-year man with recta! cancer undergoes low anterior resection. The liver is
disease free. The pathology of his tumour was reported as a large tumour invading
from the rectum into the mesorectum. Two nodes of 24 lymph nodes, close to the
tumour were carcinoma on histology. The tumour was completely excised. What is the
correct pathological staging of his tumour?

A Dukes' A B Dukes' B C Dukes' C D Dukes' D E T2 N1

119. A 21 -year-old man is admitted to the Emergency Department with a stab injury to
his right chest. Pulse rate is 110 beats/minute and blood pressure is 85/40 mmHg.
Chest X-ray shows a large right haemothorax. and a very small light apical
pneumothorax. Which is the first substance secreted in the process leading to
increased renal reabsorption of sodium in response to the above injury?

A. Angiotensin I B. Angiotensin 11 C. Angiotensinogen D. Antidiuretic hormone. E. Renin

120. During the second (proliferative) phase of wound healing the predominant cells in
the wound site are fibroblasts. This cell is of mesenchymal origin and produces the
matrix and collagen needed to strengthen the scar. Cross linkage of collagen requires
hydroxy-proline and hydroxy-lysine residues, which requires a specific vitamin to be
available in sufficient quantities. Deficiency of which vitamin results in collagen that is
unstable?

A Vitamin B2 (riboflavine) B Vitamin B6 (pyridoxine) C Vitamin C (ascorbic acid) D


Vitamin D (cholecalciferol) E Vitamin E (tocopherol)

121. A 62-year-old woman presents with a firm irregular mass in the upper outer
quadrant of the right breast. This appears malignant on mammography, and fine
needle aspiration cytology is reported as Co. Which is the first lymph node to which the
tumour is most likely to metastasise?

A Initial node B Primary riode C Secondary node D Sentinel node E Virchow's node

122. Which of the following statements is true with relation to cardiac muscle?

A Contraction does not involve release of Ca by the sarcoplasmic reticulum B. Contraction


is triggered by an influx of Ca through the sarcolemma C. Excitation is passed from one
cardiac cell to another through desmosomes D. Hypertrophy is brought about by division
of existing cardiac cells

E. There is capacity for regeneration

123. A 70-year-old retired farm worker presents with a scaly lesion on the back of his
left wrist. Biopsy shows enlarged pleomorphic squamous cells with mitoses that have
breached the basement membrane. What is the name of this process?

A. Dysplasia B. Hyperplasia C. Hypertrophy D Metaplasia E Neoplasia

124. A 45-year-old man has established cirrhosis. At a follow-up appointment a


palpable spleen four fingers breadths below the costal margin, is noticed. Full blood
counts have shown a persistent thrombocytopaenia. Bone marrow examination shows
megakaryocyte hyperplasia. What is the most likely cause of the thrombocytopaenia?

A. Ineffective production in the bone marrow B. Platelet destruction in the bone marrow C.
Platelet destruction in the liver D Platelet destruction in the spleen

E Platelet storage in the spleen

125. A surgeon is carrying out an elective splenectomy for congenital spherocytosis.


Which structure will be divided in order to mobilize the spleen from the posterior
abdominal wall?

A Gastrosplenic ligament B Lesser omentum C Lienorenal ligament D Phrenicocolic


ligament E. Short gastric vessels

126. A 50-year-old man is admitted with epigastric pain and a rigid abdomen He is
diagnosed with, a perforated duodenal ulcer. Ulceration due to excessive gastrin
activity may be: caused by increased levels of?

A. adrenaline (epinephrine) B. calcitonin C glucagon D secretin

E. somatostatin

127. A 66-year-old woman with known tumour of her spine has started to retain urine
and is experiencing decreased anal and rectal tone. The tumour is pressing directly on
the conus medullaris. At which one of the following vertebral levels is the tumour most
likely to be located?

A. T9/T10 B T11/T12 C L1/L2 D L3/C4

E L5/S1

128. A 26-year-old man presents with a two-month history of unilateral testicular


swelling. An ultrasound scan shows a heterogeneous mass within the testis with
surrounding fluid. His blood test show- an elevated alpha fetoprotein level. Which of
the following is the most likely diagnosis?

A Lymphoma B Orchitis C Seminoma D Teratoma E Tuberculosis

129. A 50-year-old woman presents with a history of faecal incontinence over the past
few year She had a prolonged and difficult first stage of labor 20 years previously.
Physical examination reveals a relatively lax anal sphincter. Which nerve is likely to
have been damaged in labour?

A Autonomic nerves to the rectum B Genitofemoral nerve C Lumbosacral trunk D


Obturator nerve

E Pudendal nerve

130. A 70-year-old man is admitted to hospital with a 12-hour history of a painful white
leg. Afemoral artery embolus is shown on angiography. He is an insulin-controlled
diabetic and takes ibuprofen for longstanding osteoarthritis. Twenty-four hours after
emergency embolectomy his blood pressure is 90/60 mmHg and he passes very dark
brown urine.

A dipstick shows myoglobin and traces of blood. His blood glucose is 15 mmol/L
(normal- 4.0-6.0), urea 12 mmol/L (normal 3.2-7.5) and creatinine 180 mmol/L (normal
35-110). What is the most likely cause of his abnormal renal function?

A Drug induced nephropathy B Hypovoiaemia C Methaemoglobinuria D Rhabdornyolysis

E. Uncontrolled diabetes

131. An 86-year-old woman, with a history of partial gastrectomy, is in the high


dependency unit for a chest infection that is being treated with antibiotics. She
complains of difficulty swallowing. Endoscopy shows multiple white plaques on the
mucosal surface of the oesophagus. Biopsy is most likely to show: A. acid reflux
oesophagitis B Barrett's oesophagitis C candida oesophagitis : D eosinophilic
oesophagitis E. herpes oesophagitis

132. A 65-year-old woman has been given a total of 15 mg of morphine/over a 30-


minute period for pain relief following a femoral neck fracture, mellowing observations
are found: Sp02 on 80% oxygen is 80%; blood pressure 90/60 respiratory rate 6
breaths/minute.-The immediate intervention should be:

A. high flow oxygen B, intravenous fluid replacement C intravenous Naloxone D. monitor


her ECG E. tracheal intubation
133. The following drugs are used to treat patients with peripheral vascular disease.
They all reduce the rate of myocardial infarction and stroke except: A. A.C.E
inhibitors B. antihypertensive medication

C. aspirin D. cholesterol lowering drugs E. oxypentifylline

134. A 4-year-old boy, is brought to the Emergency Department with a painful right
elbow. His father stales that he was swinging his son by his arms when the pain came
on suddenly An X-ray shows that the radial head is displaced from its usual position.
What is the ligament (of the proximal radio-ulnar joint) that holds the radial head in
place?

A Annular ligament B Conoid ligament C Quadrate ligament D Radial ligament

E Ulnar ligament

135. A 35-year-old man presents with a three-week history of low back pain and a
three- day history of pain and weakness in his left leg. Physical examination
demonstrates numbness over the posterior aspect of the left calf extending to the
lateral aspect of the foot. Left ankle reflex is absent. A magnetic resonance scan shows
compression of the left S1 nerve root. What is the most likely responsible structure?

A. Annulus fibrosus B. Anterior longitudinal ligament C. Nucleus pulposus D. posterior


longitudinal ligament E. Vertebral end plate

End of paper 1

1. A 26 years old man motorcyclist loses 1 L of blood secondary to an open fracture of


the femur sustained in RTA. Which of the following is the most likely earliest
compensatory response in hypovolemeia in the this patient?

A. Baroreceptors induced venoconstriction B Capillary fluid shifts C Decreased atrial


natriuretic peptide (AMP),secretion D Renal fluid retention E Renin Angiotensin system
activation

2. Histology of a discrete palpable lump in the breast o 1 a 34-year-oM woman has


shown Apocrine metaplasia, epithelial overgrowth and papillary projections. What is
the most Likely pathological process ?

A Benign breast cyst B Carcinoma of the breast C Fibroadenoma D Phylloids tumor E


Plasma mastiditis

3. A 35 -year-old woman 'presents with recurrent peptic ulceration. She is on a proton


pump inhibitor previously received Helicobacter pylori eradication therapy three
months ago. Which of the following is likely to be raised on venous blood testing?

A Cholecystokinin B Gastrin C Histamine D Pancreozymin R Secretin

4. 3 Year old male boy presented with his mother for rectal blood loss, his mother
describes it as a bright red blood in the toilet pan. He has no pain on defecation with
negative family history. What is the most likely cause?

A-Adenomatous polyp B Juvenile polyp C Metastatic polyp D Multiple polyposis coli E


peutz jegher syndrome

5. A 32-years old man presents with a painful torticollis there is no past medical history
and his only complaint is that he has been feeling rather tired over the last 4 nights. On
examination, he has large rubbery mass in the lateral aspect of his neck as well as a
few smaller masses along his internal jugular vein. You correctly assume that the
muscular neck spasm and the large mass are connected. The most likely cause of his
torticollis is due to pressure on?

A. Ansa cervicalis B. Cervical plexus branch C. Cranial accessory nerve D. Spinal accessory
nerve E. Vagus nerve

6. A thin menopausal 52-vear-frld woman is diagnosed with breast cancer. Her


menarche was at the age of 14. she had her first child at age40 years. During lactation,
she developed breast abscess that necessitated surgical intervention. She has no
family history of breast cancer. Which of the following is the most significant risk
factor for

this patient?

A Age at first full term pregnancy B Familial predisposition C History of breast abscess D
Interval between in menarche and menopause E Physical stature

7. A 30-ycar-old woman is sent to the outpatient clinic with weight loss of 5 kg over the
Last 6 months, She also complains of anxiety, panic attacks and palpations. On
Examination. There is a swelling in the anterior neck which moves on swallowing
Which of the following is most likely pathology underlying this presentation?

A Graves disease B Hasliimoto's thyroiditis C Metastasis to the thyroid D. Papillary


carcinoma E Undifferentiated carcinoma

8. A 69-yeac-oJd man has been admitted to the high dependency unit following an
anterior resection under general anesthesia. He was given 2 mg of intrathecal
morphine. On examination, he looks pale and drowsy. Arterial blood gasses results
are:

Which o f the following is the most likely diagnosis?

A. Diabetic ketoacidosis B. Metabolic acidosis C. Metabolic alkalosis D Respiratory


acidosis E Respiratory Alkalosis

PH Base Gluose
pCo2 pO2 Lactate
Excess

Finding 7.28 8.1 kPA 10.2 kPa -2.1 21 mmol/L 4.0 mmol/L

Normal 7.35-7.45 4.7-6.0 11.9-13.3 -2 to +2 4-7 0.5-2.2

9. A 32 year-old woman has a pigmented lesion excised from her left calf. The
histopathological diagnosis is melanoma in situ, which is completely excised with a- 1
cm margin. What is the next most appropriate management?

A Education about skin self-examination and discharge from follow up B. Elective inguinal
lymph node dissection C Re excision with 2 cm margin D. Removal of any other
pigmented lesions

E. Sentinel lymph node biopsy

10. A 60-year-old man with a past history of angina undergoes an uncomplicated


operation, for an inguinal hernia. Postoperatively he is found to be hypotensive,
tachycardic, and has a raised jugular venous pressure. What is the most likely
Underlying cause Of his hypotension?

A. Reduced afterload B. Reduced parasympathetic tone G Reduced preload D. Reduced


stroke volume E. Reduced sympathetic tone

11. A 4-years old boy presents to the Emergency Department with a two-day history of
fever, difficulty walking and is unable to weight bear on the right leg. He has been on
oral Amoxycillin 250 mg three times a day for a chest infection over the last five days.
He is irritable with a temperature of 39.4°C. He does not allow examination

and keeps his right hip flexed and adducted. Blood tests are, (WBC) 18.3 (CRP) 146,
haemoglobin 11.3 g/dL. What is the most likely diagnosis?
A Acute avascular necrosis of hip B Dislocation of hip C Perthes disease D Septic arthritis

E. Slipped upper femoral epiphysis

12. A 34-year-oid man is admitted to-the Emergency Department with a head injury.
On examination his Glasgow coma scale is 9. A CT scan of the brain demonstrates an
extra dural haemorrhage. Which of the following arteries is the most likely source of
this?

A. Anterior cerebral B. Middle cerebral C. Middle meningeal D Posterior cerebral H.


Superficial temporal

13. A 65-year-old man has a history of transient ischemic attacks. He is due to undergo
carotid endarterectomy. Which one of the following is true of the internal carotid
artery?

A. Begins at the level of the sixth cervical vertebra B. Divides into the anterior, middle and
posterior cerebral arteries C. Gives off the ophthalmic artery D. Is accompanied within
the skull by preganglionic sympathetic nerves E. Passes through the foramen ovale

14. A 25-year-old male athlete is in training. At rest, how many liters of blood per
minute does his heart pump out? A. 0.9 B. 2 to 3

C 5 to 6 D 8 to 10 E 15 to 20

15. A 26-year-oJd man presents to the Emergency Department after sustaining a glass
cut to his arm. On examination there is a 10 cm longitudinal laceration on the anterior
aspect of his upper arm. He has symptoms suggestive of ulnar nerve injury. On
exploring the upper part of the arm we would expect the ulnar nerve to be:

A. anterior then medial to the brachial artery B. medial then anterior to the brachial artery
C medial to the brachial artery D. posterior then medial to the brachial artery E. posterior
to the brachial artery

16. A 55-year-old man presents with acute back pain following a severe road traffic
accident. Neurological examination reveals lack of sensation of the umbilicus and
below. What is the spinal level of the neurological deficit likely to be?

A T8 B T9 C T10 D T11 E T12 17. A- 65-year-oid man complains of being thirsty and
getting up in the middle of the night to get to the toilet. His weight is 95.5 kg. height
1.65 in and blood pressure 167/94 mmHg. An oral glucose tolerance test was
performed and produced the following results:

The most likely diagnosis:


A Diabetes insipidus B Diabetes mellitus C Impaired fasting Glycemia D impaired glucose
tolerance E Normoglycemia

Fasting plasma glucose 5.9 mmol/L

Two hours plasma


11.3 mmol/L
glucose

18. in a 9-year-old child with cellulitis of the hand, which chronological sequence of
immunoglobulin production is correct?

A. IgD precedes IgE production B IgG precedes IgA production C IgG precedes IgE
production D IgM precedes IgA production E IgM precedes IgG production

19. A 60-year»old man presents to the-Emergency Department with epistaxis. The


source 0f the bleeding is identified as Little's area and resolves with direct cautery.
Which vessel is most likely responsible for the bleeding?

A. Anterior ethmoidal artery B. Infraorbital artery C. Middle meningeal artery D


Sphenopalatine artery

E. Supratrochlear artery

20. A 75-year-old man presents with esophageal reflux. Endoscopy confirms the
Presence of a hiatus hernia. The esophagus passes through the diaphragm at which
level?

A T8 B T9 C T10 D T11 E T12

21. A 78-year-ojd woman with emphysema receiving 28% oxygen by mask has the
Following blood gas results

Which of the following is the most likely interpretation?

A Mixed respiratory and metabolic Acidosis B Partially compensated metabolic acidosis C


Partially compensated respiratory acidosis D Uncompensated metabolic acidosis

E Uncompensated respiratory acidosis

Bicarbonat
PH pO2 pCo2 Base Excess
e

Finding 7.28 6.2 kPa 8 kPA 36 mmol/L +5


Normal 7.35-7.45
11.9-13.3 4.7-6.0 22-26 -2 to +2

22, A 26-year-old man is having a stereotactic frame fitted to his skull prior
to radiosurgery on a cerebral arteriovenous malformation. Four pins secure the
frame tightly through the scalp to the outer table of the skull, two anteriorly,
two posteriorly. On insertion of one of the-posterior pins, arterial haemorrhage is
encountered. Which artery is most likely to have been punctured?

A The ascending pharyngeal artery B The middle meningeal artery C The occipital
artery D The posterior cerebral artery

E The posterior communicating artery

23. A. 2.6-year-old man presents with a two-month history of unilateral testicular


swelling. An ultrasound scan shows a heterogeneous mass within the testis with
surrounding fluid. His blood test shows an elevated alpha fetoprotein level Which of
the following is the most likely diagnosis?

A Lymphoma B Orchitis C Seminoma D Teratoma E Tuberculosis

24. A 2 day old baby presents with increasing respiratory distress. He was born at
term By normal vaginal delivery. On examination he has cyanosis of the lower limbs
and marked respiratory in drawing of the chest His femoral pulses ate absent
bilaterally and he has been anuric for the last 2 hours. Pulse rate is 140 beats/minute,
regular, and his blood pressure is 60/30 mmhg in both upper limbs. What is the most
likely diagnosis?

A. Hypoplastic left heart syndrome B. Interrupted aortic arch C. Pulmonary atresia and
ventricular septal defect D. Transposition of the great arteries E. Tricuspid atresia

25. A 50-years old woman presents with a history of right upper quadrant pain and
jaundice. She reports that her urine was dark in colour and that her stools are offensive
and difficult to flush. Which of the following explains the dark urine?

A. Increase in conjugated bilirubinuria B. Increase in unconjugated bilirubinuria C.


increase in urea excretion D Increase in urinary urobilinogen H. Reduced enterohepatic
bile salt circulation

26. An 80-year-old woman, who has suffered a fall, is found lying on the floor, where,
she has-been for over 12 hours. Initial assessment shows that she has a core
temperature of 28°C. What would the expected early physiological response to her
body temperature be?

A. Increased fat metabolism B. Increased constriction of peripheral blood vessels C.


Increased hypothalamic set-point D. Increased metabolic rate. E. Increased thyroid
activity

27. A -12-year-old boy presents to the Emergency Department two hours after helping
his father cut the grass. He complains of rhinorrhoea, itchy eyes, sneezing and a a
blocked nose. He is apyrexial with a haemoglobin of 12.2 g/dL and white blood cell
count of 6.8 * I09/L with a raised eosinophil count. Chest X-ray is; clear. Which
immunoglobulin is most likely to cause this reaction?

A. IgA B. JgD C. IgE D. IgG E. IgM

28. A 45-year-old man presented with backache and leg pain due to –prolapsed lumbar
• intervertebral' disc. The pain, which is aggravated by coughing and sneezing;
radiates to the dorsum of the foot On examination there is weakness, of the
dorsiflexion of the foot. Which nerve root is most likely to be involved?

A. T12 8- L3 C. L5 D. S1 E. S2

29. A 75-year-old woman, who has a carcinoma of the upper rectum, undergoes
anterior resection. The arterial blood supply of the upper rectum arises from which of
the following?

A. Coeliac artery B. Iliocolic artery C. Inferior mesenteric artery D. Internal iliac artery E.
Superior mesenteric artery

30. A 65-year-old man is undergoing an abdominal aortogram, A stenosis is


demonstrated in a lateral aortic branch, arising at the level of the body of the second

lumbar vertebra. The stenosed vessel is most likely to be the:

A. coeliac artery B. inferior mesenteric artery C. left renal artery D. second left lumbar
artery E. superior mesenteric artery

31. A 40-year-old man is admitted tc the surgical day case unit for repair of his left
inguinal hernia. On examination he is noted to have diffuse skin tanning, spotty
pigmentation of the elbows, nipples and buttocks, and pigmentation of the scar from
A previous right inguinal hernia repair. Three hours after the operation be becomes
severely hypotensive. 'What is the most likely cause?

A. ACTH deficiency B. Adrenal insufficiency C. Growth hormone deficiency D. Potassium


deficiency E. Thyroxine deficiency
32. A 19-year-old woman presents to the Emergency .Department profoundly
hypovolemic having fallen from a horse. A postero-anterior (PA) chest radiograph
shows a fracture to the medial third of the left clavicle. Which of the following vessels
was-most likely damaged?

A Brachiocephalic trunk B Left axillary artery C Left common carotid artery D Left
subclavian artery E Left vertebral artery

33. 65 years old man presented with an inguino-scrotal swelling in the right groin which
is non tender. A cough impulses is elicited, At operation, an indirect inguinal hernia is
repaired. The cremasteric muscle is derived from which of the following

A. External oblique aponeurosis B Internal oblique muscle • C. Rectus abdaminis


muscle D. Rectus sheath

E. Trsusversalis fascia

34. A 70-y ear-old woman is in the recovery area and receives 28% oxygen by mask.
Blood gas shows:

PH Pco2 Po2

Findings 7.1 10.0 kPa 12 kPa

Normal 7.35- 11.9-


4.7-6.0
7.45 13.3

Reduced sensitivity of which receptors is most likely to be responsible for this blood
gasses picture? A. Adrenergic receptors B. Baro receptors

C Central chemo receptors D. J receptors E Lung stretch-receptors

35. A 28-year-old man presents with pain in his left scrotum. A diagnosis of varicocele
is made. Which vessel is involved?

A Femoral vein E. Inferior epigastric vein C. Long saphenous vein D Internal pudendal vein
E Testicular vein

36. A 60-year-old man, who is a heavy smoker, presents with a 10-day history of frank
painless haematuria. His prostate is slightly enlarged on rectal examination. his
haemoglobin is 11.3 g/dL. creatinine 84 pmol/L and prostate specific antigen (PSA.) is
3,1 mg/dL. What is the most likely pathological process?
A. Benign prostatic hyperplasia B. Prostate cancer C. Renal cell carcinoma D.
Transitional cell carcinoma of bladder E. Urinary tract infection

37-. A 34-year-old woman presents with an irregular mass in the right breast which is
clinically. radiologically and histologically malignant. Her mother died of breast cancer
at the age of 58 and her grandmother died of ovarian- cancer at the age of 55. Which
gene is most likely to be involved in the development of this woman tumor?

A BRCA1 B. hMLH1 C. KRAS D. p53

E. PTEN

38. 65-year-old man presents with a non-tender swelling in the right hemi-scrotum. At
operation, the hydrocele sac is opened and 400 ml of fluid is drained. Which anatomical
structure surrounds the fluid?

A. Dartos muscle B. Patent processus vaginalis C. Testicular capsule

D. Tunica albuginea E. Tunica vaginalis

39. A 24 years old man is brought to the Emergency Department following a road
traffic accident. He has obvious airway compromise due to mouth/ facial
haemorrhage which requires a surgical airway. The surface landmarks used to localise,
the optimal site for a tracheostomy incision are midway between the suprasternal
notch and which of the following?

A Cricoid B Hyoid C Mastoid D Laryngeal prominence E Thyroid

40. A 60-year-old man has -an anterior resection for a high rectal carcinoma. The
histopathology report indicates the lesion is Dukes' stage B. What is the approximate
average five years survival rate for patients with these lesions?

A. 10% B. 20% C. 35% D 70% E 90%

41. A 20-years man presents to the Emergency Department after accidentally tripping
over and lacerating his hand on a glass bottle. On examination there is a 2 cm
laceration on the hypothenar eminence with loss of flexion in the distal
interphalangeal joint of the little finger. What is the most likely tendon to be injured?

A Flexor digiti minimi brevis B Flexor digitorum profundus C Flexor digitorum superficialis
D Fourth palmar interosseous E Lumbirical

42 A 75-year-old man, who smokes heavily, underwent coronary artery bypass grafting
six months ago. A left internal-mammary artery graft was used. He now complains of
angina on pegging his laundry on the washing line to dry. Which of the following
lesions best explains his symptoms?

A. .Micro-emboli from the left common carotid artery B. Micro-emboli from the le\t
internal carotid artery C. Stenosis.of the subclavian artery at the level of the costo-cervical
trunk D. Stenosis of the subclavian artery distal to the insertion of scalenus anterior E.
Stenosis of the subclavian artery proximal to the first branch

45. A 50-year-old alcoholic man presents-to the Emergency Department and is found
to be unable to extend his wrist, thumb and fingers of his right hand. He is also
found to have weak extension of the right elbow joint and loss of sensation on the
dorsum of the first web space . Which nerve is most likely to have been injured?

A Median nerve B Musculocutaneous nerve C Posterior interosseous nerve D Radial


nerve B. Ulnar nerve

44. A 55 year-old man presents with acute back pain following a severe road traffic
accident. Neurological examination reveals lack of dorsi-flexion of the left ankle joint.
Which of the following spinal cord, segments are most likely to be injured?

A. L1, L2 & L3 B. L2, L3 & L4 C L4, L5 & S1 D. S1, S2& S3 E. S3, S4 & S5

45. A 21 -year-old man is admitted to the Emergency Department with a stab injury to
his right chest. Pulse rate is 110 beats/minute and blood pressure is 85/40 mmhg Chest
X-ray shows a large right haemothorax and a very small right apical pneumothorax.
Which is the first substance secreted in the process leading to increased renal
reabsorption of sodium in response to the above injury?

A. Angiotensin I B Angiotensin II C Angiotensinogen D Antidiuretic hormone E Renin

46. A 21-year-old man has been stabbed in the back of the knee, dividing the popliteal
artery, and is undergoing repair via a posterior approach. Which of the following
structures is most likely to be encountered first, when dissecting deep from the skin „
incision?

A Popliteal artery B Popliteus muscle C Popliteal vein D Tibial nerve E Soleus muscle

47. A 78-year-old woman presents with urinary urgency and incontinence. The external
urethral sphincter is innervated by which of the following nerve roots?

A. L 3, 4 & 5 B. L4, 5 & SI C. L 5, S1 & 2 D. S1, 2 & 3 E. S2.3& 4


48. A 40-.year-Qld woman presents with a parotid tumour. A biopsy reveals perineural
invasion. Which is the most likely pathology?

A. Acinic cell .carcinoma B . Adenocarcinoma C. Adenoid cystic carcinoma D.


Lymphoma E. Pleomorphic salivary adenoma

49. A 19-year-old man was assaulted and sustained injuries to the right side of his
head. After two weeks he notices dial his right eye is dry and it could not produce
tears. from which ganglion is post synaptic fibers arise to supply lacrimal gland?

A. Geniculate ganglion B. Inferior ganglion of the vagus nerve C. Otic ganglion D.


Pterygopalatine ganglion E. Superior cervical ganglion

50. A 35-year-old man undergoes a right inguinal hernia repair under genera!
anaesthetic as a day patient. He has a nerve block after the procedure. On recovery he
has weakness of the right leg. Which nerve has been affected?

A. Femoral nerve B. Genitofemoral nerve C Ilioinguinal nerve D Lateral cutaneous


nerve E Sciatic nerve 51. A man suffers a brachial plexus injury. On examination, he
has a Horner's syndrome in association with upper limb paralysis. Which nerve roots
does the Homer's syndrome suggest involvement of?

A. C2 & C3 roots B. C4 & C5 roots C. C6 & C7 roots D C8 & T1 roots E. T2 & T3 roots

52. A 60-year-old woman with breast carcinoma complains of difficulty chewing her. .
food and is found to have numbness of the lower lip on one side. CT scanning shows a
small metastatic lesion affecting the bony skull base on. The same side as the lip
numbness. Which foramen is the most likely to involve?

A. Foramen caecum B Foramen magnum C Foramen ovale D. Foramen rotundum E.


Foramen spinosum

53. Which one of the following muscles is an extensor of the hip?

A. Adductor longus B. Gracilis C. Iliopsoas D. Pectineus

E. Semitendinosus

54. In order to expose the right axillary artery, a transverse skin incision is typically
made below the clavicle from a point just lateral- to the sternal end of the clavicle to
the deltopectoral groove. Which of the following structures would be encountered in
the dissection down to the vessel?

A. Lateral thoracic artery B. Phrenic nerve C. Suprascapular artery D. Thoracic duct


E. Thoraco-acromial artery

55. A 55-year-bld man presents to the-Emergency Department after collapsing. On


examination his pulse is 124 beats/minute, blood pressure is 60/30 mmHg, respiratory
rate is 34 breaths/minute and his peripheries are warm. What is the most likely
diagnosis, for this patient?

A. Cardiac failure B. Haemorrhage C Hypovolemia D. Pulmonary embolus E. Sepsis

56. In the posterolateral approach to a posterior malleolar fracture, an incision is made


between the calcaneal (Achilles) tendon and the distal-.fibula. Which of the following
structures is at risk?

A. Deep peroneal (fibular) nerve

B. Saphenous nerve C. Superficial peroneal, (fibular) nerve D Sural nerve E. Tibial nerve

57. A 25-year-old woman undergoes.an elective right thoracoscopic procedure for


treatment of right palmar hyperhidrosis. Diathermy is applied to a neural structure
lying anterior to-the neck of tire right first rib. What is the most likely complication to
occur as a result of this procedure?

A. Bradycardia B. Hoarseness C. Homer's syndrome D. Raised right hemi diaphragm E.


Reduced right biceps tendon reflex

58 / A 33-year-old man presents to the orthopedic outpatient clinic with a six-month


history of low back pain radiating to the lateral aspect of the left upper thigh.
Micturition and defecation are normal and there is no history of previous Injury. On
examination, left lateral flexion of the spine is limited but a full range of hip
movements are observed, although it is painful. Sensation is altered over the front of
the knee. Left knee reflex is reduced. Which spinal nerve-is most likely to have been
compressed?

A. L1 B. L2 C. L3 D. L4 E. L5

59. A 2S-year old motorcyclist is admitted following a road traffic accident, having
sustained- bilateral femoral fractures and a ruptured spleen. Three days
postoperatively lie is noted to be confused, hypoxemic and difficult to ventilate. His
observations show a blood pressure, of 120/8O.mmHg, regular pulse of 88
beats/minute and he is apyrexial. A chest X-ray shows bilateral diffuse lung infiltrates.
What is the most likely underlying diagnosis? A. Adult respiratory distress syndrome B
Atelectasis C Bronchopneumonia D. Pulmonary oedema E: Pulmonary thrombo-
emboilism
60. On-an ultrasound scan of the popliteal fossa when investigating-a swelling, which
of the following structures is closest to the capsule of the knee joint? A Great (long)
saphenous vein

B Popliteal artery C Popliteal vein

D. Small (short) saphenous vein E. Tibial nerve

61. A 25 years old man sustains a twisting injury while playing football. He develops
immediate swelling of the knee and he can not continue the game. Six months later,
he Is still not able to play football. His knee feels unsteady and tends to give way. On
examination, he has a full range of knee motion. There is a positive anterior draw test
and a small effusion. What is the most likely structure damaged?

A Anterior cruciate ligament B. Lateral collateral ligament C. Medial collateral ligament -


D. Oblique popliteal ligament E. Posterior cruciate ligament

62. A 62-year-old man required a partial gastrectomy for a large benign ulcer in his
gastric antrum. When he is reviewed in clinic, six months later he complains of
palpitations, weakness and sweating along with a cramp-like abdominal pain which
occurs within an hour of eating n meal. Usually, he has to lie down for 30-40 minutes
until the symptoms subside. What is the most likely cause of his symptom pattern?

A Chronic gastroparesis B Dumping syndrome C. Delayed gastric emptying D Reflux


gastritis E Recurrent ulceration 63. in septic shock, noradrenaline is used to increase
the systemic vascular resistance This action is a result of the stimulation of which of
the following ?

A B Cβ1 Dβ2 E. ∂1

64. A 56-year-old man presents with acute epigastric pain and vomiting. On
examination he has guarding in the upper abdomen His test results are:

α1

α2

Value Normal

Serum amylase 900 IU/L ≤100

Serum ALT 61 IU/l ≤50


Alkaline Phosphatase 93 IU/l 20-120

Albumin 38 g/L 35-50

Gamma GT 500 IU/L ≤60

Bilirubin 15 umol/L 0-20

2.9 mmol\L ≤1.7


Triglyceride

Which of the following is the most likely etiology of this condition?

A. Alcohol B. Choledocholithiasis C. Hypertriglyceridemia D. Hypocalcaemia E. Mump

65. A 47-year-old woman with end stage renal failure requires an


elective cholecystectomy for symptomatic gallstone disease. Her hemoglobin
preoperatively is 7.2 g/dL. What is the major cause of her anaemia?

A. Calcium deficiency B. Erythropoietin deficiency C Folate deficiency D Iron


deficiency E. Vitamin B12 deficiency

66. A 60-year-old man undergoes cystectomy a bladder carcinoma. During surgery, the
ureters are identified. On which region of the bladder do the ureters pierce the bladder
wall?

A. Anterior surface B. Apex C. Lateral surfaces D. Neck

E. Posterior surface

67. A 65 year-old man had a colonic resection for carcinoma 12 hours ago. He is now
passing concentrated urine at a rale of 0,5 ml/kg/hr. Which endocrine response is most
likely to have caused this?

A. Decreased aldosterone release B. Decreased insulin release C. Decreased thyroxine


release D Increased adreno-coriticotrophic hormone (ACTH) release E Increased
Vassopressin release

68. A 28-year-old man presents with an ischiorectal abscess. Where is ths abscess
cavity likely to be sited?

A. Above the levator ani B. Between external and internal anal sphincter C. Lateral to the
obturator internus D Medial to the internal anal-sphincter E Medial to the pudendal canal

69. A 36-year-pld man falls on his outstretched right hand.-Examination reveals •


tenderness in the anatomical snuffbox. Which one of the following tendons form a
boundary of the anatomical snuff box?

A. Abductor pollicus brevis . B Extensor carpi radialis brevis C. Extensor cairpi radialis
longus D. Extensor indicis E. Extensor poliicis longus

70. A 55-year-old- woman complains of cramps and tingling in her legs and arms 48
hours following a sub total thyroidectomy. Her vital signs, are stable. Investigations
reveals a normal white-cell count and normal haemoglobin. The NA is 132 mmol/L
K+3.2 mmoI/L and Ca 1.60 mmol/L. Serum albumin is normal The next appropriate step
is intravenous infusion of?

A. Calcium chloride B. Magnesium sulphate C. Potassium chloride D. Sodium bicarbonate


E. Sodium chloride

71. A 40-year-old. woman presents with faecal incontinence and; anatomically intact
internal and external anal- sphincters. Which structure is most likely to have been
damaged?

A. First sacral nerve-root B. Obturator nerve: C. Pelvic splanchnic nerves D. perineal nerve

E. Pudendal nerve

72. Abdominal free fluid will collect in the lowest part of the peritoneal cavity. At
operation with the patient supine, in which of the following will the fluid collect first?

A Hepatorenal pouch B. Left subphrenic space C. Lesser sac D. Right paracolic gutter E.
Right subphrenic space

73. A 45-year-old homeless man presents with a cough and weight loss over three
months. On examination his BMI (Body Mass Index) is 19 and he has reduced breath
sounds in the right upper zone. His chest X-ray shows a cavitaing lesion in the right
upper lobe. He undergoes a bronchoscopy and a brornchial biopsy. The

biopsy shows featureless necrosis surrounded by epithelioid macophages and giant


cells. Which of the following is the most likely diagnosis?

A.. Actinomycosis B Bronchiectasis C Sarcoidosis D Squamous cell carcinoma E.


Tuberculosis
74. A healthy 36 years old man is being assessed with a view to being a life related
kidney donor which are of following investigations is most accurate for measuring the
GFR?

A Creatinine clearance B Glucose Clarence C Inulin Clarence D PAH

E Urea clearance

75. A 34-year-old pregnant woman develops a swollen leg. Her mother and maternal
aunt also had a similar problem during their pregnancies. Which of the following tests
is likely to be positive?

A. Antiendomysial antibodies B. Antimitochondrial antibodies C. Antinulear antibodies D.


Antiphospholipid antibodies E. Antithyroglobulin antibodies

76. A 72-year-old alcoholic man is known to suffer from chronic obstructive pulmonary
disease (COPD). He has now been diagnosed with carcinoma of the urinary bladder
and-has a history of a gastric ulcer. Which of the following agents is most likely to be
responsible for all these problems?

A. Alcohol B Cigarette smoking C. Drugs D. Dust E. Virus

77. A 3-year-old boy is admitted to hospital with severe vomiting. Radiographic


examination and history reveals that he is suffering from annular pancreas. Which of
these structures is constricted? A. First part of duodenum

B Second part of duodenum C. Third part of duodenum D. Proximal jejunum E. Pylorus of


stomach

78. A 45 year-old man presents with a 3 cm * 4 cm swelling in the right groin which is
non-tender. A cough impulse is elicited. At operation, an indirect inguinal hernia is
found. The external inguinal ring is a defect in which of the following abdominal

A. External oblique aponeurosis B. External oblique muscle C. Internal oblique muscle D


Transversus abdominis muscle E Transversalis fascia

79. A 45-year-old woman suffers from spina bifida and is confined to a wheelchair. Her
legs are not fully developed. What is the pathological process which has occurred in the
legs?

A. Apoptosis B Atrophy C. Hyperplasia D. Hypertrophy E. Hypoplasia

80. A 23-year-old man presents to the Emergency Department-with non-specific: chest


pains. A postero-anterior (PA) chest radiograph is performed and is normal.
Immediately inferior cauidal to the outline of the aortic knuckle was a farther structure
with a convex border. What is this structure?

A. The left atrium B. The pulmonary trunk C. The oesophagus D. The right atrium E. The
superior vena cava

8I. A 45-year-old woman presents to the hospital with jaundice. She- undergoes an
ultrasound scan and is diagnosed with a hepatocellular carcinoma affecting the left
lobe of her liver. Which virus is the most likely to be implicated?

A. Epstein Bar virus (EBV) B. Hepatitis B virus (HBV) C. HIV D. Human papilloma virus
(HPV) E. Human T cell lymph a trophic virus-1 (HTLV1)

82. An 82-year-old man has complete occlusion of his inferior mesenteric artery on
angiography but no symptoms or .signs of colonic ischaemia. Which of the following
arteries is the .most likely additional source of blood supply to the territory of the
inferior mesenteric artery?

A Left colic artery B Left gastroepiploic C. Middle colic D. Splenic E. Superior rectal

83. A 56-year-old man with documented Type 1 diabetes mellitus presents to the
Surgical outpatient clinic with 3 months history of right leg pain brought on
by walking 50 meters. He has no history of back pain. He has been a smoker for 10
years (10 cigarettes a day);-He stales his alcohol consumption to be 30 units per week.
On clinical examination, significant findings include weak right foot pulses with altered
sensation on dorsum of the right toot. What is the most likely diagnosis D from the
following list?

A. Autonomic neuropathy B. Femro popliteal stenosis C. Neuropathic joint disease D.


Polyneuropathy E. Radiculopathy

84. A 35-year-old man presents with a three-week history of low back pain and a three
day history of pain and weakness in his left leg. Physical examination demonstrates
numbness over the posterior aspect of the left calf extending to the lateral aspect, of
the foot. Left ankle reflex is absent. A magnetic resonance scan shows compression of
the left S1. nerve root. What is the most likely structure compressing the nerve?

A. Annulus fibrosus B Anterior longitudinal ligament C. Nucleus pulposus D. posterior


longitudinal ligament E Vertebral end plate

85. A 72-year-old man who is a smoker presents to his doctor with a second episode of
frank hematuria. When an ultrasound scan of his urinary tract was performed it a solid
lesion in the right kidney. What is the most likely pathology underlying this
presentation?

A. Angiomyolipoma B Renal cell carcinoma, C Renal cyst D Renal oncocytoma E.


Transitional cell carcinoma

86. A 20-years old man presents with abdominal pain and shock. He is found to have
hemoperotonium Due to a ruptured spleen. He denies any history of trauma what is
the most likely predisposing cause of his splenic rupture?

A. Epstein Barr virus infection B. Human immunodeficiency virus (HIV) infection C.


Measles virus infection D. Mumps virus infection E. Varicella zoster virus infection

87. A 30-year-old woman presents to the outpatient' clinic witlr a three-month history
of diarrhoea, which is intermittently bloody. On rectal biopsy, the histology has
shown' a granulomatous inflammation. What is the most likely diagnosis?

A. Ameobiasis D. Crohn's disease C. Sarcoidosis D. Tuberculosis E. Ulcerative colitis

88. A 56 year-old motorcyclist presents to the Emergency Department after being


involved in a road traffic accident. He is a smoker and known to have hypertension. He
is conscious (Glasgow coma score of 1-3/15), and maintaining his own airway and
breathing. He is found to have an open right femoral fracture With normal distal pulses
and- sensation. Resuscitation is started. Urethral catheterisation drained 250 ml.
immediately but over the following 60 minutes he drained only 10 ml. Which of the
following is the most likely cause of his low urine output?

A. Blocked catheter B. Cardiogenic shock C. Hypovolemic shock D. Ruptured bladder E.


Septic shock

89. A 35-year-old man is admitted with pancreatitis, which leads to admission to the
surgical high dependency unit. After 48 hours, he develops breathing difficulties with
elevated, respiratory rate and increasing hypoxia. You suspect he is developing
respiratory failure. Which of the following mechanisms is responsible for the regulation
of respiration?

A Brainstem receptors producing voluntary breathing. B CO2 concentration in medulla C


Elevated bicarbonate levels in cerebrospinal fluid D. Hydrogen ion diffusion across blood
brain barrier E. Hypoxic stimulation' of chemoreceptors

90. A 70-year-old man undergoes transurethral resection of bladder uimonr (TURBT).


The tumour lies over the lateral side wall of the bladder just above the opening of the
right ureteric orifice. Use of the diathermy suddenly causes the patient to 'kick'
because of contraction of the hip adductors muscles. Which nerve has been
stimulated?

A Femoral nerve B Genitofemoral nerve C Lateral femoral cutaneous nerve D Obturator


nerve E Sciatic nerve

91. Witch of the following statements is true with relation to cardiac muscle?

A. Contraction does not involve-release of Ca 2 + by the sarcoplasmic reticulum B.


Contraction is triggered by an influx of Ca 2+ through the sacrolemma C. Excitation is
passed from one cardiac cell to another through desmoses D. Hypertrophy is brought
about by division of existing cardiac cells

E. There is capacity for regeneration

92. A 39-year-old man presents with searing left-sided loin to groin pain. Investigation.
reveals a radiodense opacity, confirmed to be in the ureter on intravenous urography.
What is the most likely composition of this man's ureteric stone?

A. Ammonium magnesium phosphate B. Calcium oxalate C. Cholesterol D. Cysteine

E . Urate

93. A 25-year-oid man is playing football when he complains of posterior thigh pain
whilst sprinting with the ball. On examination he has tenderness in the lower lateral
posterior thigh. He is unable to fully extend the knee due to pain. What is the most
likely injury?

A Biceps femoris tear B. Gracilis tear C. Gastrocnemius tear D. Semimembranosus tear E


Semiteodinosus tear

94 . A 23 years old man is assaulted and sustains a stab injury to the left groin. The
wound is cleaned and a simple dressing applied. Six months later the patient returns-
with a pulsatile swelling in the site of the injury. The most probable lesion present is:

A. Abscess 8. False aneurysm of the femoral artery C. lymphocele D. mycotic aneurysm


of the femoral artery E Saphena varix

95. 12 years old girl has a diastolic murmur it is maximally audible in the 2nd left
intercostal space. Pathology of which structure is the most likely cause ? A. Aortic
valve B. Ductus arteriosus

C Mitral valve D. Pulmonary valve E. Tricuspid valve


96. A 25-year-old man is admitted having been stabbed in the interior chest. On
examination he is alert and coherent. He has congested neck veins. His pulse is 140
beats/minute and his blood pressure 90/60 mmHg. He has normal breath sounds. What
is the most likely cause of his cardiac arrest 15 minutes later?

A. Cardiac tamponade B. Congestive cardiac failure C. Hypovolemia D Tension


pneumothorax E Ventricular arrhythmia

97. A 20-year-old woman presents acutely with abdominal pain. Imaging reveals a
right ovarian cystic lesion which is excised. Histological examination shows a cyst lined
by keratinising stratified squamous epithelium. Areas of fat,muscle, thyroid and neural
tissue are seen in the wall. What is the appropriate pathological designation for this
lesion?

A. Cystadenoma B. Dysgerminoma. C. Mesenchymoma, D. Squamous cell carcinoma E.


Teratoma

98. A 21-year-old map comes into the Emergency Department with a severe
headache and neck stiffness of recent onset. You suspect acute meningitis and decide
immediately to treat with antibiotics and perform a lumbar puncture. After your third
attempt to obtain for cerebrospinal fluid (CSF) you notice that the fluid obtained is
stained red. However during the collection of CSF into three containers it is noted that
the final bottle is now cleared of any blood. Which anatomical structure is the most
likely to be responsible for this bleeding?

A.. Anterior spinal artery B. Epidural artery C. Subarachnoid vein D. Veins within the
erector spinae muscles E. Vertebral venous plexus

99. A 25-year-old woman presented in the surgical clinic with a three-month history of
diarrhoea. On examination she is found to have a smooth 2 cm swelling in the front of
her neck her sister has a similar swelling which was operated on 2 years ago The
swelling moves on swallowing and the patient is clinically euthyroid Fine needle
aspiration cytology of the swelling shows amyloid stroma and serum calcitonin levels
are elevated. Which of the following is the most likely diagnosis?

A. Anaplastic thyroid carcinoma B Follicular thyroid carcinoma C. Medullary thyroid


carcinoma D. Metastatic thyroid carcinoma E Papillary thyroid carcinoma.

100. A 74-year-old man with known prostatic hyperplasia presents to the Emergency
Department with acute urinary obstruction. Thirty minutes after urinary
catheterization he Collapses with a blood pressure of 74/34 mmhg a heart rate of 128
beats/minute. What is the most likely diagnosis?
A. Cardiac failure B. Haemorrhagic shock C. Latex allergy D. Septic shook E. Vasovagal
syncope

101. A 70-year-old man has a basal cell carcinoma resected from his left temple. What
is the most important prognostic indicator?

A Clinical pathological type B. Completeness of excision C. intensity of the lymphocytic


infiltrate D Lymphovascular space invasion E. Mitotic index in the tumour islands

102. A 68-year-old man has undergone emergency surgery to repair a leaking aortic
abdominal aneurysm. He underwent transfusion with 6 units of packed red cells, Blood
tests were performed two hours postoperatively: haemoglobin 10.1 g/dl WBC 12.3 x
109/L , platelets 40 x109 /L, APTT 36 seconds, INR 2.2, fibrinogen 1.5 g/L, fibrin
degradation products 25 mg/mi. What will these results be consistent with?

A Disseminated intravascular coagulation B Inactivation of Antithrombin III C increased


level of vitamin K dependent D Plasminogen inactivation

E Protein C deficiency

103. A 46-year-old woman is seen in the pre-admission .clinic. She had a myocardial
infarction two years ago. On examination she is in atrial fibrillation and a recent. .
echocardiogram show that she had a dilated left ventricle. She is on warfarin and her
INR. (International Normalized Ratio) is 6.1 Warfarin inhibits which of the

A Antithrombin B. Kallikrein C. Plasminogen D. Prothrombin E. von Willebrand factor

104. A 29 year oId pregnant woman is having uncontrolled labor pains. She has an
epidural for pain relief. Which of the following is true?

A. An epidural puncture should only be performed below the L1 level to avoid damage to
the spinal cord B. An epidural puncture should only be performed below the L4 level to
avoid the spinal cord C.The epidural space (also known as the extradural space) contains
numerous nerve roots and a venous plexus

D. The liganientum flavum is rarely pierced during a low epidural procedure E. The
supraspinous and interspinous ligaments are rarely pierced during the procedure .

105. A 28-year-old motorcyclist presents to the Emergency Department after being


involved in a road, traffic accident. He has a closed isolated injury to his right leg.
Radiographs show a segmental fracture of the tibia and the fibula. Intramedullary
nailing of the tibia is undertaken. Six hours following the operation he complains of
severe calf pain which does not respond to analgesics. Movement of the toes severely
increases the pain. What is the most likely cause of this pain?
A. Deep vein thrombosis B. Deep-infection C. Embolic episode D . Raised intra-
compartmental pressure E. Movement of the unstabilized fibula

106. A 65-year-old woman with metastatic breast cancer is admitted to hospital


confused and acutely unwell, with nausea and vomiting. Her vital signs are within
normal limits. What abnormality is most likely to contribute to her clinical condition?

A . Hypercalcaemia B. Hyperkalaemia C. Hypocalcaemia D. Hypokalemia

E Hyponatraemia

107. during recovery from varicose veins surgery 35 years old woman complains of
weakness of dorsiflexion of the ankle Physical examination reveals absent sensation
over the dorsum of the foot. Which of the following nerves is most likely injured? A .
Common peroneal nerve B. Deep peroneal nerve C. Saphenous nerve D. Superficial
peroneal nerve E. Sural nerve 108. A 78-year-old woman, with chronic obstructive
pulmonary disease (COPD), becomes confused after a right hemicolectomy. She is oh
30% oxygen and her blood gas analysis shows:

From what is she most likely to be suffering?

A. Severe asthma attack B. Emphysema C. Pulmonary embolus D. Type 1 respiratory


failure E. Type 2 respiratory failure.

109. A 19-year-old man fell down a flight of stairs. On presentation to the Emergency
Department his Glasgow coma scale (GCS) was 14/15 with no focal neurological signs.
An hour later he vomited and his GCS deteriorated to 10. He developed a fixed and
dilated left pupil-. Which of the following accounts for the pupillary signs?

A. Compression of oculomotor nerve parasympathetic fibres B. Compression of


sympathetic fibres to the left iris C. Compression of the afferent fibres in the left optic
nerve D. Compression of the Edinger Westphal nucleus

E. Compression of the optic chiasma

110. A 32-year-oid mountain biker presents to the outpatient clinic. He fell awkwardly,
with excessive lateral flexion of his neck to the left side. He describes some paresthesia
over his right shoulder and lateral arm. Which appears to have been improving over the
past hour. What is the most likely injury suggested by this history?

A. Axonotmesis of the C5 nerve root B. Axonotmesis of the T1 nerve root C. Neuropraxia of


the C5 nerve root D. Neuropraxia of the T1 nerve root E. Neurotmesis of the C5 nerve foot
Po2 pCO2

Finding 6.2 kPa 8.5 kPA

11.9-
4.7-6.0
Normal 13.3

111. A 62-year-old woman presents with a firm irregular mass in the upper outer
quadrant of the right breast This appears malignant on mammography, and fine
needle aspiration cytology is reported as C5 which is the first lymph nodes to which the
tumour is most likely to metastasize?

A. Initial node. B. Primary node - C. Secondary node D. Sentinel node-. E. Virchow’s


nodes 112. A 50-year-old woman presents with a history of faecal incontinence over
the past, few years. She had a prolonged and difficult first stage of labour 20 years
previously. Physical examination reveals a relatively lax anal sphincter. Which nerve is
likely to have been damaged in labour?

A. Autonomic! nerves to the rectum . B. Genitofemoral nerve C. Lumbosacral trunk D.


Obturator nerve

E. Pudendal nerve

113. A 26-year-old man presents to the Emergency Department with extensive


bleeding from his arm, after sustaining a glass injury.-On examination- there is n 7 cm
transverse laceration across tire anterior aspect of his elbow. On exploring the cubital
fossa, you would expect the brachial artery to be?

A. anterior to the median nerve B. lateral to the biceps tendon. C. lateral to the median
nerve D. medial to the median nerve E. superficial to the bicipital aponeurosis

114. A 53-year-old man complains of 'crushing' chest pain at rest He has chronic
hypertension and is a heavy smoker. His pulse rate is 138 beats/minute and blood
pressure 140/90 mmHg. What is the principal cause underlying his reduced coronary
artery blood flow?

A. Decreased cardiac muscle wall tension B. Decreased coronary oxygen extraction C.


Decreased diastolic interval D. Decreased systolic interval

E. Decreased vagal tone


115. A 58-year-old woman, a known patient with thoracic right-sided meningioma,
presents with features suggestive of a Brown-Sequard syndrome. The clinical, findings
will include which of the following?

A Left sided weakness, left sided proprioception and vibration loss, right sided loss of pin
prick sensation B Right sided weakness, right sided proprioception and vibration loss, left
sided loss of pin prick sensation

C Right sided weakness, right sided proprioception and vibration loss, right sided loss of
pin prick sensation D. Right sided weakness, left sided proprioception and vibration loss,
right sided loss of pin prick sensation

E. left sided weakness, right sided proprioception and vibration loss, left sided loss of pin
prick sensation. 116. 70-year-old woman presents with a fractured left humerus. X-rays
show an osteolytic lesion at the site of the fracture. Histological examination of a
sample taken at the time of internal fixation shows a metastatic carcinoma. Where is
the most likely primary site?

A Breast B Colon C. Endometrium D. Ovary E. Stomach

117. A 74-year-old. man presents with a pulsatile swelling in his abdomen. The
principal abnormality within the wall of arterial aneurysms is loss of?

A. adventitial collagen B. adventitial elastin C. intimal collagen D. intimal elastin

E medial elastin

118. A 24-year-old woman presents with a swelling in the thyroid gland. She also has
enlarged lymph nodes in the left anterior triangle of the neck. Which of the following is
the most likely diagnosis?

A. Anaplastic carcinoma B. Follicular adenoma C. Follicular carcinoma D. Medullary


carcinoma E. Papillary carcinoma

119. A 20-year«okl man with a severe head injury is being ventilated using positive
pressure. which of the following is a physiological consequence of this?

A. Decreased extra fluid volume B. Decreased cardiac preload C. Increase in intrathoracic


blood volume D. Increased cardiac preload E. Increased cardiac stroke volume

120. A 70-year-old man presents with a one-week history of epigastric pain and –
profuse vomiting The most likely consequence is ?

A. hypochloremic acidosis B. hypckalaeinic alkalosis C. raised serum chloride D. raised


urine sodium

E. reduced renin release

121. A 22-year-old men arrives in the Emergency Department with sudden


breathlessness due to a large pneumothorax. An immediate chest drain is inserted into
the fifth left intercostal space in the mid-axillary line, There is hemorrhage into the
drainage bottle. Which of the following structures is the most likely cause of this acute
hemorrhage ?

A. Intercostal artery m B. Left pericardiophrenic artery C. Linguia of the lung D. Right


ventricle of the heart E. Spleen

122. The right and left pulmonary arteries are derived from which of the following
embryological aortic arches?

A. Second aortic arch B. Third.aortic arch C. Fourth aortic arch D. Fifth aortic arch

E. Sixth aortic arch

123. In the fetal circulation, the majority of blood passing from the inferior vena cava
into tire right atrium subsequently passes next into which; one of the following?

A. Aorta B. Left atrium C. left ventricle D. pulmonary artery E. Right ventricle

124. A 36-year-oid butcher injures his left index finger with a knife, suffering a deep
laceration on the palmar aspect of the middle phalanx. On examination he is unable to
flex DIP, which tendon is likely to have been injured?

A. flexor carpi facialis B. flexor carpi ulnaris C. Flexor digitorum profundus D. Flexor
digitorum superficialis E. Flexor indicis

A 39-year-old woman with large, expanding ovarian mass that is removed surgically,
requiring a lateral pelvic wall clearance, of lymph nodes. Four days postoperatively she
complains of painful spasms in the groin area and remarks that she has an area of
numbness on the medial side of her thigh. The most likely explanation for these,
problems is that during surgery an injury was inflicted on the:

A. femoral nerve B. genitofemoral nerve C. lumbosacral trunk D. L3 ventral ramus E.


Obturator nerve

126. A 70-year-old patient is found to have a sodium of 125 mmoI/L on examination


There is no signs of hypovolemia. His plasma osmolality is 280 mOsm/L and his Urine
osmolality is 1000 mOsm/L. "Which of the following is-the best explanation of these
findings?

A. Central diabetes insipidus B. Cerebral salt wasting syndrome C Nephrogenic diabetes


insipidus . D. Syndrome of inappropriate anti-diuretic hormone (ADK) secretion. E. Water
deprivation

127. A .surgeon is carrying out an elective splenectomy for congenital spherocytosis.


Which structure will be divided in order to mobilise the spleen from the posterior
Abdominal wall?

A. Gastrosplenic ligament B. Lesser omentum C. Lienoreaal ligament D. phrenicocolic


ligament E. Short gastric vessels

128. A 45-year-old man has established post hepatic cirrhosis. At a follow-up


appointment, a palpable spleen four fingers breadths below the costal margin, is
noticed. full blood

counts have shown a persistent thrombocytopenia. Bone marrow examination has


shown megakaryocytic hyperplasia. 'What is the most likely cause of the
thrombocytopenia?

A. Ineffective production in bone marrow B. Platelet destruction in the bone marrow C


Platelet destruction in the liver D. Platelet destruction in the spleen E. Platelet storage in
the spleen

129 A 7 years old boy presents with recurrent attacks of pneumonia. Culture showed.
Psudomonas aeruginosa. His mother confirms that he had meconium ileus in the
neonatal period. Which of the following is the most likely finding in. this boy?

A. Increased sweat alanine transaminase B Increased sweat chloride C. increased sweat


creatinine D. Increased sweat potassium

E. Increased sweat urea

130. A 42 years old woman has a cholecystectomy and develops a self-limiting


postoperative wound infection. By what process would bacterial ingestion have been
enhanced?

A. Apoptosis B. Autophagy C Metaplasia D. Opsonisation E. Phagocytosis

131. After resection of a rectal tumour a patient experiences erectile dysfunction.


Which of the following nerves is most likely to have been damaged in surgery?

A. Genitofemoral nerve B, Lumbosacral, plexus C. Pelvic splanchnic nerves D. Perinea!


branch of S4 E. Pudendal nerve

132, A 15-year-old girl has been treated for tuberculosis over the previous 6
months She presents to Her doctor with fatigue, loss of Appetite and weight loss. On
examination. her blood pressure is 90/55 mmHg and she has increased pigmentation.
What is the next most appropriate investigations

A, Aldosterone level B Oral glucose tolerance test C. Random Cortisol level

D ACTH stimulation test E water deprivation test

133. A 73-year-old man with a history of irregular bowel movements presents with
dysuria, pneumaturia and an Escherichia ccli urinary tract Infection. CT scans show a
mass involving the sigmoid colon and the bladder. What is the commonest cause of
this presentation? A. Adenocarcinoma of the sigmoid colon

B Colonic diverticular disease C Crohn's disease D Transitional cell carcinoma of the


bladder E Ulcerative colitis

134. A 30-year-old man is assaulted with a hammer and sustains a depressed fracture
at the Vertex, of the skull Profuse venous bleeding is noted at the fracture site. Which
vascular structure is most likely to have been perforated?

A. Cavernous sinus B. Inferior sagittal sinus C. Sigmoid sinus D. Superior petrosal sinus E.
Superior sagittal sinus

135. A 57-year-old heart transplant recipient is keen-to join the cardiac, rehabilitation
programme Which of the following factors is most likely to increase cardiac output in
this patient during moderate exercise?

A. Decreased negative intrathoracic pressure B. Decreased venous tone C Decreased


ventricular compliance D Increased atrial filling

E. Increased intrapericardial Pressure

136. An 18-year-old woman fell 9 meters while rock climbing. She had a renal
transplant two years ago. She is brought to the Emergency Department for
resuscitation and appropriate analgesia given. When correctly prescribed, witch of
following analgesic drugs would be most .likely to adversely affect her renal function in
this situation?

A. Diclofenac B. Dihyclrocodeine C. Morphine D. paractamol E. Tramadol

137. A 77-year-old man is admitted with acute urinary retention secondary to benign
prostatic hyperplasia. A urinary catheter is inserted and ot has free flow of
concentrated urine. Which of the following best accounts for the action of the

counter current multiplier mechanism that leads to the concentration of urine in the
Loop of Henle? A. Active transport of solute out of thin section ascending limb B.
Impermeability to water in thick ascending limb

C. Permeability to solute in descending limb D. Permeability to solute in thick ascending


limb E. Permeability to water of thin section ascending limb;

138- A 52-year-old man is found to have multiple myeloma, What skull X-ray
appearances would be characteristic?

A. Diffuse thickening of the calvarium B. Hair on end* appearance C. Multiple fractures D.


Multiple osteolytic lesions

E. Multiple osteosclerotic lesions

139. A 56-year-old man undergoes resection of his ileum secondary to ischemic bowel
Which of the following effects is most likely to be seen in this patient postoperatively?

A. Decreased bile salt synthesis by the liver B. Decreased concentration of bile salts in
colonic content? C. Decreased mean corpuscular volume D. Decreased reabsorption of
vitamin B12 E. Decreased stool frequency

140. A previously healthy 45 years old woman undergoes emergency splenectomy


following traumatic rupture of the spleen. A left subclavian central venous pressure
(CVP) line to guide fluid therapy is inserted. At this time she becomes distressed and
acutely dyspneic, with a blood pressure of 75/40 mmHg. There are decreased breath
sounds on the left. Her temperature is 35.5°C. What is the most likely cause?

A. Acute blood Loss B. Anaphylaxis C. Myocardial infarction D. Sputum retention E.


Tension pneumothorax

141. During the second (proliferative) phase of wound healing the predominant cells in
the wound site are fibroblasts. This cell is of mesenchymal origin -and produces the
matrix and collagen needed; to strengthen the scar. Cross linkage of collagen requires
hydroxy-proline and hydroxy-lysine residues, which requires a specific vitamin to be
available in sufficient quantities. Deficiency of which vitamin results in collagen that is
unstable?

A. Vitamin B2 (riboflaVine) B. Vitamin B6 (pyridoxine) C. Vitamin C (ascorbic acid) D.


Vitamin D (cholecalciferol) E. Vitamin E (tocopherol)

142. A 19- year-old man fell down a flight of stairs. On presentation to the Emergency
Department his Glasgow Coma Score (GCS) was 14/15 with no local neurological signs.
An hour later he Vomited and his GCS deteriorated to 10. He developed a fixed and
dilated left pupil. Which of the following is likely to seen on the scan?

A. Left-sided acute subdural haematoma B. Left-sided extradural haematoma C. Left-


sided midbrain haematoma D. Right-sided acute subdural haematoma E. Right-sided
extradural haematoma

143. A 70-year-old woman weighing 5Q kg undergoes open cholecystectomy.


Postoperative pain relief is administered by intravenous morphine infusion (morphine
1 mg; 5 minute lock out).-Twenty-four hours postoperatively she is in an agitated,
confused state. Her temperature is 37.1°C-.pulse 90 beats/minute 110/80 mmHg. Her
urine output is 35 ml/hour. Arterial blood gases are

Which of the following is the most likely explanation to account for false findings?

A. Hypovolaemia B. Opiate induced hypoventilation C. Pain induced hyperventilation D.


Pulmonary embolus E. Silent myocardial infarction

144. A 70-year-old woman from a nursing home is brought to' the Emergency
Department with abdominal pain and vomiting. On examination, she is dehydrated
and her abdomen is distended. There is a 3 cm x 4 cm swelling in the right groin which
is nontender, and there is no cough impulse. At operation, a femoral hernia is found.
Which one of the following lies immediately lateral to the hernial neck?

A. Femoral artery B. Femoral nerve C. Femoral vein D. Inguinal ligament E. Pectineal


ligament

PH pO2 Bicarbonat
pCo2 Base Excess
e

Finding 7.27 9.6 kPa


10.0 kPA 29 mmol/L -6

Normal 7.35-7.45 11.9-13.3


4.7-6.0 22-26 -2 to +2

145. A 64-year-old woman is-reviewed in the Emergency Department with an


acute right groin swelling, below and lateral to the pubic tubercle. Which of the
following forms the medial boundary of the ring through which this hernia is
protruding?

A. Conjoint tendon B. Inferior epigastric artery C. Inguinal ligament D. Lacunar


ligament E. Pectineal ligament

146, A. 27-year-old has had pancolitis for the last five years. Which one of these
complications is he least likely to develop?

A. Arthritis B. Cholangitis C. iritis D. Polyarteritis E. Toxic megacolon

147. An 82-year-oid woman had a hemiarthroplasty of the hip six hours ago! She is an
insulin-dependent diabetic and has been on long-term non-steroidal analgesics ,In the
last three hours she passed,, via her urinary catheter, a total of 20 ml of urine. Her
pulse rate is 120 beats/minute and regular, her blood, pressure 100/60 mmHg and
capillary refill is 3 seconds. Blood urea and electrolyte levels are:

The most likely cause of this is:

A. Acute Tubular necrosis B. blocked urinary catheter C. diabetic nephropathy D. non-


steroidal: anti-inflammatory drugs E. Reduced renal perfusion

148. Witch single respiratory test parameter (FEVJ Forced expiratory volume in one
second; FVC - Forced vital capacity) is the best prediction for morbidity and mortality
after surgery?

A. Confronted flow volume loop B. FEV1 < 1L C. FEV1/FVC <70%. . D. Maximum


inspiratory Volume < 2 L E. Reduced residual volume

Potassium
Sodium Creatinine Urea

Finding 135 mmol/L 4.0 mmol/L 140 umol/L 12.0 mmol/L

137-144 35-110 3.2-7.5


Normal 3.5-4.9

149. A 58-year-old man, admitted for inguinal hernia surgery, is found to have an
enlarged heart shadow on his chest radiograph. Which one of the/following
statements is true of the right ventricle?

A. Has a muscular wall that is normally thicker than that of the Left ventricle B. Forms the
apex of the heart C. Forms the right border of the heart D. Is directly related to the
diaphragm
E. Gives rise to the aorta

150. A 67-year-oid man is admitted to hospital for investigation of at irregular heart


beat. The 'conducting system' of the heart includes which one of the following
anatomical structures?

A. Atrioventricular node B. Chordae tendineae C. Fibrous skeleton of the heart D.


Interatrial septum E. Phrenic nerve

151 A 28-year-old man is found in the sea by the emergency services. he is alert but
confused. His core temperature is below 33°C. ECG changes that may occur include
which one of the following?

A. Atrial flutter B. inverted T wave at the junction of the QRS & T wave C. Narrowed QRS
complex D. Prolonged PR interval E. Shortened QT interval

152. A 75-year-old woman presents to the outpatients clinic with a three-months


history of painless ulcer on the sole of her left foot. On examination she is obese" with
a 1.5 cm deep ulcer over the first metatarsal head. Her foot is warm and pedal
pulsesare palpable. She has a hemoglobin of 13 g/dL, normal urea and electrolyte level.
What is the most likely cause of her ulcer?

A. Atherosclerosis B. Diabetic neuropathy C. Embolism D. venous hypertension E.


Vasculitis

153. A 24-year-old man is assaulted and sustains a deep 2 cm long laceration to the
lateral aspect of his lower leg. Physical examination show's that the laceration is
anterior to the lateral malleolus. Which of the following structures is most at risk
of being divided?

A. Peroneus brevis B. Peroneus longus C. Saphenous nerve D. Superficial peroneal nerve


E. Sural nerve

154. A 48-year-old woman presented to the Emergency Department with persistent


diarrhoea. On examination, she was apyrexial and had an irregularly irregular pulse: of
140 beats/minute. The most likely cause of the abnormal cardiac rhythm is:

A. hypernatraemia B. hypokalaemia C. hypovolemia D. myocardial ischemia E. sepsis

155. A 45 years old woman is investigated after two episodes of renal colic. A plain
abdominal X-ray reveals small opacities in both kidneys. No abnormalities are found
on physical examination. Biochemical investigations show:
Which, of the following is the most likely cause?

A. Primary hyperparathyroidism B. Primary hypoparathyroidism C.


Pseudohyperparathyroidism D. Secondary hyperparathyroidism E. Tertiary
hyperparathyroidism

156. A 35-year-cld man develops septicaemia following surgery for perforated acute
appendicitis. He is hypotensive. Arterial blood-gas analysis reveals

Which of the following is the most likely explanation to these findings

Findings Normal

Adjusted ( corrected) Calcium


2.85 mmol/L 2.15-2.55 mmol/L

Phosphate
0.70 mmol/L 0.6-1.25

Alkaline phosphatase 195 IU/L 20-120

Creatinine 72 umol/L 35-110

PTH 12.7 pmol/L 0.95-5.7

Bicarbonat
pO2 pCo2
PH e

Finding 7.25 7.2 kPa 7.5 kPA 17 mmol/L

7.35-7.45 11.9-13.3 4.7-6.0 22-26


Normal

A. Compensated metabolic acidosis B. Compensated respiratory acidosis C. Mixed


metabolic and respiratory acidosis D. Uncompensated metabolic acidosis E.
Uncompensated respiratory acidosis

157. A 75-year-old man is assessed for surgery. he has smoked 25 cigarettes a day for -
the past 45 years. He is short of breath on minimal exertion. Chest: auscultations
reveals a widespread wheeze. A chest X-ray shows hyperinfilated lungs. An arterial
blood gas sample is likely to reveal which of the following changes?

Bicarbonat
pCO2
e

A High High

B High Low

C Low High

D Low Low

Norma
E Normal
l

158. An 18-year-old woman present with recurrent tonsillitis and spontaneous skin
bruising. On examination there is no lymphadenopathy or splenomegaly. What is the
most likely diagnosis? A. Acute myelocytic leukemia B. Agranulocytosis C. Aplastic
anemia D. Hodgkin's disease E. Human immune deficiency virus infection

159. A 62-year old woman presents with a firm "irregular mass in-the upper outer
quadrant of the right breast. This is shown to be malignant on mammography and fine
needle aspiration-cytology. She is treated with wide local excision and axillary
clearance. Which of the following histopathological findings would imply a better
prognosis?

A. Absence of HER2 amplification B. Bloom and Richardson Grade III C. More than three
positive axillary nodes D. Presence of lymphovascular invasion E. Presence of estrogen
receptors

160. A.58 year-old man is due to have surgery for Dupuytrens contracture to release
the ring finger of the light hand. 'Which one of the following regarding the anatomy of
the ring finger is correct? A. Has a cutaneous innervation from the radial nerve

B. Has no cutaneous innervation from the median nerve C. Has no cutaneous innervation
from the ulnar nerve D. Has the flexor digitorum profundus inserting into the middle
phalanx E. Is abducted by a palmar interosseous muscle

161. A 75-year-old man presents with deep vein thrombosis. Computerized


Tomography confirms the presence of a thrombus' in His common iliac vein extending
into the inferior vena cava The common iliac veins join to form the inferior vena cava
at which vertebral level?

A. L3 B. L4 C. L5 D. S1 E. S2

162. A 35-year-old motorcyclist is involved in a road traffic accident and fractures his
temporal bone, completely transecting the facial nerve just distal to the geniculate
ganglion. Which function of this nerve remains?

A. Lacrimation B. Salivation from the submandibular gland C. Sensation to the anterior


tongue D. Smile E. Taste

163. A 30-year-okl motorcyclist is brought to the Emergency Department after a road


traffic accident. He has an open fracture of his right femur, On arrival he-is tachypnoeic
and confused, with cold and clammy skin. Which of the following physiological
changes is most likely to be seen?

A. Decreased reabsorption of water from, the collecting tubules B. Decreased serum


bicarbonate level C. Increased pH of blood D. increased secretion of sodium in the urine

E. Increased synthesis of glycogen in the liver

164. A 50-year-old man on the waiting list for a hip replacement attends the
preoperative clinic. He has a body mass index (SMI) of 41 and a risk of perioperative
hypoxia. Reduction of which of the following volumes or capacities is this risk most
likely to be due to?

A. Expiratory reserve, volume B. Functional residual capacity C. Inspiratory reserve volume


D. Total lung volume

E. Vital capacity

165. A 4-year-old boy presents to the Emergency Department with a two-day history
of Headache, vomiting and drowsiness. A. CT scan reveals dilatation of both his lateral
ventricles and his third cerebral ventricle: His fourth ventricle was of normal size. It is

suspected that he has an obstruction to his cerebrospinal fluid flow At which of the
following sites is the obstruction most likely to be?

A. Cerebral aqueduct of Sylvius B. Infundibular recess C. Interventricular foramen (of


Monro) D. Lateral foramen of fourth ventricle (foramen of Luschka) E. Median foramen of
fourth ventricle (foramen of Magedie)

166. A 74-ysar-old man, who has undergone emergency major abdominal surgery
two days previously, is noted to be confused. He has been on furosemide for mild
heart failure;. The plasma sodium is 122 mmol/L. Inspection of the fluid chart shows
that he has been written up for four-hourly intravenous 5% glucose infusions. What is
the most likely cause for the hyponatremia?

A. An ACTH (Adrenocorticotrophic hormone) response to surgery B. Excessive intravenous


fluid administration C. Osmotic effect of hyperglycemia induced by glucose infusions D.
Syndrome of inappropriate antidiuretic hormone

E. Use of loop diuretic in the long term

167. During arch aortography, the vertebral artery would be seen to; a rise directly
from which of the following?

A. Arch of the aorta B. Brachiocephalic artery C. Common carotid artery D. Internal


carotid-artery E. Subclavian artery

168. A 60-year-old non-smoker presents with a three-month history: of loss of weight,


malaise and breathlessness. On examination, he has left supraclavicular lymph node
enlargement! Chest radiograph reveals multiple bilateral opacities. 'What will biopsy of
left supraclavicular lymph nodes most probably show?

A. Chronic inflammatory cells B Granuloma C. Langerhan's type giant cells D. Multiple


abnormal mitotic figures in cells E. Reed Stenberg cells

169. A 12-year-old child was admitted to the Emergency Department having inhaled a
peanut. Where will the peanut most likely to have lodged?

A. Left lower lobe bronchus B. left upper lobe bronchus C. Right lower lobe bronchus D.
Right middle lobe bronchus E. Right upper lobe bronchus

171. A 39 years old multiparous woman who smokes 10 cigarettes per day presents
with 2cm x 2 cm tender lump in her right breast. There is a brown discharge from the
nipple. Mammography is unhelpful. And ultrasound shows some dilated lactation
ducts What is the most likely pathological diagnosis?

A. Breast abscess B. Fat. Necrosis C. Fibrocystic disease D. Phylloides tumor E. Plasma


ceil mastitis

172. A-56 year oId man presents with headache and vomiting with deterioration over
the last two weeks. On examination he has papilledema. A CT scan shows a space
occupying lesion in the right parietal lobe. He develops a sudden homonymous
hemianopia. Which past of the visual system to be affected?

A. Lens B. Optic Chiasm C. Optic radiation D. Retina E. Second cranial nerve


173. Atrial depolarization is shown by which ECG change?

A. P B. Q C. R D. S E. T 174. A 65 years old man with known ischemic heart disease


and atrial fibrillation presents with a paralysis of his right lower limb. Which arterial
territory is most likely to be involved?

A. Left anterior cerebral B. Left middle cerebral C. right anterior cerebral D. Right middle
cerebral E. Right posterior cerebral

175. A 91 year old man becomes hypotensive two days after THR. He is found to be in
atrial fibrillation with a pulse of 140 beats/minute. After treatment with digoxin his AF
slows and he has a pulse rate of 90 BPM. Which form of the venous wave form is
absent?

A. A wave B. C wave C. V wave D. A decent E. Y decent

176. A 56-year-old man with chronic emphysema is on the high dependency unit
(HDU), ten days after anterior resection. He has developed acute shortness of breath'
and hypotension): Which of the following landmarks would be the most appropriate to
use to obtain a femoral arterial blood gas sample?

A. A point midway between the anterior superior iliac spine and the pubic tubercle B. A
point midway between the anterior superior iliac-spine and the' pubic symphysis C. A
point midway between the greater trochanter and the pubic symphysis D. a point 1 cm
inferior and 4 cm lateral to the pubic tubercle E. A point 2 cm lateral to the midway point
between the anterior superior iliac spine and pubic symphysis

177 . A 4S-year-old woman with insulin-dependent diabetes attends to the ER with


abdominal pain and confusion. She has been treated for three days with antibiotics for
chest infection. The creatinine is 400umol/L and her blood sugar 42 mmol/L. The most
likely severe electrolyte abnormality is:

A. Hypercalcemia B. Hyperkalemia C. Hypermagnesemia D. Hypocalcemia E.


Hypomagnemsia

End of paper 2
[11:23 PM, 4/14/2018] +249 11 331 6200: What was that thumb abduction with hand
on table answer
✅EPL
what about injury in axilla and numbness in lat forarm:
✅C6
source of blood during lumbar puncture ?
✅lumber venous plx
type of femur fracture in the blunt injury
✅Transver fx
child who was hit from side ?
✅greenstick
Athelts Womnan with Fx?
✅Stress Fx
YOUNG child with twested leg injury (Tibial twisted)
✅Spiral tibial Fx
Child playing football and fall on outstretched hand?
✅Supracondylar FX
Pt e prolonged catheter this frank hematuria and w loss :
✅(SQuamous cell carcinoma of the bladder
special notes:
(SHISTSOMA AND INDWELLING CATHETER LEAD TO SCC OF BLADDER Dr.Ali
Mansour)

Cyst in breast aspirated cytology free what next?


discharge
There was melonoma in situ removed.?
✅Discharge
pt e breast cancer and metastatic disease come e hypotonia:
✅hyper ca
After vericose vein surgery numbness dorsum of foot which nerve damaged:
✅saphenos Nerve
Ttt of post op phelbitis:
✅bandiage
Pt e antithrombin 3 Def and dvt :
✅Warfarin life long
Rx for recurrent pulm embolism :
✅V.C filter
Surface anatomy of IJV ???
✅From lobe of the ear to sternal edge of clavicle
Ulnar nerve , artery relation upper or lower forarm:
✅ulnar to ulnar art.
There was this question about GCS. Patient localizes pain(5) , open eye to pain(2) and
sounds(1) whats the GCS ?
✅8 in answer choices
Peronial hematoma and lower abd hematoma
✅Membranous urethra injur
with leg flexion at 30degree and moves lateral after trauma I chose
✅medial colateral injury
Then a man hit by car when viewed from above left tibia lie posterior
✅pcl
Sciatic nerve injury what other movement not possible:
✅extention of hip
6 yrs Child e sudden hip pain Increased abduction& Int. Rotation & limitation of active /
passive movement of hip joint in all directionX-rays(this finding just for revision, By
Dr.Ali Mansour😉)
✔ flattened femoral head. In untreated case femoral head will fragment
✔ Sub-chondral crecent shaped radiolucent line
✔ Calcification lateral to epiphysis
✔ Metaphyseal rarefaction
✔ Lateral extrusion of head
✔ Abnormal physeal growth
✔ Gage’s sign:
✅perthis
Oh there was a question u operating parathyroid and find nodule in thyroid what to do?
✅thyroidectomy
33 years old diagnosed as papillary 0.5cm nodule without *LN spreading
✅Hemithyroidectomy+ regular follow up using thyroglobulin
Pt with MEN 2 have anterior Neck swelling with significant increase in *calcitonin and
diagnosed as medullary thyroid ca CA what to do?
✅Total thyroidectomy+ regular calcitonin follow up
Anexity adrenaline released from endings or gland ??
✅gland
Injury to the dorsal root ganglia???
(sensoey,motor,sympathetic ,parasympath) ✅All
The one with cervical subcutaneous emphysema this pt have Esophageal rupture :
✅(Ix of choice:water soluble contrast esphagography(too look for extension of this
perforation)
*Ejection systolic murmur with pulmonary edema …
[11:23 PM, 4/14/2018] +249 11 331 6200: Then there was this thiazide where did that
act :
✅DCT (inhibt NA/CL co-trasport)
Amiloride act on(you might found spironlactone also)?
✅Aldestrone antagonist(ENaCs)
Loop diuretics act inhibt?
✅1Na,1Kand 2Cl Co-transporters
pt had pancreatitis thin hyperpara .. inv what next ??
✅Pituitary adenoma(this patient of MEN 1 Pitutary,parathyroid,pancrease)
Factor which doesnt cause colon ca.
✅diverticulitis
Golfer with first metatarsophalangeal joint xray showed decreased space and alsub
chondral cyst:
✅Psudo-gout( you can find calcium pyrophosphate dehydrate deposite in joint mainly
hand)
Man used opoid then constipation painful fresh bleeding
✅Anal fissure due to constipation secondary to opiod use
Then was about prostate lymphatics drainage
✅internal iliac and sacral LN
Lymphatic drainage of upper anal canal(endoderm)?
✅Internal iliac LN
lymphatic drainage of lower anal canal(ectoderm)?
✅Superficial inguinal Ln
lymphatic drainage MIDDLE third vagina?
✅internal iliac LN
lymphatic drainage lower third vagina?
✅superficial inguinal LN
Lymphatic drainage of upper third vagina?
✅External iliac LN
Lymphatic drainage of fallopian tube?
✅Para-Aortic LN
Lymphatic drainage of fundus of the uterus?
✅Para-Aortic LN
Lymphatic drainage of upper part of body of the uterus+fundus+fallopian tube?
✅(Para-Aortic LN
Lymphatic drainage of overies?
✅Pra-aortic LN

lymphatic drainage of Urinary Bladder?


✅external iliac LN
Lymphatic draing of corn of the uterus?
✅Superficial inguinal LN

Testis lymphatics:
✅para aortic
Scrotum:
✅Medial medial horzontal Superficial inguinal LN.

Pt e vien graft 16 month ago then stenosis pathophysiology:


✅ intimal hyperplasia
Nerve to upper sctotum and penis ??
✅ileoinginal
Which artery anurysn compressed renal vein:
✅SMA
ant relation to rt adrenal gland:
✅IVC
Q about pt e head injury on IPPV ,at the 5 day techypnea and hypoxia ????
✅ tracheotomy
Other q pt start weaning in icu???
✅CPAP
Patient underwent operation for IBD complicated by infection...now developed
diarrhea? ✅pseudomembranous colitis
Esophageal perfiration type of feeding :
✅TPN
Staged ediphagectomy:
✅PEG
Perforated appendix organism?
✅e.coli
Bilious vomting ,scaphoid abdomen ?
✅dudenal atresia
9 month What was in child with rt upper quadrant mass empty rectum and vomiting ?
✅ Valvolis
There was a child with family history of cystic fibrosis?
✅meconium ileus
And then there was child vomiting with Distended abdomen And meconium plug was
renoved after 3 days by suppository
✅hirschprung
Test for Addison des? ?
✅short synacthen
Spleen rupture without trauma?
✅Ebv
study comparing fluid requirements? after burn in male n females...most patient had
small values and some larger values...what test to apply?
✅paired t TEST
Case pt have 5 unite blood then bleeding profile: decr pltls ,normal ptt, incr INR , inc
fibroplastin norm fibrinogen products:
✅ she said i chose DIC bcoz I don’t know the others (She is Right hahaha AliMansour).
intrinsic pathway?
✅ factor 9
(Ascending Lymphangitis) creeping redness around vessels caused after
thorne/gardening?
✅Strep pyogenes
Root of transmesion of mrca in some surgical ward 5 pt infected?
(Contact
Renal mass on USS?
✅MRI
15x9 cm lesion deep fascia,what to do:
✅excision bx
That liver accumulation prussian blue:
✅iron( in another scenario inx is transfertin)
Pt e history of FAP and colectomy done 30 yr ago now he came e jundice pale stool and
vomiting:
✅deud.Ca
Melanoma suspected 1cm;
✅Excise with 2cm margin(study berslwo )
Patient 6 days post-op burn surgery with hiccups n abdominal distension...soft
abdomen?
(Acute gastric dilatation
Forhead mass pt itch cause bleeding follwed by incr in size during 1 yr:?
✅Bcc
special notes:
BCC is the commonest type of cutaneous cancer and affects approximately 30% of all
Caucasian Americans in their lifetime with the majority (80%) being on the head and
neck.
They usually present as raised pearly nodules with telangiectasia rather than the less
common “Rodent ulcer”. Although frequent, they are the least concerning of the skin
cancers, rarely showing metastatic spread and usually exert their influence by local
destruction if left untreated. Although the vast majority (2/3) are found on sun exposed
areas, 1/3 do not, emphasising the genetic link. Surgery with an appropriate margin is
the treatment of choice although chemotherapy, immunotherapy and radiation therapy
are also used.

Which is the first immunologic cell in rosethorn and infl:


✅Neutophil
what does warming reduce levels of?
✅TSH
Mediastinal mass with keratin and flat cells :
✅SCC of the Lung
Hyponatraemia in man post op with glucose fluids? Was it stress response or excess
lack of sodium containing fluids?
✅extra fluid admition
Heat loss intra op by
✅Radiation
Marathon runner sudden death;
✅SAH

What the hell do we give to pregnant lady with megaloblastic macrocytic:


✅B12 first and then you can give folate
What about gastric acid secretion reduced by:
✅somatostatin (read about gastrin please)
What drug is a precursor to noradrenaline?
✅dopamine
Duct papilloma proplimatuc :
✅duct excion
Post mastoidectomy loss of tast:
✅chora tympany
Then facial nerve
second arch
diaphramatic hernia
✅Pleuroperitoneal memberane
hypothermia
✅J waves
Dorsal surface opening of meatus
✅Exostrophy bladder
Nerves at risk Of injury surgical approaches:

medial ankle ;
✅Saphenous
distal femur:
✅tibial
During hip arthroplasty surgeon note Artery run in superior border of pectenius ms ?
✅Sup gluteal artery
clamped unbiblical cord ,which will not carry oxigintated blood to the hrt?
✅Ductus venosis,hepatic artery,umbilical artery
Burn question with oedema?
✅hypoalbumin
For onion peel:
✅Ewing sacoma
Smoking and microscopic?
✅transitional cell ca
egyptian farmer heamaturia?
✅squamous cell ca
Keloid scar:
✅ntralitional steroid
Median nerve and brachial artery in in arm to cubital fossa?
✅Lat ant medial
Parathyroid adenoma?
✅sestimibi
extraintenstinal features of crohns except:
✅polyarteritis
There was a question about low potassium, high creatinine, scarred small kidneys?
✅(Chronic renal failure
Cause of AAA ??
✅arthrosclerosis
Efferent arteiole vasoconsrricted?
✅Angiotensin 2
Pcwp 20?
✅pul oedema
Pcwp 11?
✅ARDS
High riding truma ?
✅Membranous urethra
Pineal hematoma+ excessive collection?
✅bladder
Cushing?
✅Hypok
Metabolic acidosis?
✅Hyperk
Wide qrs.?
✅Hyperk
Sickle cell disease +bone pain+pneumonia agent:
✅streptococcus pneumonae
Pneumonia in ventilated?
✅Psedomonas
Guys there was a question about achllis tendon ruptute what was the test ?
✅Simmons squeeze test
What do they clean plastic syringes with?
✅Radiation
Patient with decrease sperm account?
✅fragile-X-syndrom

What was that thumb abduction with hand on table answer


✅EPL

what about injury in axilla and numbness in lat forarm:


✅C6

source of blood during lumbar puncture ?


✅lumber venous plx

type of femur fracture in the blunt injury


✅Transver fx

child who was hit from side ?


✅greenstick

Athelts Womnan with Fx?


✅Stress Fx

YOUNG child with twested leg injury (Tibial twisted)


✅Spiral tibial Fx
Child playing football and fall on outstretched hand?
✅Supracondylar FX

Pt e prolonged catheter this frank hematuria and w loss :


✅(SQuamous cell carcinoma of the bladder

special notes:
(SHISTSOMA AND INDWELLING CATHETER LEAD TO SCC OF BLADDER Dr.Ali
Mansour)

Cyst in breast aspirated cytology free what next?


discharge

There was melonoma in situ removed.?


✅Discharge

pt e breast cancer and metastatic disease come e hypotonia:


✅hyper ca

After vericose vein surgery numbness dorsum of foot which nerve damaged:
✅saphenos Nerve

Ttt of post op phelbitis:


✅bandiage

Pt e antithrombin 3 Def and dvt :


✅Warfarin life long

Rx for recurrent pulm embolism :


✅V.C filter

Surface anatomy of IJV ???


✅From lobe of the ear to sternal edge of clavicle

Ulnar nerve , artery relation upper or lower forarm:


✅ulnar to ulnar art.

There was this question about GCS. Patient localizes pain(5) , open eye to pain(2) and
sounds(1) whats the GCS ?
✅8 in answer choices

Peronial hematoma and lower abd hematoma


✅Membranous urethra injur
with leg flexion at 30degree and moves lateral after trauma I chose
✅medial colateral injury

Then a man hit by car when viewed from above left tibia lie posterior
✅pcl

Sciatic nerve injury what other movement not possible:


✅extention of hip

6 yrs Child e sudden hip pain Increased abduction& Int. Rotation & limitation of active /
passive movement of hip joint in all directionX-rays(this finding just for revision, By
Dr.Ali Mansour😉)
✔ flattened femoral head. In untreated case femoral head will fragment
✔ Sub-chondral crecent shaped radiolucent line
✔ Calcification lateral to epiphysis
✔ Metaphyseal rarefaction
✔ Lateral extrusion of head
✔ Abnormal physeal growth
✔ Gage’s sign:
✅perthis

Oh there was a question u operating parathyroid and find nodule in thyroid what to do?
✅thyroidectomy

33 years old diagnosed as papillary 0.5cm nodule without *LN spreading


✅Hemithyroidectomy+ regular follow up using thyroglobulin

Pt with MEN 2 have anterior Neck swelling with significant increase in *calcitonin and
diagnosed as medullary thyroid ca CA what to do?
✅Total thyroidectomy+ regular calcitonin follow up

Anexity adrenaline released from endings or gland ??


✅gland

Injury to the dorsal root ganglia???


(sensoey,motor,sympathetic ,parasympath)
✅All

The one with cervical subcutaneous emphysema this pt have Esophageal rupture :
✅(Ix of choice:water soluble contrast esphagography(too look for extension of this
perforation)

Ejection systolic murmur with pulmonary edema ;


✅aortic calcification
Furacmide act on which part of loop of henle?
✅Ascending limb of of loop of henle

Then there was this thiazide where did that act :


✅DCT (inhibt NA/CL co-trasport)

Amiloride act on(you might found spironlactone also)?


✅Aldestrone antagonist(ENaCs)

Loop diuretics act inhibt?


✅1Na,1Kand 2Cl Co-transporters

pt had pancreatitis thin hyperpara .. inv what next ??


✅Pituitary adenoma(this patient of MEN 1 Pitutary,parathyroid,pancrease)

Factor which doesnt cause colon ca.


✅diverticulitis

Golfer with first metatarsophalangeal joint xray showed decreased space and alsub
chondral cyst:
✅Psudo-gout( you can find calcium pyrophosphate dehydrate deposite in joint mainly
hand)
Man used opoid then constipation painful fresh bleeding
✅Anal fissure due to constipation secondary to opiod use

Then was about prostate lymphatics drainage


✅internal iliac and sacral LN

Lymphatic drainage of upper anal canal(endoderm)?


✅Internal iliac LN

lymphatic drainage of lower anal canal(ectoderm)?


✅Superficial inguinal Ln

lymphatic drainage MIDDLE third vagina?


✅internal iliac LN

lymphatic drainage lower third vagina?


✅superficial inguinal LN

Lymphatic drainage of upper third vagina?


✅External iliac LN

Lymphatic drainage of fallopian tube?


✅Para-Aortic LN

Lymphatic drainage of fundus of the uterus?


✅Para-Aortic LN

Lymphatic drainage of upper part of body of the uterus+fundus+fallopian tube?


✅(Para-Aortic LN

Lymphatic drainage of overies?


✅Pra-aortic LN

lymphatic drainage of Urinary Bladder?


✅external iliac LN

Lymphatic draing of corn of the uterus?


✅Superficial inguinal LN

Testis lymphatics:
✅para aortic

Scrotum:
✅Medial medial horzontal Superficial inguinal LN.

Pt e vien graft 16 month ago then stenosis pathophysiology:


✅ intimal hyperplasia

Nerve to upper sctotum and penis ??


✅ileoinginal

Which artery anurysn compressed renal vein:


✅SMA

ant relation to rt adrenal gland:


✅IVC

Q about pt e head injury on IPPV ,at the 5 day techypnea and hypoxia ????
✅ tracheotomy

Other q pt start weaning in icu???


✅CPAP

Patient underwent operation for IBD complicated by infection...now developed


diarrhea? ✅pseudomembranous colitis

Esophageal perfiration type of feeding :


✅TPN

Staged ediphagectomy:
✅PEG

Perforated appendix organism?


✅e.coli

Bilious vomting ,scaphoid abdomen ?


✅dudenal atresia

9 month What was in child with rt upper quadrant mass empty rectum and vomiting ?
✅ Valvolis

There was a child with family history of cystic fibrosis?


✅meconium ileus

And then there was child vomiting with Distended abdomen And meconium plug was
renoved after 3 days by suppository
✅hirschprung

Test for Addison des? ?


✅short synacthen

Spleen rupture without trauma?


✅Ebv

study comparing fluid requirements? after burn in male n females...most patient had
small values and some larger values...what test to apply?
✅paired t TEST

Case pt have 5 unite blood then bleeding profile: decr pltls ,normal ptt, incr INR , inc
fibroplastin norm fibrinogen products:
✅ she said i chose DIC bcoz I don’t know the others (She is Right hahaha AliMansour).

intrinsic pathway?
✅ factor 9

(Ascending Lymphangitis) creeping redness around vessels caused after


thorne/gardening?
✅Strep pyogenes

Root of transmesion of mrca in some surgical ward 5 pt infected?


(Contact
Renal mass on USS?
✅MRI

15x9 cm lesion deep fascia,what to do:


✅excision bx

That liver accumulation prussian blue:


✅iron( in another scenario inx is transfertin)

Pt e history of FAP and colectomy done 30 yr ago now he came e jundice pale stool and
vomiting:
✅deud.Ca

Melanoma suspected 1cm;


✅Excise with 2cm margin(study berslwo )

Patient 6 days post-op burn surgery with hiccups n abdominal distension...soft


abdomen?
(Acute gastric dilatation

Forhead mass pt itch cause bleeding follwed by incr in size during 1 yr:?
✅Bcc

special notes:
BCC is the commonest type of cutaneous cancer and affects approximately 30% of all
Caucasian Americans in their lifetime with the majority (80%) being on the head and
neck.
They usually present as raised pearly nodules with telangiectasia rather than the less
common “Rodent ulcer”. Although frequent, they are the least concerning of the skin
cancers, rarely showing metastatic spread and usually exert their influence by local
destruction if left untreated. Although the vast majority (2/3) are found on sun exposed
areas, 1/3 do not, emphasising the genetic link. Surgery with an appropriate margin is
the treatment of choice although chemotherapy, immunotherapy and radiation therapy
are also used.

Which is the first immunologic cell in rosethorn and infl:


✅Neutophil

what does warming reduce levels of?


✅TSH

Mediastinal mass with keratin and flat cells :


✅SCC of the Lung
Hyponatraemia in man post op with glucose fluids? Was it stress response or excess
lack of sodium containing fluids?
✅extra fluid admition

Heat loss intra op by


✅Radiation

Marathon runner sudden death;


✅SAH

What the hell do we give to pregnant lady with megaloblastic macrocytic:


✅B12 first and then you can give folate

What about gastric acid secretion reduced by:


✅somatostatin (read about gastrin please)

What drug is a precursor to noradrenaline?


✅dopamine

Duct papilloma proplimatuc :


✅duct excion

Post mastoidectomy loss of tast:


✅chora tympany

Then facial nerve


second arch

diaphramatic hernia
✅Pleuroperitoneal memberane

hypothermia
✅J waves

Dorsal surface opening of meatus


✅Exostrophy bladder

Nerves at risk Of injury surgical approaches:


medial ankle ;
✅Saphenous

distal femur:
✅tibial
During hip arthroplasty surgeon note Artery run in superior border of pectenius ms ?
✅Sup gluteal artery
clamped unbiblical cord ,which will not carry oxigintated blood to the hrt?
✅Ductus venosis,hepatic artery,umbilical artery

Burn question with oedema?


✅hypoalbumin

For onion peel:


✅Ewing sacoma

Smoking and microscopic?


✅transitional cell ca

egyptian farmer heamaturia?


✅squamous cell ca

Keloid scar:
✅ntralitional steroid

Median nerve and brachial artery in in arm to cubital fossa?


✅Lat ant medial

Parathyroid adenoma?
✅sestimibi

extraintenstinal features of crohns except:


✅polyarteritis

There was a question about low potassium, high creatinine, scarred small kidneys?
✅(Chronic renal failure

Cause of AAA ??
✅arthrosclerosis

Efferent arteiole vasoconsrricted?


✅Angiotensin 2

Pcwp 20?
✅pul oedema

Pcwp 11?
✅ARDS

High riding truma ?


✅Membranous urethra

Pineal hematoma+ excessive collection?


✅bladder

Cushing?
✅Hypok

Metabolic acidosis?
✅Hyperk

Wide qrs.?
✅Hyperk

Sickle cell disease +bone pain+pneumonia agent:


✅streptococcus pneumonae

Pneumonia in ventilated?
✅Psedomonas

Guys there was a question about achllis tendon ruptute what was the test ?
✅Simmons squeeze test

What do they clean plastic syringes with?


✅Radiation

Patient with decrease sperm account?


✅fragile-X-syndrom

September 2017, MRCS part A

EMQ Rib # management


1- young athlete with single rib #, CXR free after 3 hrs
Analgesia and discharge
2- old age with COPD with single rib # , CXR free after half an hour Admit and analgesia
3- middle age with multiple rib # in severe pain, CXR pneumothorax Rx-chest drain

EMQ Femoral #
4- 1-80 years SELF DEPENDANT female with # neck of femur ? Hemiarthoplasty (there
wasn’t THR in options )
5- 2- known case of ca breast with subtrochantric #
Internal fixation
6- 3-40years old with displaced subcapital #
Internal fixation

EMQ about pulse oximetry/spo2/level of co2 7- 1-shows saturation of hemoglobin


Pulse oximetry
8- 2-affected by low perfusion

Pulse oximetry

9. 9- 12 y old boy with osteoid forming tumor in lower femur?


osteosarcoma??ewing??osteoblastoma??benign

bone cyst

10. 10- RTA pt intubated and mechanical ventilation while examination and turning
patient on his back he had

sudden dyspnea??
tube displacement??neurogenic shock??pneumothorax??sudden increase in ICP

11. 11- Young age return from Bangladesh With bloody diarrhea and lower abdominal
pain
Mucosa severely distorted on sigmoidoscopy?? Entameba, UC, Camphylobacter
12. 12- Man ate barbecued chicken then had bloody diarrhea?? Campylobacter, C
difiicile
13. 13- Inf epigastric vein drains into- external iliac vein
14. 14- Black toe with aortic aneurysm? Embolus
15. 15- Black toe with DM and positive pedal pulse? Small vessels disease
16. 16- Patient Post sigmoid colectomy then discovered he has antithrombin 3 def
??give warfarin for 3

months??give warfarin for life??give heparin ??

17. 17- Acute intestinal ischemia by emboli

EMQ vitamins
18- -Alcohol patient with peripheral neuropathy

B1
19- Post gastrectomy anemia

B12 20- Dry eye and skin

1
A
21- Easy bruising and delayed wound healing

22- Stat q of paired t test


23- Stat q of man whitney test 24- Some audit q cant remember

Pregnant woman with DVT and family history of similar attack ?? anticardiolipin ab

25- A patient with RTA and multiple injuries.


Developed dyspnea .. X ray show edema bilaterally....Defect in ARDS...DIFFUSION DEFECT

26- What is the JVP in AF--- absent "a" wave AF


27- Signs and symptoms of Hyperthyroidism with eye sign? Graves
28- Long history given about hypothyroidism with TSH low/T3 low? 2ry hypothyroidism

29- Patient with Acromegaly how to investigate


PET scan, AP skull , Lateral skull, Cephalometry

CT (there wasn’t mri in options)

30- Clear q about CPP calculation


BP 110/70 and ICP 18 was given

31- Transpyloric plane at what level- L1


32- Origin of inf parathyroid?? 3rd pharyngeal pouch or 3rd pharyngeal cleft or 3rd
pharyngeal arch

32- Question about planter flexion of ankle...asking root value

33- History of trauma to wrist...lunate dislocation


Asking nerve damage. Median n lesion ?

34- Q about young boy with trauma wrist.


Xray normal and discharged. Came back after one week with severe pain at anatomical

snuff box....Scaphoid fracture clear q

35- Q of defect in distribution of both ulnar and median nerve ?? lower trunk brac
plexus(C8,T1)

median trunk of plexus

or median cord
36- What is true about Right coronary artery
1- orign after the aortic valve
2- run in interventicular groove
3- give rise to anterior descending 4-arise from ant ascending aorta

37- Where does great cardiac v run? Ant septum

38- Patient for elective hernia repair under general anaesthesia, Temperatures 35.8 how to
proceed? Change the room temperature forcefully and proceed UGA
Give IV fluid worm UGA
Proceed without changes

Switch to local?? (my answer) post pone until at least 36??

39- SA node supplied by vagus

40- Lacrimal gland....Pterygopalatine ganglion

41- Patient with injury to finger while gardening.. Came with swelling of pulp space?? I
choose Streptococcus pyogenes

Other options: Klebsiella, staph aureus 42- thorn prick something injury??strept pyogenes

43- Prosthetic valve bacteria ? S. epidermidis


44- Anasthesia in hemodynamics unstable patient

Ketamine

45- Patient with RTA, Developed bradycardia and hypertension why? Intracerebral hge

46- RTA with breast trauma and swelling lesion Fat necrosis
Knee injury
47- 1-twisted knee while playing

Next day with swelling and positive tap

Unable to fully extend? Medial meniscus

48- 2- Trauma to knee from side.. with valgus injury ....mcl clear q
49- Recurrent chest infection with meconum ilius in infancy? Which test will be
abnormal..Sweet chloride

50- Gene in familial polyposis coli ? APC

51- Breast micro calcification in xray/mamo no mass do what? Mri ?fnac

52- Hypotension in RTA patient ?


Give fluid and cross match?? ct??

53- Best prognosis in Melanoma?? Breslow thickness 54- Best prognosis in BCC??

completeness of excision
55- Post splenectomy defect seen in blood?? Increase platelets

56- Post phlebitic syndrome treatment?


I choose compression bandage

57- Neonate with rectovaginal fistula defect in ? Cloaca

58- Unilateral cleft lip defect?


Defect in fusion of max with lat nasal

process(my answer)

max with median nasal process

mandibular defect

59- There is another Qs about RTA and lucid interval Extradural hge

60- Why dilated pupil in extra dural left side ?


Injury to left parasympathetic oculomotor

Patient can't smile symmetrical after trauma to facial nerve during surgery?
Buccal or marginal mandibular br of facial nerve(acc. To medscape it’s the second one)

61- Patient can not look downward and inward? Trochlear Nerve
62- Patient with papillary carcinoma with no evidence of LN on ULS ? total thyroidectomy
alone

total thyroidectomy with lymph node excision

One more Q about thyroid nodule, palpable thyroid mass on swallowing FNAC negative
Further

plan of action

63- Q Post renal transplant 18 months with generalised LAP? EBV

Another one
64- Generalized LAP for 2 years I think?

Lymphoma or reactive lymphadenopathy

65- Q Woman with renal ca and bone metastasis pain?? Radio(did it) NSAID Paracetamol

66- Rectal bleeding in child? Juvenile polyp

67- 7 yr old boy ...pain in knee and hip limping Q about perthe's disease

68- 11 yr old obese boy with hip pain

SUFE

69- A nurse developed contact allergy to gloves type 4 or 1 hypersensitivity ??

70- child with his father working in garden develop rhinorrhea and iching IGE

EMQ oral lesions


71- One with precancerous red velvety lesion in the mucosa under the tongue lesion ?

Erythroplakia

72- One with blue transparent lesion under tongue ? Runula

73- Patchy lesion and Bleeding from tongue Smoker and alcoholic? Scc

74- Patient with history of pancreatitis Presented with yellow stool difficult to flush bulky
What deficiency ? I chose lipase

75- Patient with ca rectum


From which artery blood supply derived from? I chose IMA
76- Perforated gastric ulcer at the anterior wall of greater curvature? Left gastroepiploic
artery or Rt gastroepiploic
Splenic artery, Gastroduodenal

77- Patient received warfarin and developed bleeding Deficiency of ?? 1972 factors or vit k

78- Q on scrotal swelling after long standing and has small testes Disappeared on supine
position?

Varicocele

79- Q First metabolic response of adrenaline? Glycolysis (not tachycardia like fawzia)

80- Q patient fasting blood glucose > 7 mmol and post prandial > 11? Dx? Diabetes mellitus

81- Patient developed recurrent attack of dizziness and fainting attack? Hypogycemia. What
is elevated in blood

Insulin level
C- peptide leve
Increase insulin + Increase C peptide

82. 82- Which cell depends on Coricycle for its metabolism? Hepatocyte
83. 83- Q Beta1 found in? Ventricle
84. 84- Q Patient return from Bangladesh develop painless jaundice? Hepatitis A
85. 85- Patient with UC presented with abdominal pain... max in LIF and High grade
fever X ray shows Distended transverse colon?

Toxic megacolon
86. 86- Localised Annular ca sigmoid colon, no secondaries Type of surgery? left
colectomy and anastomosis

Hartman (did it and mostly wrong)

87. 87- Q Patient with difficult swallowing after long time of reflex Cancer oesophagus
88. 88- Acidosis in diarrhea, due to defect in Lose of HCO3
89. 89- Patient smoker with continuous cough Wash out shows Squamous cell what
changes happened? Irreversible or reversible changes??

90. 90- Q What goes with absolute acidosis ?something


Cant remember options but mostly was urinary ph
91. 91- Patient with multiple # Develop sign of PE Diagnosis? CT angio
92. 92- Cardiac output at rest in athletic heart?
5 to 6 lit per min or 8 to 10 lit per min
93. 93- Temporal tenderness high ESR? Giant Cell Arteritis
94. 94- Non caseating necrosis found in Tb

Sarcoidosis Crohn disease Celiac

95. 95- Scenario of DIC


96. 96- 2 q about shock types can't remember

97-
98- ECG in Repolarization of ventricle-T wave

by exclusion

99- External urethra sphincter Supply by what root S2,3,4

100. 100- Incontinence after delivery Normal sphincter anatomy ?

Pudendal nerve

101. 101- A male patient was examined PR. Posterolateral structure in PR exam 4
cm from anal verge ? Puborectalis, seminal vesicles, prostate
102. 102- A doctor examine female patient...PV examination --ant structure
palpable is base of urinary bladder, fundus of uterus,
103. 103- Site of ectopic testes ? root of penis

EMQ Incisions and structure divided

104. 104- One pfennestine? Rectus sheath


105. 105- Upper midline ? linea alba
106. 106- Inguinal surgery ? ext oblique aponeurosis
107. 107- Lumbar puncture surface marking...Level of ilac crest ? L4
108. 108- Q of male vs female anatomy of pelvic bones Difference in male and
female pelvis
109. 109- Renal artery posterior to renal vein
110. 110- 2nd cervical vertebra feature ?? bifid spinous process ( No odontoid in
options )
111. 111- Q about hydrocephalus and Dilated lateral and third ventricle?
Obstruction of Silvius aqueduct
112. 112- Q about structures at popliteal fossa..Closest to knee joint ? popliteal art
113. 113- Q tumour marker of melanoma ?
I choose none of above
114. 114- Q marker of breast cancer??
CA125?? CEA?? NON OFABOVE??

2q similar

115. 115- 15 patient with loss of lumbar lordosis and leg numbness
116. 116- 20s weight lifter with loss of groin sensation and leg numbness

Central prolapse Post lat prolapse Spondylolisthesis

Stupid EMQ resuscitate

117. 117- One Post appendicectomy and low urine out put
118. 118- One with RTA severe hypotension
119. 119- One Severe asthmatic and in distress

Answers
500 ml hartman solution and observe in half hour 2 litre hartman rapidly
CPP ventilation

120- Old age with history of cervical OA


Presented with numbness in upper limb how to investigate? MRI

121. 121- Complex supracondylar Investigation # ? ct


122. 122- Q Pregnant lady with suspected perforated DU? Asking for investigation?
CT, USG

Chest x ray

123. 123- Patient post cholecystectomy 3 day with abdominal pain tachycardia
investigation? CT, USG, Coeliac angio
124. 124- Same scenario With PR 100
BP 90/60 Cool peripheries Afebrile
Cause?
Intra abdominal bleed

125. 125- Q chronic COPD ABG shows ? High co2 high hco3
126. 126- Patient post op. Diabetic and on insulin. No oral intake. On IV fluids NS
and Hartmann developed

confusion and convulsions. Reason?

Hyponatremia, Hypoglycemia

127. 127- Q of peri op management of diabetic pt? did glucose monitoring can't
remember others
128. 128- Q rapidly increases thyroid mass with fixation to other tissue? Anaplastic
129. 129- Medial border of femoral hernia
130. 130- Q relationship of brachial artery and median nerve at cubital fossa
131. 131- Multiple myeloma on Xray finding
132. 132- Q Patient with general fatigue and weight loss
With positive Benz jones protein in urine and passing

pink color particles Amyloidosis

133. 133- Obstuctive jaundice with pale stool and yellow urine what is Increased
urine ? conjugated bilirubin

Urobilinogen

Q initiation of abduction of shoulder? Supraspinatous

134. 134- Q renin act on what? Angiotensinogen


135. 135- Q cerebropontine angle tumour involve which cranial nerves? CN 7 8
136. 136- Q patient presented with hyper pigmentation ? addisson crisis
137. 137- Nerve supply to angle of mandible? Great auricular or auriculotemporal
138. 138- Fluid maintenance postop ?

2litrNS and1.5 GW5%

139. 139- There is Q about patient with history of carcinoid Diagnosis? Ct or barium
or urine VMA
140. 140- Increase ADH lead to?
Increase fluid in ECF or ICF or trasnscelluar
141. 141- One young age male with neck pain and uvula shifted to another side?
quinsy
142. 142- Child with high grade fever and neck spasm to one side with noisy
breathing?
Retropharyngeal abscess or Ludwig angina

143. 143- 39 years female with family history of ca breast, asymptomatic. Screening
by? Mammo or us??
144. 144- Q Multiple small gall stone.. due to Increased what? Unconj bil
145. 145- Q Patient with Pancolitis Least likely to happen? Which extra intestinal
manifestation is least likely to

occur in patient with UC?

Polyarteritis nodosa

146. 146- Q Patient can not do opposition of thumb


Recurrent br of median nerve (specific for opponens) Deep branch of Ulnar
147. 147- there is a case RTA with shock Sign of acidosis? In the choices decrease
bicarbonate
148. 148- The process which Enhance bacterial ingestion? Opsonization

149. 149- Q Multiple LN What to find on biopsy? Multiple abnormal mitotic figures
150. 150- History of appendicectomy
With yellow granules from RIF wound.. easy
151. 151- Case post op received large amount of blood transfusion developed SOB?
Fluid overload or hemolytic
152. 152- Q post TURP Hypotension and confusion.. electrolyte abnormality
153. 153- Most common sign of hyponatremia?

Confusion

154. 154- late complications of blood transfusion jaundice


155. 155- Bleeding while passing chest tube
156. 156- Source of Bleeding from? intercostal A
157. 157- Hx of pelvic surgery With Post op erectile desfunction? Pelvic splanchnic
n
158. 158- PE what ECG finding direct
159. 159- Patient with long standing celiac disease What type of lymphoma
developed?

B cell or T cell ( t acc to medscape)

160. 160- Which one of the following increase gastric acid secretion?

Histamine

161. 161- Prosthetic metallic valve behind the sternum at the level 3rd ICS? Aortic,
Mitral or pulm
162. 162- During tracheostomy Structure in front 2nd, 3rd and 4th tracheal
cartilage Thyroid isthmus
163. 163- Q Epithelial lining of the urinary bladder? Transitional cell
164. 164- Patient post op sigmodectomy and stoma formation Which of the
following is part of recovery enhance

program?

I did that 1ry care option cant remeb

165. 165- Weird q of frozen section? did Slide before staining


166. 166- Hyponatremia post op asking the cause? Administration of Na free fluid

167- Q lose of sensation in dorsum of foot and dorsflexion? Common peroneal

10

168. 168- Q Recurrent UTI and pneumoturia? Diverticulosis


169. 169- Q Hypoxia and hypo ventilation?
insensitivity of central chemoreceptors to PCO2
170. 170- One of the border of anatomical snuff box EPL
171. 171- Q Which of the following increase water and bicarbonate secretion from
pancreases? Secretin
172. 172- Lose of foot eversion The sensory area affected is?
173. 173- Q Why to choice inulin to measure GFR
174. 174- Q Which one used to measure GFR? Inulin or creatinine
175. 175- Protein C deficiency Defect in what ? Factor 5 or 8
176. 176- Noradrenaline act on which receptor ? alpha1
177. 177- Scenario of parotid swelling and facial nerve involvement? Adenoid cystic
Ca
178. 178- Numbness of cheek and upper frontal teeth after trauma to face ?
Infraorbital N
179. 179- Which of the following is correct regarding meneitriere disease? I choase
gastric ca

DVT prophylaxis 2 q emq cant remeb

180- 181-

182-

Patient undergoing sigmoid colectomy??


Patient Dm undergoing above knee amputation??

Patient with duck B classification 5 y survival ? 70%


Patient with fall from height or burn With highly urea and creatinine With dark urine
contain myoglobin?
Rhabdomyolysis

Emq from hell of lymphaadenopathy Options


Anal cancer
Melanoma

Cervical cancer Papillary thyroid cancer

Qs

11

183. 183- Patient return from Australia to UK has unilateral inguinal LN


184. 184- Patient has underwear bleeding and bilateral inguinal LN
185. 185- Patient has no neck mass and post triangle LN

EMQ 2 Q
186- Patient with # neck femur before one year
Treated by internal fixation Now presented with pain and stiffness of hip?AVN

187- Patient with metatarsal dislocation # , wire fixation After ten years with pain and
stiffness ? post traumatic OA

188. 188- Patient with left side leg weakness What circulation? Right anterior
cerebral artery
189. 189- Defect in pleuroperitoneal membrane result in ? bochadelek hernia
190. 190- From sheet fawzia, Absent femoral pulse and radial pulse intact
Inturrupted aortic arch syndrome
191. 191- Hypothermia first response ? Vasocontriction
Or something about hypothalamic responce
192. 192- Blood supply to cecal tumor? ILEOCECAL
193. 193- Procesus vaginalis derived from? Peritoneum
194. 194- Patient with miosis and hyperhydrosis with blinking defect? I chose
stellate ganglia

Emq 2 q

195. 195- Patient planning for elective inguinal surgery Hb 9 How to correct?

I chose give haeme oral iron preparation

196. 196- Cant remember other one


197. 197- Patient with renal failure Serum k 7 What is the first action ? Calcium
gluconate
198. 198- About anatomy of spleen
Which of the following is true? I chose contain macrophage

199. 199- Patient works in vinyl chloride factory Develop liver mass ? Angiosarcoma
200. 200- Patient with HIV and thyroid mass what are the action? Repeated
investigations

Post pone surgery


Inform all the staff in the theater

Continue with prophylaxis(did it)

12

201- Patient with anal ca Associated with? HPV

202. 202- Patient on oral antibiotics developed dysphagia? Candida


203. 203- patient with multiple radio opaque stone Analysis confirm ca oxalate
stone What is the cause? I chose hyperparathyroidism

Familial hyperoxaluria (right answer)

204. 204- Q Patient on warfarin with INR 5

He is about to undergo surgery for strangulated femoral hernia IV vitamin K (my answer)
FFP
Aspirin
prothrombin complex

205- Patient post gastrectomy developed B12 deficiency Blood picture will show??
Megaloblasts

206. 206- Correct about right ventricle ? Forms the apex of heart contact with
diaphragm
207. 207- Q Most of pain stimulus transmitted by?
208. 208- Temp tract ? spinothalamic too

Spinothalamic tract

209. 209- Q Pt with trauma presented with Marked clawing of hand injury to?

Ulnar nerve injury at wrist Ulnar nerve at elbow

210. 210- Sensation at first web space by ?? Deep personal nerve


211. 211- Surgery to short saphenous vein, post op loss of sensation over lateral
foot ? sural n
212. 212- Abduction of hip impaired Injury to?
Sup gluteal nerve

213. 213- Q Patient with torticollis

Injury to spinal accessory nerve

214. 214- Patient with numbness of hand and tingling sensation with dripping
objects Awake him at night

Diagnosis as CTS? Treatment Decompression

215. 215- Posterior leg compartment syndrome diagnosis by Hallux flexion, Hallux
extension ( my answer) Ankle

planter flex

216- Q Injury to ring finger distal phalanx>fdp

13

217- Patient with ca breast treated by wide local and radiotherapy Now presented with
back pain How to investigate ? Ct, Xray MRI(answer) PET

Emq Management of head trauma

218. 218- Patient with GCS 11 and depressed skull #


219. 219- Patient old age with fall and repeated vomiting
220. 220- Patient alcoholic and fall

Not knowing if lose Q or vomiting There's small wound on eye brow

My answers Neurosurg refer Immediate ct, observe 3rd

221. 221- Patient with thyroid nodules


FNA shows benign cell? Hemithyroidectomy
222. 222- Q Erythematous tender swelling of the breast How to differentiate it
from inflammatory

carcinomatous?

No response to Ab(my answer) nipple inversion


Associated with mass

223. 223- Q 55 year old with ca pancreas Obstructed jaundice With single liver
metastasis ? Treament ERCP stent
224. 224- Q Patient with thyroid nodule and pulsatile forehead mass? Follicular Ca
225. 225- Q testicular torsion How to treat ? Fixation orchidopexy
226. 226- Q 25 yr old male ..appendicectomy done 1 yr ago for carcinoid of tip of
appendix. Presented with facial flush and diarrhea?

Investigation? Ct, small bowel enema, large bowel enema

Q After passing central line Patient collapses? Tension pneumo

227. 227- Herpes zoster on T8 T10 site?? Costal margin


228. 228- Disease resulted from failure of neural crest cells to migrate? HSP
229. 229- Alcoholic drunk patient with urgent desire to pass urine, Run and fall with
trauma from wall His urgency

disappeared. Next day presented to ER with lower abdominal pain and complaining of no
urine out put Bladder empty by exam

Just tender lower abdomen? UB RUPTURE

230. 230- Q Reason to give steroid to brain tumor before surgery? Dec peri tumour
edema (my anaswer) Dec tumour vuscularity

Dec tumour size

231. 231- Q Osteomyelitis in SCD? Salmonella

14

232. 232- Patient post op for anal fistula


Not healing With RIF pain ? CD
233. 233- Botulinum toxin act by? BLOCK ACH
234. 234- There's is Q About Old age with claustrophobia patient and metallic valve
With trauma and fall on

shoulder

How to investigate? Ct, Mri, Xray (MY ANSWER) PET sca, USG

235- There was that q of finger swelling History of trauma not tender not attached to skin
no foreign body inside? Abscess, Fb granuloma, Aneurysm digital a, Epidermoid(did it by
exclusion)

236- A man came with left loin pain, USG shows left hydronephrosis and a bladder mass..
Dx? TCC

237- A patient with painful shoulder. Painful arc with pain between 60 to 120 degree
shoulder movement. Tendon involved?
Supraspinatous
238- A question on Baroreceptors... which mechanism acts first in the body in case of
hypovolemia and hypotension.

239- A patient with problem of finger extension and mild wrist extension difficulty.. no
sensory loss... nerve injury? PIN
240- In cushing disease due to pituitary tumor abnormality seen in adrenal gland is....
I choose adrenal hyperplasia

241- A patient developed hypotension immediately after passing foleys catheter..... latex
allergy ( Fawzia sheet )
242- A patient with pharyngeal diverticulum. Asking the insertion site of diverticulum..
Through inferior constrictor muscles ( I chose it)

Between medial and lateral constrictor Above the inferior constrictor

243- A 8 yr child with swlling in neck. Biopsy shows pleomorphic malignant cells with
osteoid formation...Dx? Gaint cell tumor, enchondroma, lymphoma

244- A patient with tingling and numbness in the thumb and index finger... more at night.
Positive Tinel's sign. What is the most appropriate management?
Analgesia and rest
Division of flexor sheaths

Brachial plexus exploration

245- Prolonged QT with pancreatitis Asking the electrolyte abnormality? Hypocalcemia


Hypercalcemia

Hypokalaemia
Hypo or hypernatremia

15

246-A patient received blood transfusion. 30 min later he developed hypotension,


tachypnea and back pain.? Acute haemolytic reaction
Non haemolytic reaction

247- which one of the following blood transfusion reactions occur late? Fever, malaise,
jaundice, haemotylic reaction

248- 50 yr old diabetic male, admitted for THR. After surgery, he had anuria. Vitals signs
normal. No signs of dehydration.
Underactive bladder
Renal stone

Bladder rupture
249- 35 yr old male patient. Admitted 4 days ago for perianal abscess drainage. Discharged.
He presented to ER with diffuse painful gluteal skin irritation redness and fever?
Erysipela
Cellulitis

Necrotizing fascitis

250- A 45 yr old female patient admitted 4 days ago for elective LAP chole and discharged
without any problems. Patient came to ER with severe abdominal pain. Vitals: HR 96, BP
100/90. Temp: 36. RR 25, WBC 3.5 and Hb 10.
Dx?
Biliary trauma
Post biliary syndrome
Gram negative sepsis
Post op bleeding

251- similar question, same scenario Asking for investigation?


CT, USG, coeliac artery angiogram,

252- EMQ ear discharge


A patient with 40db hearing loss and painless discharge- cholestetoma

A child with ear discharge, pain relieved after ear discharge- ASOM

253- long scenario patient with post op diarrhoea due to antibiotic use. Dx as cl diffiile
diarrhoea. Asking for treatment?
Oral metro
IV metro

Oral vanco

254- a young patient presented with slow urine stream and increased frequency of urine.
Had fever , urethral discharge and dysuria 6 months before.....
Urethral stricture
UTI

January 2018 Recalls

1. Tumor marker for Testicular teratoma? a. AFP (correct)

2. Numbness of the lower lips, which nerve involved? a. Buccal

b. Mental
3. DM patient undergoing angiography. Which drug causes LACTIC ACIDOSIS? a. metformin
(correct)

4. Malignant melanoma. Which one is the good prognostic factor? a. Breslow’s skin
thickness (correct)

5. Statistics question asking what is proportion of test positive from all positive? a.
Sensitivity

b. Speficitcity
c. Number needed to treat d. Odds ratio

6. There has been ongoing audit regarding what is done to the patient. What type of audit?

1. Process audit
2. Outcome audit

EMQs on MRSA infection – 2 Qs

7. A patient is being admitted for scheduled elective surgery. He has been found to have
nasal MRSA positive. What to do?

i. Mupirocin ointment X 5 days (correct)

8. A patient is shifted to ward after laparotomy. Blood culture is positive for Gram +ve
cocci. He is asymptomatic. But another patient in the ward has MRSA infection.
What Rx would you give?
i. Oral vancomycin
ii. IV ticoplanin
iii. IV penicillin
9. Femoral Hernia Q – What is behind the neck of the hernia sac

Pectineal ligament (correct)

10. A clinical vignette describing a Peutz-Jeghers syndrome. What type of polyp is found? a.
Hamartomatous (correct)

11. Patient is found to have extreme hypothermia. Which of the following be best
management for rewarming the pt.?

1. Intraperitoneal fluid infusion (correct)


2. Rectal Warm water
3. Bladder warm water
4. Warm iv fluids

12. External spermatic fascia of the cord is derived from which structure? a. External
oblique aponeurosis
1

13. A Clinical Vignette describing a nodule of 3 cm size in the region of elbow in the
extensor surface. Biopsy showed necrobiotic granulomatous inflammation. Dx?

1. Rheumatoid nodule (answer)


2. Erythema nodosum

14.A clinical scenario of polyuria, polydipsia with 11.3 mmol/L of blood sugar. Dx?
1. Diabetes Mellitus (answer)
2. Diabetes Insipidus
15.During cholecystectomy, pt. develops extreme bradycardia. Past history of heart
transplant

present. Which drug to give?

a. Atropine
b. Glycopyrrolate c. Ephedrine

EMQ on local anesthesia – 3 Qs

16.Old lady with colles fracture. She has CKD and IHD.
17.Injury to nail
18.Forgot

Options

i. Biers block
ii. Ring block
iii. Median nerve block
iv. Radial Nerve block
v. Ulnar Nerve block
19.Nerve emerging from lateral border of psoas major cause numbness to lateral thigh

a. Meralgia parasthetica

20.Neonate with cyanotic heart disease. Dx? a. TGA


21.Wound healing one week? What process is undergoing? a. Angiogenesis
22.To estimate GFR, which substance is used? a. Inulin
23.A clinical vignette easily diagnosed as TB. Lymph node was excised. Which of the
following

cells are in the periphery of this lesion seen in the histopathology?

a. Macrophage

24. Type of amyloid in Renal failure a. AL


b. AA
c. β-amyloid
d. Transthyretin

25. Which of the following is the 1st branch of internal carotid as soon as it leaves the
cavernous sinus?

a. Ophthalmic Artery (correct)

26. Badly infected wound with crepitus. Dx? a. Clostridium perfringens

27.Rose pink rash around cellulitic wound which was also indurated. Which bacteria?
1. S. aureus
2. S. pyogenes
3. Anthrax
28.60% burn case with bilateral leg edema. Cause? a. Hypovolemia

b. DVT
c. Hypoalbuminemia

EMQ on wound cover – 4 Qs

29.Anterior tibial defect of 5 x 7 cm


30.Necrotic scar at the heel in an immobilized patient
31.1 cm wound in cheek
32.Burn wound which is crusted and dry in the anterior abdominal wall

Options

i. Surgical debridement and primary closure


ii. Free flap
iii. SSG
iv. Wet dressing to the wound
33.A clinical scenario of hypercalcemia. What is the first step in Rx? a. Rehydration
34.A child with audible bowel sound in left chest. What is the congenital defect?

a. Failure to closure of pleuroperitoneal membrane

35. A 3-day old neonate had abnormal meconium stain in the infra-umbilical part of the
diaper. Dx?

a. Patent vitello-intestinal duct

36.Which of the following serves as the axis of gut rotation during development? a.
SMA
37.A injury to orbit of eye due to direct hit by a squash ball. Now presents with
diplopia. Visual

acuity Normal. X-ray orbit anatomy is normal. Best next investigation?

a. MRI b. CT

c. X-ray skull

38. Case of pneumonia in a 3 years post renal transplant patient not responding to
antibiotics. Which organism responsible?

39. 40.

a. CMV
Morton’s neuroma cause or pathophysiology? nerve tissue with fibrosis

Patient removed from burnt house. In what scenario patient needs intubation? a. resp rate
of 35
b. pH 7.2
C. Sweating confusion anxious

d. Pco2 5.7

41.

42.What investigation is best for a patient thought to have labral tear of acetabulum.
MRI
43.True about FFP
a. its leucodepleted

b. can't use them after 24 hours out of fridge

44.Pt with serum potassium 6. What should be imidiate step?

A. iv fluid was the only logical answer

45. Which of the following requires immediate renal replacement? A. K+ 6.1 mmol/L

46. A clinical vignette that describes a woman with hypothyroidism and microcytic anemia
(MCV was low).Which of the following is the patient at highest risk of during perioperative
period?

A. Decreased cardiac contractility


47. Clinical vignette describing gross picture on UGIE pertinent to atrophic gastritis. Which
of the following antibodies wil be found?

i. Antibody to parietal cells

48. Clinical scenario describing a DRE findings of BPH. What pathology is underlying? a.
hyperplasia

b. Hypertrophy

Clinical Vignette describing tibia fracture who underwent internal fixation developing
repiratory distress .

a. Fat embolism

49. Renal transplant patient described and a table given showing High calcium/ low
phosphate/ high alkaline phosphate. What is the diagnosis?

a. Tertiary hyperparathyroidsm b.
c.

50. Post operative patient who is recently getting restless. Urine output chat is tabulated
hourly as 80 ml / 80ml/ 80 ml /0ml/ 0ml/ 0ml in last 5 hours . what is the cause?

a. Blocked Catheter b.

51. Over infusion 3.5 L of NS to shock patient ? which of the following is the potential result
? a. hyperchloremic acidosis

52. Fracture Shaft of Femur was reduced and implant placed. At the end of operation you
note that the lower leg seems pale with redused pulse. What is the best next step in
management ?

a. Reexplore the wound b. Embolic=sation


c. Angiography
d. Compartment release

53.Can't pass colonoscope beyond hepatic flexure. What investigation shall follow? a.
ct colonography

b. Barium enema

54.During splenectomy,short gastric arteries ligated. which structure at risk of direct


damage?
1. Greater curvatureof stomach
2. Tail of pancreas
3. Left kidney
55.Clinical Vignette describing locked knee joint in a young male with trauma

a. medial meniscus tear

b.

56. Similar scenario in old age non-trauma patient a. Loose body

57. Patient can't lift hand from table patient can't extend distal interphnageal of thumb a.
radial post interoseous

58. Which of the following muscles is responsible for quiet respiration a. Diaphragm

b. Intercostals c. Scalene

EMQ on Breast lesions investigation

59.Breast abscess
60.Bil implants with suspected mass and positive family history
61.Retroareolar lobulated mass
62.RUOQ 3 cm mass Options

i. FNAC ii. USG iii. MRI

iv. Core Biopsy

63. Bleeding per rectum in an Old Gentleman. Colonoscopy revealed its around Sigmoid
colon. Planned for angioembolisation. Which vessel should the interventionist enter to stop
bleed?

1. R colic
2. L colic
3. Middle Colic
4. Rectal middle artery

64.Loss sensation medial leg. Which nerve ?


65.Loss sensation 1st web foot. Which nerve ?

a. Deep peroneal

66. Action on which of the following receptor is responsible for the depressant action in an
old man who underwent laparotomy and is on Morphine, RR is 6/min?
a. Mu receptor b. Delta
c. Dopamine d. Serotonine

67. A young guy with testicular pain describing features of Epidydemoorchitis. Which
organism C

1. Chlamydia
2. E coli
3. Staph

68.After excision of submandibular deviation of tongue is noted. Nerve injury? a.


Hypoglossal
69.General sensation lateral tongue by which nerve? a. Lingual

EMQS on trauma scenarios

70.Complicated Flail segment scenario but no hemo/pneumothorax – cyanotic, old


man
71.Young boy with stable hemodynamics but had few ribs fractured
72.Young girl with decreased breath sounds

Options

1. Intubate and PPV


2. Bupivacaine block
3. Lignocaine block
4. Chest tube insertion

EMQs on Consent
73. The mentally retarded patient but her parents refuse surgery which seemed not much

urgent

74. Patient with appendicitis and also has down’s syndrome but intellect good that she
knows consequences and complications of the procedure Comatosed pt coming to ER ...
consent the relatives

75. One more but forgot Options

1. Get a court order


2. Consent parents
3. Consent the patient

76. Which of the following is not in immediate relation of 3rd part of duodenum
1. Right kidney
2. R ureter
3. Psoas

77. Which of the following makes the posterior wall of inguinal canal? a. Conjoint muscles
/tendons was only logical optio

78. Which of the structers listed is the reason that testis descends to scrotum?

1. Gubernuculum
2. Processus vaginalis
3. Tunica

EMQs on LN drainage

79.Scrotal ulcer
80.Cervical lesion
81.Anal canal lesion

Options

1. Deep inguinal
2. Lumbar paraaortic
3. Superficial inguinal
4. Internal iliac

82. From which structure listed does the External spermatic facia derive from ? a. External
oblique apneurosis

b.

83. Lateral side of ankle injured in an inversion trauma. Which ligaments?

1. deltoid
2. calcaneocuboidal and talo- calcaneal

c. d.

84.Cuboid bone has following joint relations in its distal part


1. Fourth and fifth metatarsal
2. Media cuneiform and navicular
3. 1st and 2nd metatarsal

d.
85.Prior to Episiotomy, which nerve is blocked?

a. internal pudendal nerve block

86. Relation of extensor indices to extensor digitorum in index finger ? a. Medial and lateral

b. Lateral c.
d.
e.

87.Old man with prostatic cancer, has bony mets to shaft of femur and asymptomatic.
1. Radiotherapy
2. Morphine
3. PCM
4. Further investigation
88.L chylothorax after thoracotomy and esophagectomy. Where in the course is the
Thoracic

duct injured?

1. Before it entered subclavian in neck


2. At pleural reflection in superior mediastinum
3. during its course from diaphragm before it reached superior mediastinum

89. One Question on SIADH. Which ca lungs

a. b. c. d.

Small cell ca Squamous cell

Adenocarcinoma

90. Coin ingested. And seen on xray somewhere a. ....Lodged at level of cricopharyngeal
level

91. Mechanism behind pale offensive stoolsin Obstructive jaundice a. decreased


enterohepatic circulation

92. Which of the following intervention will decrease raised ICPP with immediate afftect
but short duration?

a. Hyperventilation

93.Long scenario on Pancreatitis. Which of the following is the mediator of injury?


1. Trypsin
2. Lipase
3. Amylase

d.

94.A clinical vignette which you will understand esily that it’s a subclavian steal. What

investigation will you do?

a. Echo
b. Selective arteriography of aortic branches

95. A clinical vignette of Afib, widened mediastinum and leg going pale a. Aortic dissection

96. Four hours of testicular pain in a young boy after bicycle riding. Examination shows a
tender scrotum. No further description.What would you do?

a. Doppler
b. Explore immediately

97. Tender testis with black dot in upper pole – a. hydatid testicle morgagni answer

98.that typical question of Anaphylaxis but patient was not that sick
1. IM adrenaline
2. Anti-allergic medication
3. Oxygen
99.Which of the following changes occur after cholecystectomy?

a. decreased rate of bile flow after meals b.


c.

d. e.

100. Slowly growing painful lateral swelling in knee a. Menescial cyst

101. A man developed cellulitis in hand after bitten by some insect. At operation table it
was now found that he has a goiter, heart rate is high. What drug wil you give ?

a. Beta adrenoceptor antagonist

102. Ova cyst bloody diharrea

a. metronidazole b. tinidazole

c. Mebendazo d.
e.
103. Trauma scenario with tender pain in thigh with external rotation with no
distal pulses
1. femoral shaft fracture
2. posterior dislocation of hip
3. anterior dislocation
4. complex # neck of femur
104. A vignette describing Brocas Aphasia

a. Middle cerebral

105. Similar scenario with nystagmus a. PICA Syndrome

10

b. c.

EMQs on Nutrition

106. A young guy with Midgut volvulus underwent resection and has 30cm of gut
left
107. Redo operation for esophageal cancer
108. Right hemicolectomy
109. Something

Options

1. Can be fed orally


2. TPN
3. Feeding Jejunostomy placed at the time of operation

110. A confusing scenario of patient developing sudden urinary retention and lower limb
paresis. Which artery?

1. Anterior spinal
2. Artery of Adamchweick
3. Lumbar artery

d.

111. While doing a LP, whats the last structure to pierce ? a. Arachnoid mater

112. A case describing a horner syndrome. Other features are already given and you have
to choose one more .

1. Ptosis
2. Mydriasis
c.

113. Another similar scenario, that tells you that its horner. Which site may be affected? a.
Lateral horn of spinal cord

114. Pancoast tumor you need to find out T1 symptoms

EMQs on stomas

115. One young guy underwent Panproctocolectomy for UC


116. One old man had sigmoid perforation and has fecal peritonitis and is in shock

Options

1. Ileostomy
2. Hartmans procedure
3. Diverting ileostomy with descending –rectal anastomosis

11

d. Ileoanal anastomosis and diverting colostomy e. Caecostomy

117. Choose a good maintenance fluid regime for a fit young guy a. 2L 5%dex with
1L Na

b. 2L Na and 1L dex

118. Which is true regarding left coronary?


1. It runs behind pulmonary trunk
2. It runs in anterior interventicular groove
3. It comes out of aorta just below the valves
119. Newborne with dyspnea and the NG not passing into stomach a. TOF
120. Need to do a tracheostomy or something like that. Which of the listed
structures have the

least chance to get injured in this procedure?

1. Ascending pharyngeal artery


2. Rec. laryngeal nerve

121. Mechanism of action of heparin


1. V/ VIII/ X inhibition
2. X inhibition

c.

122. Urachus, the remnant will be seen as


1. median umbilical ligament
2. medial umbilical ligament
3. ligament teres

d.

123. Spinal cord contents outside - menigiocele meningeomyelocyle\


124. Abdomail anuerysm 4.7 cm mnx - went with serial ultrasound . Other option
explore serial ct
125. Another scenario I thought to be keratocanthoma

12

126. Some pearly lesion on air pinna I went with basal cell ca when I saw pearly
127. Dermatofibroma scenario with history of trauma
128. Bluish lesion of toe and inguinal lymphadenopathy? a. melanoma
129. Sebhorruc keratosis scenario cos I saw the word crusty or ulcerated
130. One scenario of cavernous sinus I did adducent nerve
131. ABG
132. ABG
133. Ranula described. Which gland does it originate from?

a. sublingual gland

134. Why is it important to excise thyroglossal cyst?


1. Deglutiton problem
2. infection
3. cancer
135. trauma to head and verterx is depressed. Which structure may be damaged ?

a. superior sagital sinus

136. A female with 2 week h/o fo stridor , dysphagia n hoarseness. O/E thyroid swelling n
lymph node +ve. FNA shows spindle cells etc

1. Anaplastic ca
2. MCT

c.

137. Brain Tumor with necrosis and invasion push to opposite side through corpus
callosum.
1. GBM
2. Astrocytoma
3. Medulloblastoma

d.
138. Young girl cervical lymph node. Diagnosed on Histopath to be MCT . what is
appropriate

management ?

1. total thyroidectomy with nodal clearance


2. total thyroidectomy only
3. hemithyroidectomy

d.

139. Esophagus hiatus how many cm from incisior a. 40

b. 48 c. 27 d. 22 e. 15

140. Some post esophagectomy patient having a stricture on anastomotic site

13

1. Resect and redo anastomosis


2. Baloon dilatation
3. Stenting

d.

141. A women with jaundice, imaging described what they mean to be primary biliary
cirrhosis. Ehich antibody you test positive?

a. antimitochindrial ab b.

142. Peritonitis and air under diaphragm described. Which Investigaion is best in detection?
a. Xray erect

b. CT

143. The typical question describing pain in legs on walking and gets better by walking
uphill and pushing trolley .

a. Spinal stenosis

144. That typical question of patient in Intermittent positive pressure ventilator setting .
what occurs?

a. decreased venous return

145. A man has continuous ooze from wound for 2 days after a femoropopliteal bypass. He
was on aspirin and clopilet till 48 hours before operation. Why do you think is the bleed?
a. Side effects of Aspirin and clopidogrel b. Anastomotic disruption at the proximal site c.
d.

146. Some fev1 1.8 L fvc 2.1 L what's the pathology? a. Restrictive disease

147. A young girl with dyspnea. fev 1 / fvc 36 then some treatment give and raises
to 78. DX? a. asthma
148. Pelvic trauma. Unable to pass urine. Where is the site of injury?
1. Membranous urethra
2. bladder rupture
149. that typical question on bladder mucosal change with Schistosoma ?

a. Squamous metaplasia of transitional cells

150. Lung tumor in a women , nonsmoker and the lesion is situated peripherally. a.
Adenoca

b.

14

151. That multiple times repeat of scaphoid blood supply a. From the distal end

b. c. d. e.

152. Neonate abnormal skin crease in groin a. development hip dysplasia


b.

153. 65/ F with Breast Ca which is ER –ve . PR –ve but Her2 neu + ve . what postop regime
you give as adjuvant ?

a. Tamoxifen
b. Tamoxifen and ECF regime c. Herceptin and ECF
d. Herceptin only

154. DVT diagnosis best intial

a. b. c.

compression ultrasound plethyshmography

Venography

155. Fracture patella management

1. Wire binding
2. Excision of temdon
3. Boot cast
d.

and there were lot of questions on ortho/ sup limb nerve and hand tendons ....i do not wish
to and even if I wish I cant remember those Rowdy question

thanks .

APRIL MRCS A 2018 _ RECALL

1. Posterolateral hernia in child…..reason?


a. Failure of closure of right pleuroperitoneal membrane
b. Failure of closure of left pleuroperitoneal membrane
c. Failure of closure of septum transversum
d. Failure of closure of contribuiton fromnthe thoracic cavity wall

2. A statistician wanted to compare some parameter which was not distributed


normally (the q mentioned few patients had extremes of values) between two
groups
a. Chi square
b. Mann whitney U
c. Paired T test
d. Unpaired T test
e. Kruksal wallis
3. 2 cm mass in lung in a smoker, another 1 cm mass in liver suspected to be
hemangioma. No systemic signs
best investigation NEXT?
a. MRI liver
b. MRI whole body
c. CT chest
d. FDG PET CT
e. USG

4. Loss of weight, cough, tiredness, Multiple lymphadenopathy, Lateral aspect of


tongue hairlike lesions
a. Lichen planus
b. Candidiasis
c. AIDS
5. Nerve injury, deltoid affection, which other muscle will be affecting ..teres minor
6. Fat overweight child, pain hip, external rotation on attempting to flex … Slipped
femoral epiphyses
7. 6yo child pain hip in all range of motions, systematically well, no fever Perthes
8. Trauma to knee, swelling, excess lateral anguity on stress testing, injury to… MCL
9. Injury to knee and swelling, on 90 degrees tibia is more posterior than other side…
PCL injury
10. 3 patients in surgical ward with Cl. Difficile, mode of transmission…
a. Endogenous
b. feco-oral
c. nosocomial
d. respiratory
e.
11. Test for rupture tendon achilles… Simmonds squeeze test
12. Sinus tarsi between bones?
a. … between talus and calcaneus
b. Talus and navi
c. Calc and navi
13. Lift thumb with hand 90 degrees on table (retropulsion test) … EPL
14. Mobile extensive femoral thrombus management initial management
a. Heparin,
b. Venous exploration,
c. Aspirin,
d. Embolectomy
15. Which hormone decreases in warm condition…
a. TSH,
b. ACTH,
c. Vasopressin
16. Cleft palate embryology defect in fusion of what structures? … palatal processes
17. Result of failure to develop caudal portion of metanephros………….Kidney

18. Testicular cancer with normal AFP and HCG


a. Seminoma
b. Teratoma
c.
19. Relation of median nerve with respect to brachial artery in proximal arm… lat to
ant to medial
20. Relation of ulnar nerve to ulnar artery… ulnar to the ulnar artery
21. Lesion in Dorsal root ganglion in neck, what type of loss?
a. Sensory
b. Motor
c. Sympathetic
d. Parasympathetic
e. All of the above
22. Atrophy thenar eminence… median nerve
23. Dermatome of the little finger… C8
24. Injury to esophagus during upper GI scopy ?
a. TPN
b. Nasoenteral feed
c. PEG
d. Elemental diet
25. Ix for rupture esophagus after endoscopy…
a. water soluble contrast
b. barium
c. upper GI scopy
d. Endo USG
e. Chest xray
26. Crohns …operated… resection of distal ileum and many stricturoplasties?...what
nutrition??
a. Elemental diet
b. High nutrition oral or enteral diet
c. TPN
d.
27. Reciprocal of absolute risk reduction …NNT
28. Cushing reflex/triad… ↑BP ↓HR ↓RR
29. Athlete sudden death after marathon
a. SAH
b. IVH
c. EDH
d. SDH
e. Raised ICP
30. 6yo, kid with groin swelling, observed when the kid is playing around, which
spontaneously reduces on lying
a. patent processus vaginalis
b. direct hernia
c. femoral hernia
d. indirect hernia
e. tortion testes
31. Child with pain in the right scrotum with redness and irreducible groin mass
a. direct hernia
b. femoral hernia
c. indirect hernia
d. tortion testes
e.
32. Intrinsic pathway…factor IX

33. Old woman displaced # distal radius, reduced and put in slab/POP for 6 weeks,
complaining of pain and limited ROM in both supination and pronation after 3
months…
a. CRPS
b. Malunion
c. Nonunion
d.
34. Cervical lymph node, FNAC squamous cell ca, upper tract scopy was found to be
normal. What investigation next ?
a. CT Neck
b. MRI neck
c. FDG PET CT
d. MRI whole body
e. ??
35. Diarrhea, pruritus ani, cellotape examination shows egg in stools…
a. Mebendazole
b. Metronidazole

36. Some q on clinica scenario of hypercholesterolaemia….


a. Hypothyroidism decreases cholesterol
b. Hypothyroidism increases cholesterol
c. ??

37. Advantage of carbohydrate rich drink in enhanced recovery program?


a. decrease post-operative insulin resistance and negative nitrogen
balance
b. better abdominal muscle function
c. better respiratory muscle function
d. shorter hospital stay
e.
38. What’s in direct contact with T5 body…
a. azygos,
b. left atrium,
c. aortic arch OR its junction with ascending aorta
d.
39. Shoulder pain between 60-120 degrees… supraspinatus tendinitis
40. Gun shot to on right side below costal margin, injury to...
a. pylorus,
b. gallbladder
c.
41. Some trauma, led to UMN lesion of leg muscles. Where can be the lesion?
a. L1-2,
b. T12-L1
c. L2-L3
d. L3-L4

42. Chinese man with LN enlargement and headache and deafness nasopharyngeal
carcinoma
43. Spastic unilateral leg paraplegia which artery
a. ACA
b. MCA
c. PCA
d. Ant choroidal art
44. Impotence and vascular insufficiency, clue to Leriche syndrome occlusion to
both common iliacs
45. Femoral canal lateral border… femoral vein
46. Diagnosis of DVT by… compression US
47. Patient with Barret’s esophagus… adenocarcinoma
48. Golfer with 1st Carpometacarpal joint pain, XR showed narrowed joint space and
subchondral cysts… OA
49. Gangrene of the anterior abdominal wall…Cl. perfringens
50. Post-partum lady dies after 3 days causative organism… (???)
51. Osteomyelitis causative organism… Staph aureus
52. Brown sequard $... ipsi motor and proprioception + contra pain and temp
53. Hypercalcemia in a metastatic scenario, Ca 3.7 mmol/L, ECG abnormalities,
“Initial” treatment…
a. IV 0.9% sodium chloride
b. Steroids
c. Pamidronate
d. Cinacalcet
e.
54. Hypothermia ECG change… J wave
55. 1cm melanoma over the back, best management…
a. excision biopsy with 2mm margin,
b. excision biopsy with 2cm margin
c. core biopsy
d.
56. 23-25 yr old female, single mobile breast mass, most appropriate investigation
a. USG
b. Mammo
c.
57. Site of action of fursemide… ascending loop of Henle
58. Site of action of thiazide… DCT
59. Male gynecomastia which drug… spironolactone
60. Posterior duodenal ulcer collapse, which artery… gastroduodenal
61. Site of Dartos muscle subcutaneous
62. Waddling gait, injury to… superior gluteal nerve
63. Bypass with graft, later is blocked… neo intimal hyperplasia
64. COPD patient… ↑CO2 ↑HCO3
65. Anterior relation to right adrenal…IVC
66. Good prognosis for melanoma…
a. Breslow thickness of 0.6
b. Clarks level 5
c. Lymphatic invasion
d. Satellite nodules
e. Ulceration

67. 62% burn patient presents with lower leg edema after 3 days…(pain was not
mentioned in the q)
a. Hypoalbuminemia
b. DVT
c.
68. Picture of x-ray with arrow I think was pointing at a structure at C3 level
a. hyoid bone
b. cricoid
c. thyroid
d. epiglottis
69. Male with pain in testes and urethral discharge…
a. Gonorrhea
b. Chlamydia
c.
70. Blood supply to transverse colon… middle colic artery
71. CPP calculation…BP was 125/80, ICP was 20….75
72. Submandibular duct excision injury to… lingual nerve
73. Post parotidectomy patient received radiation treatment, followed by
presentation with dryness of eyes, which ganglion involved…
a. Pterygopalatine
b. Otic
c. Ciliary
d. submandibular
74. Injury to head, fixed dilated pupil, cause…
a. unopposed sympathetic fibers
b. unopposed parasym
c. overaction of parasym
75. Osteoporotic crush to vertebra, serum calcium is likely to be…
a. Hypercalcaemia
b. Hypocalcaemia
c. Normocalcemia
76. Some clinical scenario, hypercalcaemia, given steroids, resolved………sarcoidosis
77. HPE in rheumatoid arthritis??????
a. Necrobiotic granuloma
b. Foreign body giant cell with lymphocytes
c. Macrophages
d. Eosinophilic granuloma
78. Fall in blood BP, 1st reaction…
a. sympathetic system
b. RAAS
c.
79. 11yo child with special need, going for laparoscopy, type of consent… (consent
parents)
80. Ligation of splenic hilum, avoid injury “during the surgery” to what…
a. tail of pancreas
b. greater curvature
81. Ureter stone radiating to groin, which nerve root… (T11-L1)
82. What structure is in front of uncinate process… SMA
83. What forms the anterior surface of the heart
a. Mostly Rt atrium and ventricle
b. Mostly left ventricle
c.
84. Pigmentation,,,, clinical picture like addisons, electrolytes Na 128, K 5.8 or 6.8,
which hormone to check (ACTH)
85. Some scenario with single site metastatic bone pain treatment…
a. radiotherapy, ??
b. NSAIDs
c. PCM
d. steroid
86. Ovarian mass and inner thigh numbness, which nerve… obturator
87. Bilateral cervical lymph node spread, which type of cancer…tongue
88. Skin lesion, pearly… BCC
89. Skin lesion, central keratinous something…keratoacanthoma
90. UK NICE guidelines on people who received transfusion
a. can’t donate in future until viral markers are negative
b. Can donate within 6 weeks
c. Can donate within 12 weeks
91. Anterior compartment syndrome affecting… 1st dorsal web space
92. Pathology of berry aneurysm in 45yo…
a. Atherosclerosis
b. Cystic medial necrosis
93. site of lumbar puncture…
a. at the level of iliac crest ??
b. at the level of transtubercular plane
c. at the level of posterior superior iliac spine
d. at the level of posterior inferior iliac spine
e. at the subcostal level ??

94. ulcer at the tip of penis


a. squamous cell ca
b. adeno
c. transitional cell ca
d.
95. Ix for pheochromocytoma… urinary VMA
96. post-op day 6 , operated after trauma, with continued bleeding PT 18 secs, APTT
60 secs, Fibrinogen 0.1 … (DIC)
97. a hypertensive taking Ramipril. Q was , constriction of efferent arteriole by:
a. angiotensin 1
b. renin
c. angiotensin 2
d. aldosterone
e. ANP

98. some guy with a knee swelling and sudden pain in legs after some 7 days which
and scenario of Moses sign although the name wasn’t there, with disappearance of
knee swelling …DVT
99. caecal cancer, resected, patho gave diagnosis T4, what do u mean by that?
a. Involving surrounding abdominal musculature
b. Multiple lesions in the resected specimen
c. Muscle invasion
d. Lymphatic or vascular invasion
e.
100. post op patient with known Graves, P 120, BP 160, treatment after salvaging then
acute phase
a. betablocker + iodide
b. betablocker + thioamides
c.
101. Best investigation of choice for parathyroid adenoma
a. Sestamibi scan
b. USG
c. CT
d. MRI
e.
102. Can’t remember, most likely risk factor for HCC… (alcohol)
103. Some 40s male, nausea, vomiting, blurring of vision and headache, particularly in
the morning with no focal neuro signs
a. Raised ICT?
b. Frontal glioma?
c. EDH?
d. SDH?
e. SAH?
104. Clinical scenario of MM, HPE what to expect
a. Plasma cells
b. Macrophages
c. Lymphocytes
d. Eosinophils

105. Asian middle aged man, found to have cavitatory lesion at the apex of left lung,
HPE expected … (TB)
106. Associated with epispadias……(extrophy of bladder)
107. Hypospadias…developmental anomaly of urogenital fold
108. Can’t remember, some question about diffuse lung infiltrates or ARDS, cause of
clinical picture?
109. Complete expiration from complete inspiration….lung volume?
a. VC
b. TLC
c. FRC
d. ERV

110. 30 yr old man, presented with Mediastinal mass. FNAC shows glandular pattern,
absence of neuroendocrine markers, illdefined clumps of cells.
a. Adenocarcinoma ??
b. Thymoma ??
111. Some trauma or surgery on condyles of femur or tibia. Followed by loss of
pulsations. Injury? .... (popliteal)
112. Patient with liver laceration, during ot clamp free margin of lesser omentum.
Possible damage to?
a. CBD
113. Post cardiac transplant… what increases Cardiac output?
a. Increased atrial filling
b. Increased intrathoracic pressure
c. Increased intrapericardial pressure
d. Some option acting via autonomic nervous system
114. Injury to axilla, numbness lateral forearm, which muscle will be affected as well?
a. Brachialis
b. Brachioradialis
c. Flexor carpi radialis
d. Pronator teres
e.
115. Can’t remember, question about heart valve component options had
a. Dense fibrous tissue
b. hyaline cartilage
c. smooth muscle
d. ?
116. Ureter relations,
a. Gonadal veins Cross anteriorly
b. Common iliac divide anteriorly
c.
117. pain on walking down hill but not uphill… (spinal stenosis)
118. patient post burn, with hematemeses… (acute gastric dilatation)
119. multiple bone pains, on exam, illdefined hard prostrate , PSA 300. Management?
a. TURP,
b. radical prostatectomy,
c. incision of prostrate
d. hormonal therapy
e. active surveillance
120. long standing venous ulcer, swabbed , found to have MRSA, no signs of infection
given in the q
a. oral lincopeptides( Vanco or Teico)
b. other beta lactams
c. beta lactams
d. beta lactams
e. no antibiotics

121. effect of smokig on body


122. trauma patient, “properly resuscitated”, when log rolling , suddenly became
hypoxic or decompensated
a. displacement of tracheal tube
b. spinal injury

123. history of carrying heavy weight with back pain….facet arthropathy


124. tooth extraction, bleeding stopped then, started after some time….Factor VIII
125. mass in breast of 25 yr lady, mobile, non tender, what inv ?
a. USG
b. Mammo

126. Lady underwent mastectomy. Split skin graft taken from thigh. What type?
a. Allograft
b. Isograft
c. Autograft
d. Meshed graft ???
e. Xenograft
127. Post op patient of zygomatic repositioning, ear discharge, deafness …#petrous
part
128. Pharyngeal pouch, between which muscles?... Thyropharyngeus and
cricopharyngeus
129. 16 years old boy with gynaecomastia …..physiological)
130. Renal mass noted in USG. Next investigation?
a. CT
b. MRI
c. FNAC
d. PET?
131. Vessel originating above piriformis….sup gluteal
132. Trendelenberg test +ve……. gluteus medius
133. Post op patient deteriorating picture, P 60-70, BP 80/50, CVP 20
a. Heart block
b. Sepsis
c. Cardiac failure
134. Acute abdomen, amylase 110 [normal below 100], Ca 2.4 [2.1 – 2.8], LFT normal
picture,
a. Acute intermittent porphyria
b. Perforation
c. Pancreatitis
d. Rest of the options I don’t remember , but nothing matching
the q
e.
135. Type 1 Diabetic patient on long term NSAID for osteoarthritis or some painful
condition, post op creat high, urea high
a. Reduced renal perfusion
b. Diabetic nephropathy
c. NSAID induced renal injury
d.
136. Etiopathogenesis of AAA……..atherosclerosis
137. Patient known case of iliac or femoral atherosclerosis or aneurysm. Came with
sudden onset pale pulseless limb….embolism
138. Patient with known periph vasc ds, underwent fem-distal bypass 1 yr back, now
came with painful limb, DPA not palpable, but doppler showing monophasic flow
until ends…. DVT
139. Mass in breast, with eczema of nipple areola. Q was , apart from mammo what
else will u do?
a. Exfoliative cytology
b. FNAC
c. Biopsy
d. MRI
140. Which of the following is true about Rt coronary artery?
a. Originates posteriorly from aorta
b. ………………anteriorly…………………
c. Originates from aortic sinus below the aortic valves
d. Present in posterior IV groove
141. Angle of loiu…junction of arch and descending
142. Another q Structure at/attached to T5..
a. Left atrium
b. Tracheal bifurcation
c. Bifurcation of pulmonary trunk
d. Bifurcation of right bronchus
e.

143. Some surgery led to injury to thoracic duct, surgeon wants to ligate it from
thoracic approach. Where to find
a. Behind ascending aorta
b. Venecaval opening
c. Esophageal opening
d. Aortic hiatus
144. Post appendectomy, pus collection in…..
a. rectouterine pouch
b. vesicouterine ”
c. behind the rectum
d. in front of bladder
145. Scrotum… (superficial inguinal)
146. Ovary… (para aortic)
147. Below anal verge (superficial inguinal)
148. Middle aged man, testicular mass, HPE shows cells separated by fibrous septa
containing lymphocytes…
a. Classical seminoma
b. Spermatocytic seminoma
149. Surface marking of IJV
a. styloid process to medial end of clavicle
b. ear lobule to medial end of clavicle
c. mastoid process to medial end of clavicle
d. lesser cornu of hyoid to medial end of clavicle
150. myasthenia……antibodies to acetylcholine
151. NSAIDS causing ulcer…by what mechanism … Cyclooxygenase
152. Mechanism of peptic ulcer
a. Mucous cells
b. Chief cells
c. Oxyntic cells
153. Gunshot which entered at the junction of right rectus and right costal cartilage
and exits exactly from the opposite. Structure most likely injured ?
a. GB
b. Renal hilum
c. Pylorus
154. Some surgery on leg done under spinal with anaesthesia upto umbilicus…..T10
155. Patient with 9th 10th 11th and 12th cranial nerve lesion… medulla
156. Cause of metabolic alkalosis?
a. Nasogastric aspiration
b. Alcohol
157. Post op gustatory sweating….. parasym fibres developing into sweat glands
158. Some scenario and inj ampicillin or amoxicillin given; soon presented with
scenario of anaphy rxn. Mediator?
a. Mast cells
b. Eosinophils
c. Lymphocytes
d. Macrophages
EMQ Bladder
159. Egyptian, hematuria… SCC
160. Female long history of indwelling catheter and multiple recurrent infections,
presents hematuria…SCC
161. Male in dye industry I think…TCC
EMQ on Ear
162. Discharge from ear for a 10 years, with middle ear perf and facial nerve
palsy…cholesteatoma
163. … ()
164. … ()
165. Some kind of mass in neck? Don’t rememebr ……….Otitis media with effusion
Stupid EMQ on Jaundice
166. Man with pain, jaundice, fever….had similar episodes of pain abdo
previously….choledocholithiasis (cholangitis was not in options)
167. 30 yrs follow up patient of FAP operated for colon, now presented with OJ…
Carcinoma of duodenum
168. … ()
169. … ()
EMQ Fluid burn calculation in 24h
170. 36% burn in some 60 or 70 kg
171. 19 yr old with 90 kg weight with burn of a leg and perineum
EMQ chest trauma
i. Chest drain
ii. Chest drain with suction
iii. CT
iv. Echo
v. Thoracotomy
172. Stab to right 4th ICS, normal CXR, tachy, rest normal
173. Stab to the left 5th ICS, tachy, rest normal
174. Stab to 5th ICS post axillary line, CXR showing pneumothorax with fluid level

EMQ Chest pain in pregnancy


175. 36 weeks, 28 yr, c/o Chest tightness and dyspnea. On exam, cyanosis and one
more alarming feature. Dad died of AMI at 62 yrs…..AMI
176. 36 weeks, 28 yrs, father died of AMI at 42 yrs, chest pain, no other info.
Exceptionally(word used was some different) tall….. Aortic Dissection
177. 36 weeks, 28 yrs, Preg lady with pleuritic chest pain followed by dyspnoea, father
died at 60…. PE
EMQ Child gastrointestinal disorders
178. 6 months, sausage shaped mass…intussusception
179. 6 weeks non bile stained vomit after feeds…pyloric stenosis
180. Tender distended abdomen, per rectal bleed in 6 weeks baby……midgut
volvulus
181. …
EMQ GCS
182. E2V2M2
183. E1V1M1
EMQ Ortho patterns of fracture
184. Twisting force… spiral
185. Child fracture with bowing happening after injury greenstick
186. Direct blow to femur… transverse
187. Fracture humerus with metastatic lesion, mechanism was fracture happened
while she was trying to aboard a bus and got pulled by the running bus oblique
EMQ Ortho nerve injury
188. Posterior approach to femur… (tibial? common peroneal? posterior cutaneous
nv of thigh?) (sciatic was not in option)
189. Humerus posterior approach… radial
190. Medial approach to ankle… saphenous nerve
EMQ urethral injury
191. Pelvic fracture, unstable patient, DRE prostate unusual location… rupture
membranous urethra
192. Fall with legs wide open on handle bar bulbar urethra injury
EMQ plastic
i. Full thickness graft
ii. Partial thickness graft
iii. Pedicled flap
iv. Microvascular free flap
v. Advancement flap
vi. Wound excision and primary closure
vii. Wound excision and 2nd ry closure
viii.
193. Dog bit off nose…
194. Cut wound scalp with dirt…
195. Garden fork in toe, 1cm dusky wound
EMQ Thyroid
196. 1 cm MCT … total thyroidectomy
197. 0.5 cm nodule found in one lobe of thyroid while operating for another surgery
probably parathyroid… hemithyroidectomy
EMQ Types of biopsy
198. Venous ulcer…incision biopsy

EMQ on Splenomegaly
199. Caucasian man, antibodies to EBV +... (Infectious Mononucleosis)
200.

EMQ on Acid Base

APRIL MRCS A 2018 _ RECALLS

MRCS April 2018

Paper I

1. Tertrology of fallot is characterized by which one of the following malformations?

1. Aortic Stenosis
2. Atrial septal defects
3. Hypertrophy of left ventricle
4. Pulmonary stenosis
5. Transposition of great vessels

2. Posterolateral diaphragmatic hernia in child reason?

1. Failure of closure of right pleuroperitoneal membrane


2. Failure of closure of left pleuroperitoneal membrane
3. Pleuropericardial membbrane
4. Failure of closure of septum transversum
5. Failure of closure of contribuiton fromnthe thoracic cavity wall

3. A patient developed extradural hematoma following a road traffic accident, on


examination he was found to have inability to abduct and paralysis of downward guaze,
what nerve is most likely affected?

1. Abducsence
2. Occulomotar
3. Optic
4. Facial
5. Trochlear
4. Adrenaline increases heart rate by acting on which of the following receptors

1. α1
2. β1
3. α2
4. β2
5. ɗ1

5. In Patients with Myasthenia Gravis, The autoantibodies are directed against which of the
following?

1. Presynaptic adrenergic recerptors


2. Presynaptic Acetylcholine receptors
3. Postsynaptic Adrenergic receptors
4. Postsynaptic cholinergic receptors
5. Central cholinergic receptors

6. A patient presented with a central nervous system tumor causing paralysis of the
glossopharyngeal and vagus nerves. What is the lost likely location of the tumor?

1. Mid-brain
2. Pons
3. Medula
4. Ventricles
5. Cervical cord

7. A patient presented with involvement of the long saphenous nerve following surgery for
the varicose veins. What is the root value of the saphenous nerve
A. L2,3
B. L3,4
C. L5
D. L5S1

April 2018 MRCS A


RK

E. S 1, 2
8. Hypospadias is a congenital anomaly of the penile urethra. The underlying cause is mal-
development of which of the following structures.

1. Urogenital sinus
2. Genital tubercle
3. Mesonephros
4. Genital folds
5. Mullerian duct

9. A Patient presented with fracture of the face involving maxilla and numbness over the
cheek. Which of the following nerve is most likely involved?

1. Mental
2. Infra-orbital
3. Superior alveolar
4. Auriculotemporal
5. Supratrochlear

10. P patient presented with penetrating injury in the lower part of his neck, his primary
survey revealed no major vascular injury and the patient was stable. He later was found to
have hoarseness of voice. Injury to which nerve can cause hoarseness?

1. Glasspharyngeal
2. Hypoglossal
3. Facial
4. Vagus
5. Accessory

11. Which of the following can be used as a surface landmark for external jugular vein?
1. Lobule of the ear to the Sternoclavicular joint
2. Mastoid to the Sternoclavicular joint
3. Styloid process to the Sternoclavicular joint
4. Anterior Border of sternoclidomastoid
5. The angle of mandible to the middle 3rd of clavicle

12. A patient presented with fatiguability, ptosis. Which of the following tumors can cause
myasthenia like symptoms.
A. Thymus
B. Adrenals
C. Lung
D. Liver
E. Kidney
13. A patient presented with fracture of tibia and above knee cast was applied. He later
presented with pain on plantar flexion of the great toes. Other compartments were normal.
Which of the currospi=onding areas will have paresthesia?

1. Dorsum of foot
2. Lateral border of foot
3. 1st web space
4. Medial border of foot
5. Sole of the foot.

13. A patient presented with calf pain on walking 100 mt distance. It was diagnosed as a
claudication pain. Block at Which of the following levels may responsible for the symptoms?

1. Superficial femoral
2. Popliteal
3. Deep femoral
4. External iliac
5. Profunda femoris

April 2018 MRCS A

RK

14. After eating peanuts, patient developed significant wheezing and dyspnea. Which of the
following cells are responsible for these symptoms

1. Neutrophils
2. Basophils
3. Eosinophils
4. Macrophages
5. T lymphocytes
15. Sinus Tarsi is an important landmark, located between which 2 bones?

1. Talus and calcaneum


2. Talus and navicular
3. Calcaneum and navicular
4. Navicular and the metatarsals
5. Navucular and cuniforms

16. A Patient presented with crutch related Injury to axilla, he has numbness over the
lateral aspect of his forearm, which muscle will be affected as well?

1. Brachialis
2. Brachioradialis
3. Flexor carpi radialis
4. Pronator teres
5. Triceps

17. A statistician wanted to compare some parameter which was not distributed normally
(the q mentioned few patients had extremes of values) between two groups

1. Chi square
2. Mann whitney U
3. Paired T test
4. Unpaired T test
5. Kruksal wallis

18. Nerve injury following shoulder dislocation, deltoid is affected, which other muscle will
be affected

1. Latissimus Dorsi
2. Teres minor
3. Teres Major
4. Subscapularis
5. Supraspinatus

19. 1. A patient was presented to emergency with palmar laceration over the radial aspect
of the Left palm at the base of the thumb. Which muscle is responsible for the movement
of the thumb, when hand placed flap on the table to bring the thumb at 900 to the plane of
plam.

1. Adductor pollicis
2. Abductor pollicis
3. Opponens pollicis
4. Flexor pollicis
5. Dorsal intreossi
20. In a patient with Cleft palate, it represents defect in the embryological fusion of what
structures?

1. Frontonasal
2. Maxillary
3. palatal processes
4. lateral nasal
5. medial nasal

21. Embrionic failure to develop caudal portion of Metanephros leads to what abnormality
in the new born A. Kidney
B. Uterter
C. Bladder

April 2018 MRCS A

RK

D. Seminal vesicle E. prostate

22. A patient is undergoing exploration of his brachial artery. What is the Relation of
median nerve with respect to brachial artery in proximal arm till the elbow

A. B. C. D. E.

Lateral to anterior to medial Anterior to lateral to posterior Medial to anterior to medial


Lateral to anterior to lateral Posterior to lateral to medial

A patient presented with a laceration over the volar ulnar aspect of the wrist. He is
scheduled for exploration

23.
of the wound. What is the most likely relation of ulnar nerve to ulnar artery at the wrist.

A. B. C. D. E.

Anterior to the artery Ulnar to the ulnar artery Posterior to the artery Artery if ulnar to the
nerve Lateral to the artery

24. Lesion in Dorsal root ganglion of a spinal nerve in the neck is most likely to lead to what
time of loss?

1. Sensory
2. Motor
3. Sympathetic
4. Parasympathetic
5. All of the above

25. A patient presented with a healed laceration at the level of the elbow. Association of
which nerve injury would lead to the Atrophy of the thenar eminence muscles of the hand?

1. median nerve
2. Ulnar nerve
3. Radial nerve
4. Musculocutaneous
5. Anterior intresseous

26. A Patient presented with cervical brachial plexus injury while undergoing resection of a
cervical rib. She presented with loss of sensations over the little and ring fingers. Injury to
which of the dermatomes below is responsible for loss of sensations over the little finger?

1. C8
2. C7
3. T1
4. C6
5. C5

27. Which of the following statement is true regarding femoral canal?

1. Pectineal fascia forms the medial border


2. The inguinal ligament of posteriorly
3. The lateral border is formed by femoral vein
4. The lacunar ligament forms the lateral border
5. Fascia iliaca is posteriorly

28. Patient with known Barret’s esophagus presents with carcinoma of esophagus. what is
the most likely histological type
A. Adenocarcinoma

April 2018 MRCS A

RK

B. SCC
C. Lymphoma D. Melanoma E. Sarcoma

29. 1. What is the most common organism causing Osteomyelitis in pediatric patients.

1. Staph aureus
2. Streptococcus pnemoniae
3. Salmonella
4. H. Influanzae
5. N. Gonorrhoae

30. Heart valve is composed of the following structure?

1. Dense fibrous tissue


2. hyaline cartilage
3. smooth muscle
4. fibrocartilage

E.

31. Lady underwent mastectomy. Split skin graft taken from thigh. What type?

1. Allograft
2. Isograft
3. Autograft
4. Meshed graft
5. Xenograft

32. Lesser sac is potential space known for intra-abdominal collections. which of the
following statements regarding lesser sac is true
A. inferior vana cave forms the lateral borber.
B. The portal vein is in the anterior free edge

C, The stomach forms the posterior wall


D. cystic duct is in the anterior free border
E. Left gastric artery is in the posterior aspect

33. A 68 years old man presented with a inguinal hernia. During repair it was found to be
direct hernia. Which of the following statemnts is correct regarding direct hernia
A. inferior epigastric artery forms the lateral border

34. which of the following statements is correct regarding the inguinal canal Lateral 1/3rd
of the posterior wall is formed by transversalis fascia

Mixed Paper I and Paper II

Q60: A caucessean girl presented with splenomegaly, jaundice and increased billurubin
levels. Which of the following is most likey disgnosis.
A. Hereditary spherocytosis

April 2018 MRCS A


RK

2. Sickle cell disease


3. Acute intermittent porphyria

D. Splenosis E. ??

1. 2 cm mass in lung in a smoker, another 1 cm mass in liver suspected to be


hemangioma. No systemic signs best investigation NEXT?
1. MRI liver
2. MRI whole body
3. CT chest
4. FDG PET CT
5. USG
2. Loss of weight, cough, tiredness, Multiple neck lymphadenopathy, Lateral aspect of
tongue hair like lesions

a. Lichen planus

b. Candidiasis

c. AIDS

d. SCC

e.

3. 13 years old, overweight child, with 2 weeks history of pain in right hip, There is no
history of trauma, child is not able to bear weight. On examination: external rotation
on attempting to flexion of the hip.
1. Perthes disease
2. DDH
3. Slipped upper femoral epiphyses
4. Chondromalacia
5. Fracture neck femure
4. 6 years old child presented with pain hip with all range of motions. He is
systematically well, no h/ofever.
1. Perthes
2. DDH
3. Aplasia of femoral head
4. Septic arthritis
5. Juvenile Rheumatoid arthritis
5. 3 patients in surgical ward with were found to be having diarrhea. Cl. Difficile was
found to be causative organism. What is possible mode of transmission of the
Clostridium difficile toxin?

A. Endogenous B. feco-oral
C. nosocomial D. respiratory E. Contact

6. A 35 years old lady had collapsed suddenly and felt that something struk suddenly on
the back of her lower leg, before collapsing, while trying to cross the road. What
injury must be responsible for this

a. Tendoachillis

2. Patellar tendon
3. Ankle dislocation
4. Posterior tibial nerve

April 2018 MRCS A

RK

e. Tibialis posterior

7. What is the Clinical test for diagnosing a ruptured tendo-achilles?

A. Simmonds squeeze test

8. Sinus tarsi between bones?


1. Between talus and calcaneus
2. Talus and navi
3. Calc and navi
9. (retropulsion test) ... EPL
10. A patient returing to UK from China after a business meeting presented with a warm
swollen right thigh with extensive mobile femoral thrombus in the femoral vein.
What is the most appropriate initial management?
1. Heparin,
2. Venous exploration,
3. Aspirin,
4. Embolectomy
5. IVC filter
11. Patient Presented with tachycardia, high blood pressure and warm peripheries.
Which of the following hormone decreases with hyperthermia?

A. TSH,
B. ACTH,
C. Vasopressin
D. Cortisol
E. Catecholamine

12. A 41 years old male presented with a 2 CM Irregular swelling in his right testes. The
hormonal assey was negative for both AFP and B HCG levels. What is the most likely
pathology.
1. Seminoma
2. Teratoma
3. Choriocarcinoma
4. Ledige cell tumor
5. .
13. A Patient sustained Injury to esophagus during upper GI scopy. He is being managed
conservatively for a limited perforation and mediastinitis and is Nil By Mouth. What is
the Most suitable way of providing him with the necessary nutritional support?
1. TPN
2. Fine bore Naso-gastric tube
3. PEG
4. Elemental diet
5. Naso-junal feeding
14. A patient sustained injury to his lower esophagus while undergoing upper GI
endoscopy. He is being managed conservatively with a limited air leak and is
currently Nil by Mouth. For last 2 weeks.
What is the best way to confirm the healing of the perforation.

A. Water soluble contrast

April 2018 MRCS A

RK

2. Barium Swallow
3. upper GI scopy
4. Endo USG
5. Chest x ray
15. A patient Operated for Crohns disease with multiple strictures for resection of distal
ileum and many stricturoplasties. He developes high output fistula with an output of
1800ml. What is the beast wasy to provide him nutritional support?

16. What is A.

B. C. D. E.

A. Elemental diet
B. High nutrition oral or enteral diet C. TPN
D. Feeding Jujunostomy
E.

Reciprocal of absolute risk reduction ...NNT Absolute Risk reduction


Relative risk reduction
Odds ratio

Sensitivity Probability

17. A patient with Extradural Hematoma Presented to the emergency department if


having hypertension and increased intracranial pressure. What of the Following are
the Components of the Cushing reflex/triad?
18. A 24 year old Athlete collapsed after marathon playing and could not be rescuscited.
His autopsy reported intracranial bleeding. What of the conditions below will be
most likely the disgnosis?
1. Chronic SDH
2. IVH
3. EDH
4. SDH
5. SAH
19. 6 years old kid is brought by his concerned mother with a groin swelling, observed
when the kid is playing

around, which spontaneously reduces on lying. What of the following may be


responsible for this presentation?

1. Patent processus vaginalis


2. Direct hernia
3. Femoral hernia
4. Indirect hernia
5. Tortion testes
20. Child with pain in the right scrotum with redness and irreducible groin mass
1. direct hernia
2. femoral hernia
3. indirect hernia
4. tortion testes
April 2018 MRCS A

Heart Rate

Blood Pressure

Respiratory Rate



RK

E. undescended testes

21. Lack of Which of the following factors is responsible for defect in the Intrinsic
pathway of coagulation
1. Factor IXa
2. Factor VIII
3. Factor XII
4. Factor X
5. Factor VII
22. Old woman displaced # distal radius, reduced and put in slab/POP for 6 weeks. After
removal of slab she is complaining of pain and limited ROM in both supination and
pronation after 3 months.
1. CRPS
2. Mal-union
3. Non-union
4. Compartment syndrome
5. .
23. A patient with Cervical lymphadenopathy was investigated and FNAC suggested
squamous cell Ca. Clinical Examination and upper tract scopy were found to be
normal. What should be the next line investigation?
1. CT Neck
2. MRI neck
3. FDG PET CT
4. MRI whole body
5. Radio-istope scan
24. 35 years female presented with chronic episodic Diarrhea, pruritus ani. She was
investigated for her symptoms and the cellotape examination shows egg in stools.
1. Mebendazole
2. Metronidazole
3. Albendazole
4. Ivermectine
5. Pyrental palmoate
25. A patient presented with low blood pressure, lethargy and muscle weakness. Which
of the hypercholesterolaemia in serum analysis of this patient

A. Hyperthyroidism B. Hypothyroidism
C. Hypertriglicridemia D. Cortisol
E. ACTH

26. Advantage of carbohydrate rich drink in enhanced recovery program?


1. decrease post-operative insulin resistance and avoids negative nitrogen
balance
2. better abdominal muscle function
3. better respiratory muscle function
4. shorter hospital stay
5. .
27. What Structure lies at the level of T5 vertebra
1. Azygos vein
2. left atrium,
3. Aortic arch OR its junction with ascending aorta
April 2018 MRCS A

RK

4. Bifurcation of trachea
5. Junction of both brachiocephalic veins

28. A Man presents after overnight painting of a celing with Shoulder pain. The Pain is
mainly during the movements of shoulder between 60-120 degrees. What is the
most likely disgnosis?
1. Rupture of the supraspinatus tendon
2. Subacromial bursitis
3. supraspinatus tendinitis
4. Frozen shoulder
5. Calcific tendonitis
29. A main sustains a bullet injury to his abdomen. The bullet passes directly posteriorly,
enternin at a point where the lateral border of the rectus abdominis touches the
costal margin. The structure that is most likely to get injured is
1. pylorus,
2. Gallbladder
3. Inferior vena cava
4. Posral vein
5. Cystic artery
30. What of the following lesions is most likely to cause brisk reflexes? A. L1-2,

B. T12-L1 C. L2-L3 D. L3-L4 E. L5-S2

31. A 51 years old chinese man presents with middle ear effusion, conductive hearing
loss and with LN enlargement. What is the most likely disgnosis?

A. SCC
B. Suppurative Otritis media C. Nasopharyngeal carcinoma D. Adenocarcinoma
E. Infected Polyps

32. Spastic hemiplegia is caused by lesion of which artery


1. ACA
2. MCA
3. PCA
4. Ant choroidal art
33. A 38 years old male presented with compalints of severe weakeness of his lower limb
and atrophy of the gluteal muscles. On close questioning he reveals impotency. Block
at what level is responsible for these symptoms
1. External Iliac
2. Common Iliac
3. Aorta
4. Bilateral femoral
5. Bilateral common Iliac
34. A 49 years old lady presented with atrial fibrillation presented with swelling,
tenderness and redness over her calf. The diagnosis of DVT can be confirmed by
which of the following tests

A. color Doppler

April 2018 MRCS A

RK

2. Angiography
3. Venography
4. D Diamer
5. Plathysmography

35. A 38 years Golfer with 1st Carpometacarpal joint pain. X-Ray showed narrowed joint
space and subchondral cysts. What is the most likely disgnosis?
1. Gout
2. Pseudogout
3. Osteoarthritis
4. Rheumatoid Arthritis
5. Scleroderma
36. Gangrene of the anterior abdominal wall
1. Cl. Perfringens
2. Aspergillus Fumingatus
3. Streptococcus Perfringes
4. Staphylococcus aureus
5. Pseudomonas Aurogenosa
37. A 32 years old lady collapsed and brought to emergency with shock and warm
peripheries. She dies after 3 days post operatively, what may be the causative
organism for her symptoms?
1. Staphylococcus aureus
2. Pseudomonas Aerogenosa
3. Bacteriods
4. Cl. Perfringens
5. .
38. A 23 years male sustained thorasic trauma and developed hemisection of the cord.
He will show the following pattern of weakness below the level of lesion.

39. Hypercalcemia in a metastatic scenario, Ca 3.7 mmol/L, ECG abnormalities, “Initial”


treatment...
1. IV 0.9% sodium chloride
2. Steroids
3. Pamidronate
4. Cacitonin
5. .
40. A 23 years old man sustained injuries during a skeing accident, he sustained severe
injuries and was recovered and brought to the emergency department. He was found
to have a temperature of 33.5 0. What is trhe possible ECG change related to
hypothermia?
1. Tall T wave
2. J wave

April 2018 MRCS A

Motor Pain and temprature


proprioception
Ipsilateral Contralateral Ipsilateral
contralateral Ipsilateral contralateral
contralateral Ipsilateral ipsilateral

ipsilateral Contralateral
Contralateral

Ipsilateral Ipsilateral Ipsilateral

RK

C. Increased PR interval

4. Narrow QRS complex


5. .

41. A patient presented to the emergency department after collapse. Her blood
investigations revealed the following results

She has been on Fursemide for her coingestive cardiac failure and was thought to be
responsible for these abnormalities what is the site of action of furosemide?

1. Proximal Convoluted Tubule


2. Distal Convoluted Tubule
3. Ascending limb of loop of Henle
4. Collecting duct
5. Descending limb of loop of Henle
42. A 56 years old man presented with severe electrolyte abnormalities. He has been
taking Benzfluthiazide for past many months. What is the site of action of this drug?
1. Lungs
2. Distal Convoluted tubules
3. Collecting ducts
4. .
5. .
43. A 45 years Male presented with enlarged breasts, He has been on multiple drugs.
Which of the following drugs may be responsible for producing gynaecomastia ?

A. Spironolactone B. Amidarone

44. A 52 years old male presented with severe hematemesis. He underwent upper GI
endoscopy which releaved significant bleeding ulcer in the first part of duodenum.
Which vessel may be responsible for his symptoms?
1. Left Gastric
2. Gastroepiploeic
3. Sup. Pancreaticodeodenal
4. Gastroduodenal
5. Common Hepatic Artery
45. A 30 years old man underwent successful repair of hydrocele. Where is Dartos
muscle located during the operation?
1. Between the external spermatic fascia and the cord
2. Between the internal spermatic fascia and the cord
3. Below Cremastric muscle
4. Subcutaneous

E.

46. A Man presented following his surgery with loss of hip abduction and a Waddling
gait, injury to which muscle

may be responsible for this

1. Gluteus Maximus
2. Gluteus Medius
3. Pyriformis
4. Qudratus femoris
5. Obturator externus

April 2018 MRCS A

Values Na 119 K 2.9 Ph: 7.34 Ca 2.2


Normal Range Na 135- 145 7.35- 7.45 2.2-2.6
RK

47. A 65 years old male with successful femoro-popliteal Bypass, presented after 3 years
with pale extremity and lack of distal pulses. He underwent a Doppler study and the
graft was found to be blocked. What may be rewsponsible for the blockade

A. Atherosclerosis
B. Neo intimal hyperplasia C. Muscular thickening
D. Embolism
E. Thrombosis

48. COPD patient... ↑CO2 ↑HCO3


49. Anterior relation to right adrenal...IVC
50. Good prognosis for melanoma...
1. Breslow thickness of 0.6
2. Clarks level 5
3. Lymphatic invasion
4. Satellite nodules
5. Ulceration
51. 62% burn patient presents with lower leg edema after 3 days...(pain was not
mentioned in the q)
1. Hypoalbuminemia
2. DVT

C.

52. A patient presented with dysphagia and history of ingestion of a fish bone. X ray of
the cervical spine was

obtained and revealed the following picture. What is the structure marked with an
arrow?

1. hyoid bone
2. cricoid
3. thyroid
4. epiglottis
53. Male with pain in testes and urethral discharge and fever. On closer enquiry he
revealed having unprotected sex. What is the most likely cause?
1. Gonorrhea
2. Chlamydia

C.
54. Which one of the below mentioned arteries contribute significantly to the blood
supply of the transverse colon
1. Right Colic artery
2. Left Colic artery
3. Middle colic artery

April 2018 MRCS A

RK

4. Iliocolic artery
5. Inferior mesenteric artery

55. A 54 years old man presented with a extradural hematoma. He had increased
intracranial pressure of 20 mm hg. His blood pressure was found to be 125/80 mm
Hg. What is his cerebral perfusion pressure?

A. 90 B. 75 C. 80 D. 65.5 E. 70

56. A 38 years old man presented with recurrent swelling under his mandible for last one
month. Investigations revealed it to be calculus in the submandibular duct. He has
been scheduled for removal of the calculus. Which nerve is likey to get damaged
during this surgery?
1. lingual nerve
2. Hypoglossal Nerve
3. Facial Nerve
4. Chorda tympani nerve
5. Greater auricular nerve
57. A patient underwent parotidectomy and radiation treatment for a parotid gland
tumor. He subsequently presented with symptoms of dryness of eyes, which of the
following is responsible for his symptoms?
A. Pterygopalatine B. Otic
C. Ciliary
D. Submandibular E. Geniculate

58. A 32 years old man, cook by profession, presented with history of road traffic
accident and had signs and symptoms of increased intracranial pressure. He was
found to have dilation of the ipsilateal pupil on the side of

the injury. What is the likely reason for his pupillay dilation?

A. B. C. D. E.

Optic nerve injury


Overaction of parasympathetic stimulation Facial nerve injury
Lack of Sympathetic innervation Unopposed sympathetic stimulation

59. A lady
She was Investigated and was found to have osteoporosis. What will her blood test reveal?

presented with significant Osteoporotic vertebrae and has lost almost 8 cm height over the
past 6 years.

1. Hypercalcaemia
2. Hypocalcaemia
3. Normocalcemia
4. Increaed parathyroid
5. Increased phosphate

60. Some clinical scenario, hypercalcaemia, given steroids, resolved.........sarcoidosis


61. HPE in rheumatoid arthritis??????

A. Necrobiotic granuloma
B. Foreign body giant cell with lymphocytes C. Macrophages
D. Eosinophilic granuloma

62. Fall in blood BP, 1st reaction... A. sympathetic system

April 2018 MRCS A

RK

B. RAAS
C.
63. 11 year child with learning disbilities, presented with enlarged tonsils and difficulty in
breathing. Laparoscopy

needs to be done urgently. Who can give concent for such procedure

1. Parental concent.
2. Patients concent
3. Ask the court
4. Can proceed without concent
5. Cannot do the procedure

64. Ligation of splenic hilum, avoid injury “during the surgery” to what...
1. tail of pancreas
2. greater curvature
3. Spenic flexure of colon
4. Disphragm
5. Ureter
65. Ureter stone radiating to groin, which nerve root... (T11-L1)
66. What structure is in front of uncinate process... SMA
67. What forms the anterior surface of the heart
1. Mostly Rt atrium and ventricle
2. Mostly left ventricle

C.

68. Pigmentation,,,, clinical picture like addisons, electrolytes Na 128, K 5.8 or 6.8, which
hormone to check (ACTH)
69. Some scenario with single site metastatic bone pain treatment...
1. radiotherapy, ??
2. NSAIDs
3. PCM
4. steroid
70. Ovarian mass and inner thigh numbness, which nerve... obturator
71. Bilateral cervical lymph node spread, which type of cancer...tongue
72. Skin lesion, pearly... BCC
73. Skin lesion, central keratinous something...keratoacanthoma
74. UK NICE guidelines on people who received transfusion
1. can’t donate in future until viral markers are negative
2. Can donate within 6 weeks
3. Can donate within 12 weeks
4. 6 months
5. 12 months
75. Anterior compartment syndrome affecting... 1st dorsal web space
76. Pathology of berry aneurysm in 45yo...
A. Atherosclerosis

B. Cystic medial necrosis

77. In adults, what is a correct statement regarding lumbar puncture?


1. Preferred at the level of iliac crest as the cord ends above this level
2. at the level of transtubercular plane as the
3. Preferred at the sacral canal as the canal is widest here
4. at the level of posterior inferior iliac spine
5. At T12 level as the spinal canal is narrowest at this level
78. A 56 years patient presented with irreducible phymosis to the surgical outpatient and
was found to have a fungating mass over his glans, his superficial inguinal lymph
nodes were enlarged. What is the most likely disgnosis?

April 2018 MRCS A

RK

1. Squamous cell Ca
2. Adenocarcinoma
3. Transitional cell carcinoma
4. HPV infection
5. Lichen Planus

79. A patient presented with headache, palpitation, and refractory hypertension. Which
of the following investigation would help to support the disgnosis of
pheochromocytoma?
1. Urinary VMA
2. Urinary cortisol
3. 1-25 hydroxycholecalciferol

D.

80. post-op day 6 , operated after trauma, with continued bleeding PT 18 secs, APTT 60
secs, Fibrinogen 0.1 ... (DIC)
81. A hypertensive taking Ramipril for controlling his blood pressure. Which of the
following causes constriction of

efferent arteriole?

1. Angiotensin I
2. renin
3. Angiotensin II
4. aldosterone
5. Angiotensinogen
82. A 45 years old male presented with a swelling over the posterior aspect of the knee
and pain. He was on conservative management and presented 7 days later with
swelling over his calf and induration, the swelling in

the posterior aspect of the knee had disappeared. What is the most likely diagnosis during
this presentation.

1. Ruptured backers cyst


2. Deep venous thrombosis
3. Varicose veins
4. Aneurysm of popliteal artery
5. Pseudoaneurysm

83. A patient presented with a right iliac fossa mass and investigations confirmed a
caecal cancer. He underwent resection. The resected specimen was reported as T4,
what do u mean by this stage?
1. Involving surrounding abdominal musculature
2. Multiple lesions in the resected specimen
3. Muscle invasion
4. Lymphatic or vascular invasion
5. Involvement of liver or lung
84. A patient presented after thyroidectomy for graves disease with sweating, diarrhea, a
pulse rate of 120, BP 160/110 mm hg, what is the treatment of choice in this case?
1. betablocker + iodide
2. betablocker + thioamides
3. Iodides and thioamides

D.

85. A patient presented with renal stones and sr. Ca 2.94 mmoles/L, what is the best
investigation of choice for solitary parathyroid adenoma
1. Sestamibi scan
2. USG
3. CT
4. MRI
5. I131 uptake scan
86. Can’t remember, most likely risk factor for HCC... (alcohol)

April 2018 MRCS A

RK

87. A 40 years male presented with nausea, and headache, the symptoms were severe
particularly in the morning. On examination there were no focal neuro signs, but
fundoscopic examination revealed papilloedema. What may be the most likely
underlying cause?

A. Raised ICT
B. Frontal glioma C. EDH
D. SDH
E. SAH

88. A Patient presented with back pain. X ray revealed multiple osteolytic lesions in the
vertebrae. A disgnosis of multiple myeloma was made based on the presentation.
Bone marrow aspitrations is most likely to reveal which of the following?
1. Plasma cells
2. Macrophages
3. Lymphocytes
4. Eosinophils
5. Negative bifringent crystals
89. Asian middle aged man, presented with cough and weight loss, x ray revealed mass in
the upper lobe of the lung, he also had mediastinal lymphadenopathy. HPE of the
mass would most likely reveal
1. Squamous cell Carcinoma
2. Small cell carcinoma
3. Tuberculosis
4. Adenocarcinoma
5. Sarcoidosis
90. A patient presented with urethral meatus opening over the dorsal surface of the
penis. This anmoly is commonly associated with which of the following.
1. Extrophy of bladder
2. Exomphalos
3. Spina Bifida

D.

91. Can’t remember, some question about diffuse lung infiltrates or ARDS, cause of
clinical picture?
92. At a pre-anesthesia clinic, spirometry was advised for a patient. Which of the
following statements is correct

regarding the lung volumes or capacities?


A. Vital Capacity is sum of inspiratory reserve volume, tidal volume and expiratory

reserve volume
B. Total Lung capacity is a sum of inspiratory reserve volume and vital capacity
C. Functional reserve capacity is a sum of inspiratory reserve volume and residual

volume?
D. Expiratory reserve volume is ?
93. 30 years old man, presented with Mediastinal mass. FNAC shows glandular pattern,
absence of neuroendocrine markers, ill-defined clumps of cells.

A. Adenocarcinoma ??

B. Thymoma ??

94. Some trauma or surgery on condyles of femur or tibia. Followed by loss of pulsations.
Injury? .... (popliteal)
95. Patient with liver laceration, during ot clamp free margin of lesser omentum. Possible
damage to?
1. CBD
2. Hepatic vein
3. IVC
4. Cystic duct

April 2018 MRCS A

RK

97. Ureter relations,

1. Increased atrial filling


2. Increased intrathoracic pressure
3. Increased intrapericardial pressure
4. Some option acting via autonomic nervous system
5. Peripheral vasoconstriction

1. Gonadal veins Cross anteriorly


2. Common iliac divide anteriorly

E. Common hepatic artery

96. Post cardiac transplant patient wants to join fitness club as he read somewhere that his
low heart rate can be compensated by exercises. Which of the following increases Cardiac
output in a cardiac patient, during controlled exercises?

C.

98. pain on walking down hill but not uphill... (spinal stenosis)
99. patient post burn, with hematemeses... (acute gastric dilatation)
100. multiple bone pains, on exam, illdefined hard prostrate , PSA 300.
Management?
1. TURP,
2. radical prostatectomy,
3. incision of prostrate
4. hormonal therapy
5. active surveillance
101. long standing venous ulcer, swabbed , found to have MRSA, no signs of
infection given in the q
1. oral lincopeptides( Vanco or Teico)
2. other beta lactams
3. beta lactams
4. beta lactams
5. no antibiotics
102. effect of smokig on body
103. trauma patient, “properly resuscitated”, when log rolling , suddenly
became hypoxic or decompensated
1. displacement of tracheal tube
2. spinal injury
104. history of carrying heavy weight with back pain....facet arthropathy
105. tooth extraction, bleeding stopped then, started after some
time....Factor VIII
106. mass in breast of 25 yr lady, mobile, non tender, what inv ?

A. USG

B. Mammo

107. Post op patient of zygomatic repositioning, ear discharge, deafness


...#petrous part
108. Pharyngeal pouch, between which muscles?... Thyropharyngeus and
cricopharyngeus
109. 16 years old boy with gynaecomastia .....physiological)
110. Renal mass noted in USG. Next investigation?

A. CT

B. MRI C. FNAC D. PET?

111. Vessel originating above piriformis....sup gluteal


112. Trendelenberg test +ve....... gluteus medius

April 2018 MRCS A

RK

113. Post op patient deteriorating picture, P 60-70, BP 80/50, CVP 20


1. Heart block
2. Sepsis
3. Cardiac failure
114. Acute abdomen, amylase 110 [normal below 100], Ca 2.4 [2.1 – 2.8], LFT
normal picture,

A.

B. C. D. E.

115. Type 1 Diabetic patient on urea high

A.

B. C. D.

Acute intermittent porphyria

Perforation
Pancreatitis
Rest of the options I don’t remember , but nothing matching the q

long term NSAID for osteoarthritis or some painful condition, post op creat high,

Reduced renal perfusion

Diabetic nephropathy NSAID induced renal injury

116. Etiopathogenesis of AAA........atherosclerosis


117. Patient known case of iliac or femoral atherosclerosis or aneurysm. Came with
sudden onset pale pulseless

limb....embolism
118. Patient with known periph vasc ds, underwent fem-distal bypass 1 yr back, now came
with painful limb, DPA

not palpable, but doppler showing monophasic flow until ends.... DVT

119. Mass in breast, with eczema of nipple areola. Q was , apart from mammo what
else will u do?
1. Exfoliative cytology
2. FNAC
3. Biopsy
4. MRI
120. Which of the following is true about Rt coronary artery?
1. Originates posteriorly from aorta
2. ..................anteriorly.....................
3. Originates from aortic sinus below the aortic valves
4. Present in posterior IV groove
121. Angle of loiu...junction of arch and descending
122. Another q Structure at/attached to T5..

A. B. C. D. E.

123. Some surgery led to injury A. B. C. D. E.


124. Post appendectomy, most A. B. C. D.

Left atrium
Tracheal bifurcation Bifurcation of pulmonary trunk Bifurcation of right bronchus

to thoracic duct, surgeon wants to ligate it from thoracic approach. Where to find Behind
ascending aorta
Venecaval opening
Esophageal opening

Aortic hiatus
At junction with the subclavian vein

likely site of pus collection in.....

rectouterine pouch

vesicouterine ” behind the rectum in front of bladder

125. Lymph Nodes involved in Scrotal infection... (superficial inguinal)


126. Ovary... (para aortic)

April 2018 MRCS A

RK
127. At anal verge (superficial inguinal)
128. Middle aged man, testicular mass, HPE shows cells separated by fibrous septa
containing lymphocytes...

129. Surface marking of IJV

1. Classical seminoma
2. Spermatocytic seminoma

1. styloid process to medial end of clavicle


2. ear lobule to medial end of clavicle
3. mastoid process to medial end of clavicle
4. lesser cornu of hyoid to medial end of clavicle

130. myasthenia......antibodies to acetylcholine


131. NSAIDS causing ulcer...by what mechanism ... Cyclooxygenase
132. Mechanism of peptic ulcer
1. Mucous cells
2. Chief cells
3. Oxyntic cells
133. Gunshot which entered at the junction of right rectus and right costal cartilage
and exits exactly from the

opposite. Structure most likely injured ?

1. GB
2. Renal hilum
3. Pylorus

134. Some surgery on leg done under spinal with anaesthesia upto umbilicus.....T10
135. Patient with 9th 10th 11th and 12th cranial nerve lesion... medulla
136. Cause of metabolic alkalosis?

A. Nasogastric aspiration

B. Alcohol

137. Post op gustatory sweating..... parasym fibres developing into sweat glands
138. Some scenario and inj ampicillin or amoxicillin given; soon presented with
scenario of anaphy rxn. Mediator?

A. Mast cells

EMQs: Bladder Cancer


B. Adenocarcinoma

2. Eosinophils
3. Lymphocytes
4. Macrophages

C. Squamous cell Carcoinoma

4. Transitional cell Ca localized to mucosa


5. Infiltrative Transitional cell Ca

71. A Female with meningomyelocoele and bladder neck obstruction is on self


catheterization program and has history of multiple recurrent infections, presents
significant hemturia hematuria ??SCC
72. 56 years Male with long history of smoking presented with hematuria, during
investigations he was found to have filling defect in his bladder in his IVU... TCC
73. A 56 years old Egyptian, working in UK presented with painless hematuria ??SCC

April 2018 MRCS A

RK

EMQ on Ear

74. Discharge from ear for a 10 years, with middle ear perf and facial nerve
palsy...cholesteatoma 75. ... ()
76. ... ()
77. Some kind of mass in neck? Don’t rememebr ..........Otitis media with effusion

Stupid EMQ on Jaundice

78. Man with pain, jaundice, fever....had similar episodes of pain abdo
previously....choledocholithiasis (cholangitis was not in options)
79. 30 yrs follow up patient of FAP operated for pancolectomy, now presented with
Jaundice and bilious vomitting... Carcinoma of duodenum
80. ... ()
81. ... ()

EMQ Fluid burn calculation in 24h

82. 36% burn in some 60 or 70 kg


83. 19 yr old with 90 kg weight with burn of a lower limbs and perineum

EMQ chest trauma

i. Chest drain
ii. Chest drain with suction
iii. CT
iv. Echo
v. Thoracotomy

84. Stab to right 4th ICS, normal CXR, tachy, rest normal
85. Stab to the left 5th ICS, tachy, rest normal
86. Stab to 5th ICS post axillary line, CXR showing pneumothorax with fluid level

EMQ Chest pain in pregnancy

87. 36 weeks, 28 yr, c/o Chest tightness and dyspnea. On exam, cyanosis and one more
alarming feature. Dad died of AMI at 62 yrs.....AMI
88. 36 weeks, 28 yrs, father died of AMI at 42 yrs, chest pain, no other info.
Exceptionally(word used was some different) tall..... Aortic Dissection
89. 36 weeks, 28 yrs, Preg lady with pleuritic chest pain followed by dyspnoea, father
died at 60.... PE

EMQ Child gastrointestinal disorders

90. 6 months, sausage shaped mass...intussusception


91. 6 weeks non bile stained vomit after feeds...pyloric stenosis
92. Tender distended abdomen, per rectal bleed in 6 weeks baby......midgut volvulus 93. ...

EMQ GCS
94. E2V2M2

95. E1V1M1
EMQ Ortho patterns of fracture

96. Twisting force... spiral


97. Child fracture with bowing happening after injury greenstick
98. Direct blow to femur... transverse
99. Fracture humerus with metastatic lesion, mechanism was fracture happened while
she was trying to aboard

a bus and got pulled by the running bus oblique

EMQ Ortho nerve injury

April 2018 MRCS A


RK

100. Posterior approach to femur... (tibial? common peroneal? posterior cutaneous nv of


thigh?) (sciatic was not in option)

101. Humerus posterior approach... radial


102. Medial approach to ankle... saphenous nerve

EMQ urethral injury

103. Pelvic fracture, unstable patient, DRE prostate unusual location... rupture
membranous urethra
104. Fall with legs wide open on handle bar bulbar urethra injury

EMQ plastic EMQ Thyroid

105. 1 cm MCT ... total thyroidectomy


106. 0.5 cm nodule found in one lobe of thyroid while operating for another surgery
probably

parathyroid... hemithyroidectomy

EMQ Types of biopsy

107. Medial malleolus Venous ulcer...incision biopsy


108. Solitary lesion in thyroid
109. 1 cm lesion over back, suspected melanoma
110. Weeping lesion over the nipple areolar complex, no palpable mass.

EMQ on Splenomegaly

111. Caucasian man, antibodies to EBV +... (Infectious Mononucleosis)


112. Jaundice, splenomegaly and multiple gall stomes: pigment stones
EMQs:

-Knee Injury

A. B. C. D. E. F. G. H.

Full thickness graft


Partial thickness graft
Pedicled flap
Microvascular free flap Advancement flap
Wound excision and primary closure Wound excision and 2nd ry closure Healing by
secondary intension

61. Patient presented with h/o Dog-bite, part of the nose is lost
62. Cut wound scalp with dirty wound edges
63. Garden fork in injury to the dorsum of foot with around 1cm area around that is dusky
indurated. 64. Patient Presented with burns over around 20% body surface area of which
5% area is deep

A. Medial Collateral Ligament injury B. Lateral Collateral injury


C. Medial meniscus tear
D. Lateral meniscus tear

E. Anterior cruciate Ligament F. Posterior Cruciate Ligament G. Fracture tibial Platue


H. Tibiofibular Dissociation

68. Twisted his Rt knee, as it got stuck in mud while playing football. He continued to play,
but developed swelling in the evening, Presented 2 weeks later with pain around 2 cm
above joint line and excess opening on varus stress testing.

April 2018 MRCS A

RK

69. 35 yr. male with Injury to left knee sustained during a Road traffic accident. There is no
apparent deformity and no fractures, During examination, the flexed position of the knee at
90 degrees tibia is more posterior on left side than right.

1. 1cm melanoma over the back, best management...

1. excision biopsy with 2mm margin,


2. excision biopsy with 2cm margin
3. core biopsy
D.
2. 23-25 yr old female, single mobile breast mass, most appropriate investigation

1. USG
2. Mammo

C.

April 2018 MRCS A

April 2018 Recalls - Reda


Thanks to MRCS telegram and WhatsApp groups, Reda Harby and all the members.
Aim is to try to remember all the questions and correct answers.
‘??’ added to unsure answers.

201. Congenital hernia defect in (pleuroperitoneal membrane)


202. Statistics question… (Man Whitney?? Unpaired T test??)
203. Mass in chest and liver hemangioma, Ix… (MRI liver, PET??)
204. Loss of weight, tiredness, hairy tongue lesion… (lichen, candidiasis, AIDS)
205. Nerve injury, deltoid affection, which other muscle will be affecting
206. Fat child, pain hip… (Slipped femoral epiphyses)
207. 6yo child pain hip (Perthes)
208. Trauma to knee, swelling medial, injury to… (MCL)
209. Injury to knee and swelling, on 90 degrees tibia is more posterior than other
side… (PCL injury)
210. 3 patients in surgical ward with Cl. Difficile, mode of infection… (endogenous,
feco-oral??)
211. Test for rupture tendon achilles… Simmonds test
212. Sinus tarsi… between talus and calcaneus
213. Lift thumb with hand 90 degrees on table… EPL
214. Mobile extensive femoral thrombus management initial management (Heparin,
Venous exploration, Aspirin, Embolectomy??)
215. Which hormone decreases in warm condition… TSH, ACTH, Vasopressin??
216. Cleft palate embryology defect in what… (??)
217. Embryology question??
218. Another embryology question… mesonephric duct??
219. Testicular cancer with normal AFP and HCG… (Spermatocytic seminoma or
classical seminoma??)
220. Relation of median nerve to brachial artery in proximal arm… (lat to ant to
medial)
221. Relation of ulnar nerve to ulnar artery… (ulnar to the ulnar artery)
222. Dorsal root ganglion in neck which type… (sensory)
223. Atrophy thenar eminence… (median nerve)
224. Dermatome of the little finger… (C8)
225. Iatrogenic small injury to esophagus, nutrition option… (TPN)
226. Ix for rupture esophagus after endoscopy… (water soluble contrast??)
227. Reciprocal of absolute risk reduction (NNT)
228. Cushing reflex/triad… (↑BP ↓HR ↓RR)
229. Athlete sudden death after marathon (SAH)
230. 6yo, kid with groin swelling (patent processus vaginalis)
231. Child with irreducible groin mass (indirect inguinal hernia??)
232. Intrinsic pathway… (factor IX??)
233. Old woman displaced # distal radius, reduced and put in slab/POP for 6 weeks,
complaining of pain after 3 months… (CRPS??)
234. Can’t remember… question about cervical lymphadenopathy… carcinoma, which
Ix… CT, MRI??
235. Diarrhea and egg in stools… (mebendazole)
236. What is true about hypercholesteremia… ()
237. Advantage of carbohydrate rich drink in enhanced recovery program… (decrease
post-operative insulin resistance and negative nitrogen balance)
238. What’s in direct contact with T5 body… (azygos, left atrium, aortic arch??)
239. Shoulder pain between 60-120 degrees… (supraspinatus tendinitis)
240. Gun shot to on right side below costal margin, injury to... (pylorus, gallbladder??)
241. UMNL lesion at which level (L1-2, T12-L1??)
242. Chinese man with LN enlargement and headache and deafness (nasopharyngeal
carcinoma??)
243. Spastic LL paraplegia which artery (ACA??)
244. Impotence and vascular insufficiency, clue to Leriche syndrome (occlusion to both
common iliacs??)
245. Femoral canal lateral border… (femoral vein)
246. Diagnosis of DVT by… (compression US)
247. Patient with Barret’s esophagus… (adenocarcinoma)
248. Golfer with 1st metatarsal pain, XR showed narrowed joint space and subchondral
cysts… (OA??)
249. Gas gangrene, causative organism… (Cl. perfringens)
250. Post-partum lady dies after 3 days causative organism… ()
251. Osteomyelitis causative organism… (Staph aureus)
252. Brown sequard $... (ipsi motor and proprioception + contra pain and temp)
253. Hypercalcemia, ECG abnormalities, treatment… (fluid, sodium chloride)
254. Hypothermia ECG change… (J wave)
255. 1cm melanoma, best management… (excision biopsy with 2mm margin, excision
biopsy with 2cm margin??)
256. Young female, breast mass investigation… (US)
257. Site of action of fursemide… (ascending loop of Henle)
258. Site of action of thiazide… (DCT??)
259. Male gynecomastia which drug… (spironolactone)
260. Posterior duodenal ulcer collapse, which artery… (gastroduodenal)
261. Site of Dartos muscle (subcutaneous)
262. Waddling gait, injury to… (superior gluteal nerve)
263. Bypass with graft, later is blocked… (neo intimal hyperplasia)
264. COPD patient… (↑CO2 ↑HCO3)
265. Anterior relation to right adrenal (IVC)
266. Good prognosis for melanoma… (Breslow thickness of 0.6??)
267. Extensive burn patient presents with lower leg edema after 3 days…
(hypoalbuminemia)
268. Picture of x-ray with arrow I think was pointing at (hyoid bone??)
269. Male with pain in testes and urethral discharge… (gonorrhea??)
270. Blood supply to transverse colon… (middle colic artery)
271. ICP calculation
272. Submandibular duct excision injury to… (lingual nerve)
273. Post parotidectomy presents with dryness, which ganglion… (pterygopalatine)
274. Injury to head, fixed dilated pupil, cause… (unopposed sympathetic fibers)
275. Osteoporotic crush to vertebra, serum calcium is likely to be… (hyper, hypo,
normocalcemic??)
276. Fall in blood BP, 1st reaction… (sympathetic system) baroreceptors wasn’t in
option
277. 11yo child with special need, going for laparoscopy, type of consent… (consent
parents??)
278. Ligation of splenic hilum, avoid injury to what… (tail of pancreas??)
279. Ureter stone radiating to groin, which nerve root… (T11-L1??)
280. What structure is in front of uncinate process… (SMA??)
281. What forms the anterior border of the heart (Rt atrium and ventricle??)
282. Addison’s, electrolytes given, which hormone to check (ACTH??)
283. Metastatic bone pain treatment… (radiotherapy, NSAIDs)
284. Ovarian mass and inner thigh numbness, which nerve… (obturator??)
285. Bilateral cervical lymph node spread, which type of cancer… (tongue??)
286. Skin lesion, pearly… (BCC)
287. Skin lesion, central keratinous something… (keratoacanthoma??)
288. UK NICE guidelines on people who received transfusion (can’t donate in future
until viral markers are negative??)
289. Anterior compartment syndrome affecting… (1st dorsal web space??)
290. Pathology of berry aneurysm in 45yo… (atherosclerosis??)

291. Can’t remember, site of lumbar puncture… stupid options…


292. Can’t remember, ulcer tip of penis or something
293. Can’t remember, Ix for pheochromocytoma…
294. Can’t remember, post-operative patient, seemed like DIC scenario
295. Can’t remember, some question about action of Angiotensin II or aldosterone
296. Can’t remember, some guy with a knee swelling and sudden pain then swelling
disappeared…
297. Can’t remember, renal Ca for staging…
298. Can’t remember, post op patient with hyperthyroid and AF or something, ttt…
299. Can’t remember, investigation for parathyroid?
300. Can’t remember, most likely risk factor for HCC… (alcohol??)
301. Can’t remember, some dude with blurring of vision and headaches in the
morning with no focal neuro signs
302. Can’t remember, some scenario I think it was MM, asking for cell of origin
303. Can’t remember, something about the association with hypospadius
304. Can’t remember, some question about diffuse lung infiltrates or ARDS, cause of
clinical picture?
305. Can’t remember some question about lung, vital capacity or something…
306. Can’t remember, something about a mediastinal mass with glandular pattern?
307. Can’t remember, post procedure, loss of sensation and pulse in leg… popliteal I
think
308. Can’t remember, epiploic foramen or free margin of lesser omentum…
309. Can’t remember, a bloke who went a cardiac operation, what increases CO?
310. Can’t remember, injury, numbness lateral arm, which muscle will be affected as
well?
311. Can’t remember, question about heart valve component options had fibrous
tissue, hyaline cartliage…
312. Can’t remember, some question of abdomen relations involving ureters and
common iliac and gonadals…
313. Can’t remember, pain on walking down hill but not uphill… (spinal stenosis??)
314. Can’t remember, patient post burn, with hematemeses… (acute gastric dilatation)
315. Can’t remember, prostate with metastatic disease I think, ttt… (TURP, radical
prostatectomy, incision??)
316. Can’t remember, ABG 1
317. Can’t remember, ABG 2
318. Can’t remember, ABG 3
319. Can’t remember, ABG 4
320. Can’t remember, ABG 5
321. Can’t remember, ABG 6
322. Can’t remember, Electrolyte
323. Can’t remember, Electrolyte
EMQ Bladder
324. Egyptian hematuria… (SCC)
325. Female long history of indwelling catheter and multiple recurrent infections,
presents hematuria… (SCC??)
326. Male in dye industry I think… (TCC)
EMQ on Ear
327. Discharge from ear for a couple of years and facial nerve palsy… ()
328. … ()
329. … ()
330. … ()
Stupid EMQ on Jaundice
331. … ()
332. … ()
333. … ()
334. … ()
EMQ Fluid burn calculation in 24h
335. … (14-16L)
336. … (3-6L, 6-8L??)
EMQ chest trauma
337. Stab to right chest, 5th space, normal CXR, tachy… (CT, ECG, chest tube, echo??)
338. Trauma to chest, pneumothorax… (chest tube??)
339. … ()
EMQ Chest pain in pregnancy
340. Chest tightness… ()
341. Tall preg lady, father died at 40, chest pain, no other info… (Panic attack
Marfanoid??)
342. Preg lady with pleuritic chest pain, father died at 60 … ()
EMQ Child gastrointestinal disorders
343. 6 months, sausage shaped mass… (intussusception)
344. 6 weeks non bile stained vomit after feeds… (pyloric stenosis)
345. …
346. …
EMQ GCS
347. … (score of 6)
348. … (score of 2)
EMQ Ortho patterns of fracture
349. Twisting force… (spiral)
350. Child fracture… (greenstick)
351. Direct blow to femur… (transverse)
352. Fracture humerus with metastatic lesion… (oblique??)
EMQ Ortho nerve injury
353. Posterior approach to femur… (tibial??)
354. Dislocation total hip… (sciatic)
355. Humerus posterior approach… (radial)
356. Medial approach to ankle… (saphenous nerve??)
EMQ urethral injury
357. Pelvic fracture, unstable patient, DRE prostate unusual location… (rupture
membranous urethra)
358. Fall with legs wide open… (bulbar urethra injury)
EMQ LN drainage
359. Scrotum… (superficial inguinal)
360. Ovary… (para aortic)
361. Anal verge (Deep inguinal??)
EMQ plastic
362. Dog bit off nose… (pedicled flap??)
363. Cut wound scalp with dirt… (wound excision and 1ry closure??)
364. Garden fork in toe, 1cm dusky wound… (wound excision and 2ry closure??)
EMQ Thyroid
365. MCT … (total thyroidectomy??)
366. Nodule… (hemithyroidectomy??)
EMQ Types of biopsy
367. Venous ulcer… ()
368. Paget disease of breast… ()
369. … ()

EMQ ABG
370. … ()
371. … ()
372. … ()

EMQ
373. … ()
374. … ()
375. … ()
376. … ()

Jan 2019

1. Schwannoma > High frequency hearin loss tinnitus and vertigo


2. Most medial nerve in the Cubital fossa > Ulnar And Median
nerve
3. In fetal circulation blood moves from right atrium to > Left atrium
4. Nasal-ethmoid fracture in eye eccymosis- Le-FORTE iii
5. Varicocele vs inguinal hernia typical history but I think he trick
was that the left testicle is small so I went with varicocele
6. Gubernaculum
7. Meckel’s Diverticulum location. Gastric mucosa was found in M.
Diverticulum. so I gave gastroduodenum junction.
8. Bitemporal hemianopia lesion site- Optic Chiasma
9. Antiphospholipid syndrome
10. Brown Sequard syndrome
11. FFP
12. ACA in spastic LL paralysis
13. ECG changes in prolonged diarrhoea?
14. Serisanguinous Dx? (IDP, Periductal papilloma, Periductal mastitis)
15. That 80 yr old female with Er and pr negative but her2 positive
and 5 lymph nodes (I chose chemo and Herceptin, ?Herceptin
only)
16. Ova in stool>ascariasis>mebendazole
17. Ear infection followed by headache and mass in cerebellum and fever
(?cerebellar abscess)
18. Cause of pale stool and dark urine (?increased bilirubin, ?decreased
circulation)
19. Morton’s neuroma
20. Metformin-lactic acidosis
21. Callot’s triangle
22. Tennis player with sudden dusky fingers
23. Impotence after prostate surgery
24. Hypocalcaemia ECG
25. Sacoidosis
26. Perineoplastic effects
27. Mental nerve for lower lip numbness
28. Epiglotitis-H-influenza
29. Epiglotitits- clinical diagnosis
30. Vascular shunting and angioplasty
31. Tricky Q about vessel disease.

So he had MI few years ago and his LL are swollen and there are
ulcers on the for dorsum of the foot and abpi was low. I thought
venous system bc of the swelling in the LL but this was when I had
a holy moment and thought that old people with MI probably have
heart failure and so LL swelling and so the answer was

Chronic Obliterative Arterial Disease


32. Testicular torsion
33. Thoracic duct injury: Thoracic duct injury at posterior medisatium
inbetween diphgram and throsic outlet
34. Adisonian crisis first line Tx – Hydrocortisone 100mg
35. Bee sting first line Tx – 0.5ml epinephrine 1:1000 I/M
36. A Q on an obstructed urinary system so without thinking I chose
Nephrostomy to decompress the system. Yes but it was focusing
on normal contralateral kidney, also there was no SIRS... Means
not an emergency..
37. Foramen spinosum
38. Diplopia with cavernous sinus involvement at the lower border of the
sinus – abducent nerve
39. Structure not divided during tracheostomy –
40. Ascending pharyngela artery – arises from the medial aspect of the
ECA
41. T8-T10 dermatome – costal cartilage
42. Sciatic nerve injury – lack of knee flexion
43. Horner’s syndrome
44. Sympathectomy complication
45. Restrictive/obstructive lung disease
46. 4 preventable deaths, what sort of audit? - ?review practice
47. Pneumothorax prior to surgery – ICT
48. AAA 4.7cm – follow up with USS
49. Benign oesophageal stricture – dilatation
50. SA node – vagus
51. Lack of ability to raise voice after thyroidectomy – External laryngeal
nerve
52. AAA, symptomatic, widen mediastinum on CXR, most app next step
– CT urgent
53. Rectal pain, normal inv – proctalgia fugax
54. Elderly lady, multiple co-morbidities, full thickness rectal prolapse,
Mx? Delorme’s procedure
55. 5cm remaining of jejunum, what type of feeding?
56. GCS 14 on admission, drop to 7, next best step? – Secure airway
57. Criteria for intubation? (?RR> 35)
58. What predicts morbidity? (?FVC1 – 3litres)
59. Morphine (Mu receptors for CNS & PNS, Kappa on GIT)
60. Bier’s block
61. Lidocaine with adrenaline for scalp wound
62. Lidocaine alone for toe procedure
63. Ureters insert to the base/posterior of the bladder
64. Cervix lymph drainage? – External Iliac
65. Ovaries lymph drainage? – Para-aortic
66. Anal ca lymph drainage – superficial inguinal
67. Midtarsal fracture? (?Malunion, ?Avascular necrosis)
68. Osteoporosis – normocalcaemia
69. Female from Australia with well defined brown lesions on her back,
one was excised. A. Malignant melanoma B. Neurofibromatosis C.
Seborrheic keratosis (?Seborrheic keratosis)
70. Incisors to oesophagus T10 distance? 40cm
71. Epistaxis and facial swelling - ?nasopharyngeal mass
72. Obstructed ureters - ?nephrostomy and JJ stent
73. Vitamin C for poor healing
74. Left coronary artery
75. Na-K – primary active transport
76. Nose bit off by dog, lost - ?Flap
77. Male 90kg with burns in whole lower limbs and perineum, how much
IV fluids in 24 hours? – 11-13litre
78. Prolonged diarrhea – ECG changes?
79. Diuretics – calcium – bendroflumethazide
80. Crusted nurn on the abdomen 5% -painless? (?Escharatomy, ?Split
thickness, ?Full thickness graft)
81. There was q about postop man becomes unwell and got
bradycardic and widened complex QRS I think choices were
atropine , digoxine , ad , emergency pacemaker. i thought there
was also disso p and QRS, i.e 3rd degree heart block, so i chose
emergency pacing wire
82. Ulnar paradox - Yes because marker clawing (unopposed ulnar
FDP)
83. Q on loose body in the knee? - ?MCL injury
84. Addditional blood supply to territory of IMA ?

MIDDLE COLic ?
85. Was there a q about spondyloitheses? - ?Disk prolapse
(posterolateral)
86. I think there was a q of carotid body tmr / aneyrsm? I chose
tmr, carotid body tumor moves sideways but not up and
down, didn't it mention side to side but not up and down?
87. There was q I think about Hodgkin or non hodgkin and other
choices I can't remember about a man with anaemia, I don’t even
remember what I chose but one of those 2, I think it was NHL , he
had axillary and inguinal lymph node as I recall
88. Free haemoglobin in plasma after aortic and mitral valve
replacement

All choices were sensible, I think acute hemolytic reaction as the


metallic valve will cause hmeolysis, I think I chose that
89. left renal v ant to aorta
patient with combined hrt and patholoical fractures I think was
breast ca
supraspinatous ... lack of abduction from zero, all correct
90. Cells in Tb – macrophages
91. congenital diaphragmatic hernia? – left pleuroperitoneal
membrane
92. there were 2 q of what happen after healing of incisions
choices were margination of neutrophils and decrease vascularity
and other 2 one can't remember, I think one of them is wound
healing after 7 days. It was after 8 weeks what happens ? One was
angiogenesis and the other was devascularization and another
was neutrophils. I think one of them was decreased vascularity as
it was long time after incision. i think so too. Yes dec. Vascularity
93. Bleeding from intercostal arteries – on chest tube insertion
94. Intercostal nerves supply parietal pleura above the diaphragm
95. Q on secondary hyperparathyroidism - Thyroid hyperplasia
96. Melanoma best prognosis - breslow
97. t vaginalis surrounding hydrocele
98. Hyperacusis I think it is facial as dt to injury of n to spadius
99. Question on gastrin,histamine and increased vagal activity
100. Question on post op ileum, ?Increased NA release
101. There was q about cholecystectomy I think it was decreased
max flow after meal
102. Melanoma with 1cm margin, discharge?
103. Pagets>DCIS
104. Bradycardia in heart transplant patient (was going for atropine
but as heart transplant no vagal supply – isoprenaline
105. Sarcoidosis in low PTH and hilar shadowing
106. Sarcoidosis in bilateral parotid swelling
107. Hashimotos most common cause of hypothyroid
108. Superior Sagittarius Sinus bleeding in – vertex
109. Blocked CSF – aqueduct
110. Lumbar puncture last layer punctured – arachnoid matter
111. Young kid with acute testicle pain torsion
112. Indwelling catheter uti sepsis
113. Hypotension with raised jvp?reduced stroke volume, I chose
reduced preload as restrictive process I guess only god and
English doctors know!
114. Fluid restriction for hyponatreamia
115. what was organism of pneumonia not repsonding to ttt in renal
ptn ? there were pseduonomas and CMV, and staph (CMV virus)
116. Muddy casts in urine ATN?
117. There was two about patient with 20 unit transfusion, I think
hypotension because ruptures spleen
118. Renin,Angio ii,aldosterone - all raised
119. in acute bleeding for reference to help other people
120. Parietal pleura – Phrenic nerve
121. 3 day post splenectomy High platelet – Thrombocytosis
122. Descending thoracic aneurysm, lower limb neuropathy &
retention – artery of Adamkiewice (?anterior spinal artery)
123. Sclerosing cholangitis -UC
124. Acutely unwell UC patient with perforation & gas in the
transverse colon - toxic megacolon
125. Crohn’s with low albumin, no vomiting – high protein diet
126. Short gut syndrome – TPN
127. Bulbar palsy – NG feed
128. Chi squared test/Statistical test Mann Witney
129. ACL tear
130. PCI tear
131. Medial meniscus
132. White cold limb after open femur fracture, ?fasciotomy,
?Explore SFA, ?Angio then Urgent vascular surgeon consult
133. Thrombosed graft - ?Thrombolysis
134. Recurrent PE – ?Lifelong warfarin, ?IVC filter
135. Stab to left sternal edge 5th ICS – right ventricle
136. Steroid in Brain ca – reduce oedema
137. Angle of mandible – Great auricular nerve
138. Lingual nerve damage in submandibular gland surgery – loss of
sensation
139. Brachial plexus x 3 questions
140. Trochlear nerve – superior oblique muscle
141. Bleeding on inserting pins in neurosurgery (posterior aspect of
the head) – occipital artery
142. ABG: metabolic acidosis -peritonitis
143. Bradycardia in Cushing’s reflex, brain trauma – Increase in
baroreceptor activity
144. VMA levels in young hypertensive man – pheochromocytoma
145. Discitis – staphylococcous aur
146. Insect bite - ?Staph aur
147. MRSA, CVC line – Tx?
148. Tennis player with emoboli, Investigation? – Duplex?
149. 3/12 post Oespohageal jejunum anastomosis, benign stricture
– Dilate
150. Post-oesophagectomy stricture – endoscopic balloon dilatation
151. Pt on aspirin and clopidogrel, had 5000unit of heparin,
cardiothoracic surgery, normal platelet, post op bleed? –
152. Right cerebral hemisphere infarct – Right ICA stenosis
153. Pain after plaster cast, shiiny skin- complex regional pain
syndrome
154. Tibia fracture 2cm proximal to ankle – plates and screws
155. 7 year old boy – Perthes
156. Split thickness graft for tibia tissue loss
157. Debride and delayed primary closure in shrapnel injury
158. C7 – middle finger
159. Ankle & Toes – L5
160. Periorbital numbness – Low calcium levels
161. Allograft
162. GCS – 3
163. Paraesthesia medial side of the arm – T1
164. Vena cava opening – T8
165. Femoral canal medial boundary – lacunar ligament
166. Near MCP joint index finger, digital median nerve – digital
nerve
167. Level of ureteric stone –
168. Gallbladder at which level - semilunaris
169. A man fell down, now has headache and hyperacusis – CN 8
injury
170. Complex and recurrent anal fistula how to investigate ? –
MRI
171. Ureteric stone with pyrexia, unstable, how to manage? –
nephrostomy
172. Carcinoid syndrome (symptomatic), what to give – Octreotide
173. Norepinephrine derivative that has Beta 1 receptor effect on
low dose and has alpha 1 receptor effect on high doses –
adrenaline
174. NON-SMOKER LADY WITH LUNG MASS, WHICH TYPE OF
TUMOUR? – ADENOCARCINOMA
175. Hilar lymphadenopathy and signs of hypercalcaemia –
sarcoidosis
176. Rapid onset, early dissemination of disease – small cell lung ca
177. Gun shot injury to the lower buttock and injury to the sciatic
nerve, which movement will be affected A. Knee flexion B. Knee
extension C. Hip extension (Answer: Knee flexion)
178. Location of renal artery/vein/ureter – ant to post: vein – art –
ureter
179. Patient with mother and sister both have breast ca (age >35),
which imaging to use to screen the patient? – Mammography
180. Patient has been on 5% dextrose for 3/7 post op, develops
hyponatraemia, what is the cause? – excessive fluid
administration of 5% d
181. A child with 2/7 H/O of nystagmus/ataxia (cerebellar signs),
he was on Abx for a week with no improvement, imaging shows
lesion (hypodense), cause? – cerebellar abscess
182. Pelvic fracture in a child who is dropping Hb, parents refusing,
Consent? – apply in court (not an emergency)
183. Pulsatile swelling near carotid moving side to side, confirmed
with angiogram, diagnosis? – carotid body tumour
184. Woman with colorectal cancer on chemotherapy has developed
abdominal pain and distention, how to proceed? – Further
investigations
185. Pus draining abscess in non-diabetic male with poor oral
hygiene,organism? ?staph aur, ?strep
186. Hematoxylin eosin in histopathology –
187. Blue stain histology in cardiac muscle – haemochromatosis
188. Pt has mets from prostate ca deposted in humerus he holds
the bus bar with his hand as to elevate him self as the bus
starts moving he develops fracture which fracture is it dx
transverse – Transverse fracture
189. 90mmHg mean arterial pressure, what pressure of blood is
supplied to the brain – 15%
190. Which of the following is not a part of the early recovery
program in caecal tumor? – NG tube
191. Pt with fracture, on azathioprine, prednisolone, chloroquine,
cause? – prednisolone
192. Palm on table, can’t lift thumb: - rupture of extensor pollicis
longus
193. On flexion of the thigh the foot goes externally rotated –
slipped capital femoral epiphysis
194. RTA, foot everted with no distal pulses -shaft fracture
195. Old female, radiolucent tumour on diaphysis near knee joint –
giant cell tumour
196. Unable to extend fingers, wrist extension partially
compromised with no sensory loss – post interosseous nerve
197. Child with cystic fibrosis passes a mucous plug on DRE, he now
has intestinal obstruction symptoms – meconium ileus
198. Cystic fibrosis – chloride sweat test
199. Patient brought to hospital, prophylaxis for ongoing bleeding –
wearing gloves
200. Ulcerative colitis symptoms with enlarged transverse colon on
X-ray – Toxic megacolon
201. Ulcerative colitits with filing defects in hepatic biliary tree on
PTC – PSC
202. Down syndrome pt, who can fully understand – consent the
patient
203. Cyanotic congenital heart – transposition of great vessels
204. Right Pulmonary and left arch derived from – 6th arch
205. Acute transplant rejection – T lymphocyte
206. HCC, AFP dumping – insulin release
207. Nasopharyngeal ca cause – EBV
208. Mediastinal mass with Hascall’s corpuscle is found in –
Thymoma
209. Motor neuroma - when nerve gets entrapped in fibrous tissue
mostly at 2nd, 3rd metatarsal
210. Sterilisation method for flexible sigmoidoscope –
Gluteraldehyde
211. Herpes zoster, area of pain distribution – subcostal margin
212. Remnant of urachus – median umbilical ligament
213. Pt with Colle’s fracture with co-morbidities, which anaesthetic
technique to treat the pain? – Bier’s block
214. Lateral cutaneous nerve of the thigh – meralgia paraesthetica
215. Nerve supply for root of penis and upper scrotum –
Illioinguinal nerve
216. Increasing traumatic coagulopathy – (?Hypoxia,
?Hyperthermia, ?Hypokalaemia)
217. Pernicious anaemia – anti parietal cell antibodies
218. Normal external sphincter tone on DRE but still incontinent –
internal anal sphincter dysfunction
219. Spina bifida – myelomeningocele
220.

You might also like